[表示 : 全て 最新50 1-99 101- 201- 301- 401- 501- 601- 701- 801- 901- 1001- 2ch.scのread.cgiへ]
Update time : 09/23 17:11 / Filesize : 316 KB / Number-of Response : 1016
[このスレッドの書き込みを削除する]
[+板 最近立ったスレ&熱いスレ一覧 : +板 最近立ったスレ/記者別一覧] [類似スレッド一覧]


↑キャッシュ検索、類似スレ動作を修正しました、ご迷惑をお掛けしました

大学学部レベル質問スレ 17単位目



1 名前:132人目の素数さん [2021/11/21(日) 08:00:44.31 ID:4j6fBnFe.net]
大学で習う数学に関する質問を扱うスレ

・質問する前に教科書や参考書を読むなりググるなりして
・ただの計算は
wolframalpha.com
・数式の表記法は
mathmathmath.dote ra.net
・質問のマルチポストは非推奨
・煽り、荒らしはスルー

関連スレ
分からない問題はここに書いてね478
rio2016.5ch.net/test/read.cgi/math/1511604229/

※前スレ
大学学部レベル質問スレ 16単位目
https://rio2016.5ch.net/test/read.cgi/math/1619727449/

96 名前:132人目の素数さん [2021/12/29(水) 00:25:09.93 ID:N4hESVZE.net]
上の式と下の式が等価であることの証明(説明)聞きたいです。
直感的にはそうだと思うのですが、ちゃんと納得できない状態です。
イプシロン-デルタ論法ぐらいまでは理解しています。

lim x -> a (f(g(x)) - f(g(a))) / (g(x) - g(a))

lim (x -> g(a)) (f(x) - f(g(a))) / (x - g(a))

f(x), g(x)ともに全ての実数xにおいて微分可能と仮定してください。
x, aはともに実数です。

97 名前:132人目の素数さん [2021/12/29(水) 00:26:24.94 ID:N4hESVZE.net]
式を間違えてました。

lim x -> a (f(g(x)) - f(g(a))) / (g(x) - g(a))

lim (x -> g(a)) (x - f(g(a))) / (x - g(a))

f(x), g(x)ともに全ての実数xにおいて微分可能と仮定してください。
x, aはともに実数です。

98 名前:97,98 [2021/12/29(水) 00:30:12.49 ID:N4hESVZE.net]
間違えてませんでした。
97が正しい式です。

99 名前:96,97,98 [2021/12/29(水) 00:31:16.37 ID:N4hESVZE.net]
うわぁああ一個ずれてました。
初めの式(96)が正しいです。

100 名前:132人目の素数さん [2021/12/29(水) 05:28:18.04 ID:jRSjeZwm.net]
>>96
>上の式と下の式が等価
等価とは?イコール?恒にではなく定義されるときイコール?値だけで無く発散の状況についてもという意味?

101 名前:132人目の素数さん [2021/12/29(水) 05:35:09.31 ID:jRSjeZwm.net]
定数関数g(x)=bだと上は定義されずfが微分可能だから下は定義されるのでこういうのは除外?とすると両者とも極限値が確定する場合にそれが一致することを等価?

102 名前:132人目の素数さん [2021/12/29(水) 05:50:24.00 ID:N4hESVZE.net]
定義されるときにイコールという意図でした。
連鎖律の証明で上の式から下の式へ当然のように置き換えられていたのがずっと気になっていたので質問しています。

103 名前:132人目の素数さん [2021/12/29(水) 07:17:03.87 ID:jRSjeZwm.net]
fが微分可能だからlim(f(y)-f(g(a)))/(y-g(a))=f'(g(a))=kとする
h(y)={(f(y)-f(g(a)))/(y-g(a));y≠g(a), k:y=g(a)}は
y≠g(a)で微分可能だから連続
limh(y)=f'(g(a))=k=h(g(a))
だからy=g(a)でも連続なので連続関数
gが微分可能だから連続関数で
h(g(x))も連続関数の合成だから連続関数
limh(g(x))=h(g(a))=k
これで納得行かない場合は
そもそもx→aで(f(g(x))-f(g(a)))/(g(x)-g(a))が定義されない点がaの周りに集積している状況での
lim(f(g(x))-f(g(a)))/(g(x)-g(a))
の意味を考える

104 名前:132人目の素数さん mailto:sage [2021/12/29(水) 14:28:07.10 ID:A2yLiDwT.net]
つのだ⭐じろう



105 名前:132人目の素数さん [2021/12/31(金) 02:21:37.24 ID:VRx/Zu0n.net]
よろしくお願いします。

フーリエ級数では、coskxの重み付けA_kやsinkxの重み付けB_kのフーリエ係数を用います。
ただし、波数k=0の要素波専用のフーリエ係数導出式A_0がありました。A_0の項も用いて級数表示していました。

しかし、フーリエ変換の公式を導いている途中、周期無限大にした結果、波数に基づく無限級数が積分で表示できるようになるのですが、その論理展開で登場するフーリエ係数がA_kとB_kのものだけになっています。A_0の項については、フーリエ変換ではどこへ行ったのでしょうか。

フーリエ係数導出式A_0は、0以外のフーリエ係数A_kの半分の大きさなので区別してきました。

106 名前:132人目の素数さん mailto:sage [2021/12/31(金) 08:52:41.09 ID:xeMJjnAr.net]
式書けや

107 名前:132人目の素数さん [2021/12/31(金) 15:48:05.00 ID:Gk6GR3xs.net]
>>105
A_0に由来する項の極限値が0である場合にしか定義できない

108 名前:132人目の素数さん [2021/12/31(金) 21:57:30.19 ID:Zh7YfBqI.net]
>>107
ありがとうございます。
もう少しでわかるかもしれません。

周期無限大で、A_0に由来する項の極限値が0になるから、フーリエ変換ではA_0は描かないということでしょうか。

A_k(k not equals 0)の項は、三角関数があるので周期関数に寄与しています。
しかし、A_0は周期性はないものの変換対象の関数全体を持ち上げる役割を果たしているので必要な項なのではないかとも思うわけです。

参考書は、極限に至る途中までA_0の項を分けて考えていたのに、周期の極限を取って、シグマを波数kの積分にした途端、なんの言及も与えられず、A_0の扱いが同解決したのかわからず困っています。

109 名前:132人目の素数さん [2021/12/31(金) 22:14:23.61 ID:pbTkRYup.net]
>>108
A_0に由来する項の極限値が0で無いと
すべて上手く行かないんだよ

110 名前:132人目の素数さん [2021/12/31(金) 22:15:04.91 ID:pbTkRYup.net]
定義ができるだけのためにも極限値が0で無くてはいけない

111 名前:132人目の素数さん [2021/12/31(金) 23:26:08.44 ID:iQBfD0rx.net]
>>108
書名を明示せよ

112 名前:96 [2022/01/01(土) 05:38:24.28 ID:SyoL23h1.net]
>>103
ありがとうございます。

113 名前:132人目の素数さん [2022/01/03(月) 07:24:38.97 ID:Zhn98PrS.net]
>>109-110
ありがとうございます。
わかってきました。

フーリエ係数A_0には、フーリエ変換したい関数f(x)の-∞から∞までの区間積分が含まれます。
(coskxで、k=0のため、1になっている)
A_0 = 1/L ∫ f(x) dx

そして、周期L→∞なので、この区間積分は0になるという理解で良いでしょうか。
-∞から∞までの区間積分 ∫ f(x) dxが、無限大に発散しないことがいかに保証されるのかが曖昧なのですが、
仮にf(x)=tとすれば、区間積分の計算は[1/2t^2](-∞から∞)になるので、すでに0になります。
たとえこれが0にならなかったとしても、∞に発散することはないと考えると、1/L(L→∞)が効いて、
A_0 = 1/L ∫ f(x) dxは、0になりそうです。

つまり、周期無限大で考えるフーリエ変換では、A_0が0として考えるという理解であってますでしょうか。

114 名前:132人目の素数さん mailto:sage [2022/01/03(月) 07:26:42.34 ID:Zhn98PrS.net]
>>113
訂正
× f(x)=t
○ f(x)=x



115 名前:132人目の素数さん [2022/01/03(月) 07:47:42.67 ID:n5vv1gOi.net]
>>113
何もわかってない
テキストを明示せよ
調べるから

116 名前:132人目の素数さん mailto:sage [2022/01/03(月) 10:00:11.51 ID:xhXdejvo.net]
>>113
絶対可積分でないとダメだよ

117 名前:132人目の素数さん mailto:sage [2022/01/04(火) 10:15:13.00 ID:3x8KnSk3.net]
>>94
分野が違う場合はしかたがないが、
数学という同じ分野で、
違うモノに別な名前つけちゃいかんだろ
んで、
フィルターっての"?しとるもの"
フィルトレーションは"?しとる行為"
この基本的な部分をそれぞれ別な意味にあてはめなんてそれは混乱のもとでしょーが
https://www.azumi-filter.co.jp/technical/filtration/

118 名前:132人目の素数さん mailto:sage [2022/01/04(火) 10:19:05.44 ID:3x8KnSk3.net]
"こしとる"
ってのを漢字つかうとはねられた

あとさ、トポロジーを位相って訳したのいったいどこの馬鹿たれなん?
数学以外、高校生以降
位相 == phaseであることは日本では動かしようのない事実なのに、
phaseと同じ訳考えた奴をぶち殺してやりたい

119 名前:117 mailto:sage [2022/01/04(火) 10:20:34.08 ID:3x8KnSk3.net]
訂正
違うモノに"同じ"名前


120 名前:132人目の素数さん mailto:sage [2022/01/04(火) 10:23:14.96 ID:BD7WZIXM.net]
馬鹿程自説に拘る

121 名前:132人目の素数さん [2022/01/04(火) 11:45:56.94 ID:0es+HySJ.net]
複素正則行列と書いてあると少し混乱する

122 名前:132人目の素数さん [2022/01/04(火) 12:17:28.21 ID:LbGLZrrs.net]
>>117
>違うモノに別な名前つけちゃいかんだろ
別に〜

123 名前:132人目の素数さん [2022/01/04(火) 12:19:17.30 ID:LbGLZrrs.net]
>>117
そもそも
>>94
>違うものに同じ用語が使われているだけ
は正しいのか?
同じものだから同じ用語なのでは?

124 名前:132人目の素数さん [2022/01/04(火) 13:34:30.92 ID:W1DIFU4j.net]
とぽろぎー=位相(いぞう)
ふぇいず=位相(いそう)
実は違う



125 名前:132人目の素数さん mailto:sage [2022/01/04( ]
[ここ壊れてます]

126 名前:ホ) 19:21:38.86 ID:TGo52aKJ.net mailto: 他スレで質問したのですが全くレスが付かず
そもそもスレチだったのではと思ったのでマルチですみませんがこちらで聞かせてもらいます

ミルナーのモース理論の以下の記述が分かりません
何を読めばわかるとかでも構わないので分かる人いたら教えて下さい(和訳だとp111です)

Mをリーマン多様体とすると
断面曲率K(U,V)は「光学」の術語で言い表せる
観測者をp∈Mとし,そこから単位ベクトルU∈TMp方向にある1点q=exp(rU)を見る
単位ベクトルW∈TMpに対応するqにおける長さLの小さな線分は,観測者には長さ
L(1+r^2/6*K(U,V))+(rの高次のベキ)
に見える
[]
[ここ壊れてます]

127 名前:132人目の素数さん [2022/01/04(火) 22:12:50.69 ID:OOF/tp1r.net]
>>116
ありがとうございます。
絶対可積分については物理の書籍では端折られていました。
区間積分が収束するという理解で、
受け入れることにしたいと思います。

128 名前:132人目の素数さん mailto:sage [2022/01/05(水) 00:49:28.34 ID:PtIs0pFf.net]
濾し取る

129 名前:132人目の素数さん mailto:sage [2022/01/05(水) 00:50:39.58 ID:PtIs0pFf.net]
書けたじゃん。濾すじゃなくてサンズイに鹿の方で書こうとしたのか?

130 名前:132人目の素数さん [2022/01/05(水) 11:41:54.70 ID:2zDh0XT0.net]
>>125
曲率一定の球面の測地線で考えてみたら?

131 名前:132人目の素数さん mailto:sage [2022/01/07(金) 16:53:28.72 ID:vqj4Lf3R.net]
物理学ではなく純粋に数学の質問です。
深谷賢治「電磁場とベクトル解析」P25補題1.34に書いてある所で疑問があるので質問します。物理学ではなく純粋に数学の質問です。

補題1.34 LをR^2の部分集合とすると、次の2つの事は同値である。
(i)Lは滑らかな曲線の和である。
(ii)任意の点p∈Lに対して、pからε未満の距離にあるLの点の全体、{q∈L│‖q-p‖<ε}が滑らかな閉曲線である様な、ε>0が存在する。

この補題は、滑らかな曲線の和は必ず閉曲線になっている部分集合を含んでいて、閉曲線を部分集合に持たないLは滑らかな曲線の和ではない事になりませんか?閉曲線がある様なεが存在すると言っているので閉曲線なければεは存在しないので。

だとしたらR^2内のどこまでも真っ直ぐな直線は滑らかではないという事になるので矛盾する気がするんですが、この補題は間違っているんですか?

どう読み替えればこの矛盾が解消出来るのか、分かった人がいたら教えて欲しいです。

132 名前:132人目の素数さん mailto:sage [2022/01/07(金) 17:00:25.87 ID:VFwr+WEv.net]
物理学ではなく純粋に数学の質問です。

133 名前:132人目の素数さん [2022/01/07(金) 17:05:08.40 ID:gdCexZlR.net]
滑らかな曲線⇔滑らかな閉曲線の一部分をいくつか持ってきて繋げたもの

とか言いたかったんじゃないの?
主張の“文章”の意味が取りにくくて色んな意味に取れてしまう事などよくある
そういう時はその補題が本当は何を言いたいのかはその補題がその先でどんなシチュエーションで使われて主張のどの部分を使ってるのか見て判断するしかない

134 名前:132人目の素数さん mailto:sage [2022/01/07(金) 17:05:21.83 ID:vqj4Lf3R.net]
文の順番を入れ替えたんですけど、コピペしたあとに元の文章を消し忘れました。強調してるみたいになってますが違います。



135 名前:132人目の素数さん mailto:sage [2022/01/07(金) 17:07:00.61 ID:vqj4Lf3R.net]
>>132
なるほど!取りあえずそう思って読んでみます。

136 名前:132人目の素数さん [2022/01/07(金) 17:08:17.76 ID:gdCexZlR.net]
じゃあわからんわ
その補題そのものだけ見ても何言いたいかなんかわかるはずない
数学の世界で誰もが使う有名な補題ならわかるだろうけど、その本の著者が自分の趣味で何回も使うステートメントをまとめただけのものならその本持ってる人間でなきゃわからんよ

137 名前:132人目の素数さん [2022/01/07(金) 17:09:41.98 ID:gdCexZlR.net]
おっと前の解釈で良かったのかな?

138 名前:132人目の素数さん mailto:sage [2022/01/07(金) 17:11:02.66 ID:vqj4Lf3R.net]
>>135
133のレスは131番さんへのレスです。噛み合ってない気がしたので多分何か誤解させてたらすみません。

139 名前:132人目の素数さん [2022/01/07(金) 17:18:00.86 ID:gdCexZlR.net]
らじゃ

140 名前:132人目の素数さん [2022/01/07(金) 19:43:36.69 ID:2oIbknmg.net]
何で深谷賢治さんに聞かない

141 名前:132人目の素数さん [2022/01/07(金) 20:04:56.25 ID:2k0Yky3g.net]
幾何学者はなぜいい加減な本を書く人が多いのでしょうか?

142 名前:132人目の素数さん mailto:sage [2022/01/07(金) 23:40:31.92 ID:q6INQ6pa.net]
数学科学部一回生がやる解析学の厳密性で足踏みしてるというより地団駄踏んでるような奴の言い張る厳密性(笑)

143 名前:132人目の素数さん [2022/01/08(土) 12:40:23.55 ID:7rWowuSH.net]
インパクトはあるけどギャップもある論文を書いて物議を醸したのでしょ。
そういうかたに、直接聞きにくいだろ。

144 名前:132人目の素数さん mailto:sage [2022/01/08(土) 18:44:16.98 ID:6IMw4/d/.net]
>>129
まずはそこから考えてみるのが良さそうですね
ありがとうございます



145 名前:132人目の素数さん mailto:sage [2022/01/08(土) 23:32:48.63 ID:1cCTS6sV.net]
>>130 その箇所だけ提示して本を持ってない人に教えてもらおうったって無理な話ですよ

p.21 滑らかな開曲線(curve)であるとは, 次の定義をみたす無限回微分可能な
写像 l : (a,b) → R^2 が存在することをいう. ...

p.23 注意1.30
お互いに交わらない曲線の有限個の和集合を曲線の和と呼ぶ. 単に曲線の和という場合は,
無限個の和である場合もあるが, 本書ではそういう場合はでてこない. ...

p.23
f: R^2 → R なる無限回微分可能関数に対して (中略)
L = { p∈R^2 | f(p) = c } が滑らかな曲線の和であるための条件 ...

この辺りを踏まえれば 補題 1.34 の 「滑らかな閉曲線」 は誤植で
単に「滑らかな(開)曲線」の事を言ってるんだろうと分かる. 著者に聞くまでもない.
なのでこの本の定義に限って言えば
曲線: x^2 - y^2 = 1 は「滑らかな曲線の和」だけど x^2 - y^2 = 0 は「滑らかな曲線の和」ではないと言える.

146 名前:132人目の素数さん [2022/01/14(金) 12:47:52.02 ID:0hiyD0FF.net]
ある命題が選択公理なしで証明できないことってどうやって証明するの?

147 名前:132人目の素数さん [2022/01/14(金) 13:00:25.74 ID:m4J6HRnN.net]
強制法?

148 名前:132人目の素数さん [2022/01/14(金) 14:55:26.56 ID:+tH78VeM.net]
>>145
一般には極めて難しい
例えばバナッハタルスキの逆理は選択公理より真に弱い仮定のもとで成り立つ。
これが証明されたのはバナッハタルスキの論文の50年以上あとである。

149 名前:132人目の素数さん mailto:sage [2022/01/14(金) 23:19:41.75 ID:a/3DXRPT.net]
>>141
何の生産性もない社会の穀潰し数学科のエタのお前が、非人差別してどーするよゲラゲラ

150 名前:132人目の素数さん mailto:sage [2022/01/15(土) 04:52:36.65 ID:lP/M2Ihp.net]
屠??殺業者だったら穀を潰すのじゃなく畜獣を潰して精肉や皮革を加工品として生産するような仏教的な殺生や神道的な不浄の伴う生産性だな。
皮肉でもなんでもなく。

151 名前:132人目の素数さん [2022/01/16(日) 09:02:23.76 ID:Kn2jwhMr.net]
スティルチェス積分のモチベーションを教えてください。何に使えますか?

152 名前:132人目の素数さん [2022/01/16(日) 09:06:22.65 ID:vQFCEajs.net]
線積分は?

153 名前:132人目の素数さん [2022/01/16(日) 11:30:02.49 ID:gJMMAKNB.net]
>>150 確率論で使われるよ。清水良一『中心極限定理』(教育出版)では、
ルベーグ・スティルチェス積分を縦横無尽に駆使して、中心極限定理を証明しています。
(リンデベルグ条件との同値性を含めて。)

154 名前:132人目の素数さん mailto:sage [2022/01/16(日) 13:57:41.04 ID:fAc4h/Do.net]
教養教育の数学の問題が分かりません。
三角形ABCの中に点Pを取った時にAB+AC>PB+PCを証明したいのですが誰か教えてください。



155 名前:132人目の素数さん mailto:sage [2022/01/16(日) 14:04:30.19 ID:fAc4h/Do.net]
分からない問題は別にスレがありました
スレチすいません

156 名前:132人目の素数さん [2022/01/18(火) 12:58:03.41 ID:q+gv6zKC.net]
代数学の問題で
C[x,y]/(x^2-y^3)の素イデアルの求め方を教えてください。

157 名前:132人目の素数さん [2022/01/18(火) 13:45:55.65 ID:zAAUfQDg.net]
割れなさそうなやつを探す

158 名前:132人目の素数さん mailto:sage [2022/01/18(火) 14:28:44.32 ID:4PsfNjkd.net]
>>155
整域だからまず0,でなければ極大.これは(x-a,y-b)の形.(a,b)は曲線x^2-y^3=0上の点.

159 名前:132人目の素数さん [2022/01/19(水) 13:21:19.51 ID:y+VYIrau.net]
三角関数の最もいい定義ってなんだと思う?学部1年生の微分積分学の授業でやるとして。

160 名前:132人目の素数さん [2022/01/19(水) 13:33:06.56 ID:Cvmwu/OB.net]
あなた高校生の質問スレッド荒らしてませんか?

161 名前:132人目の素数さん mailto:sage [2022/01/20(木) 21:41:29.10 ID:Z7V8Vw9r.net]
>>149
済まん屠??殺業者と表示された
2文字目3文字目は何だ?牛?豚?絲?

あー青椒牛絲じゃなくて青椒肉絲を山盛りで食いてー

162 名前:132人目の素数さん mailto:sage [2022/01/23(日) 23:51:34.17 ID:fjKrQdm2.net]
テンソルの特異値分解で
T_{i,j,k,l }=Σ_m U_{i,j,m} V_{k,l,m}
ということができるようなのですが
テンソルの特異値分解は色々あるようで、検索しても上記の形の分解についての説明が見つからなかったのですが、上記の分解により詳しい名前ついてますか?

163 名前:132人目の素数さん mailto:sage [2022/01/23(日) 23:55:49.03 ID:fjKrQdm2.net]
>>161
式がおかしかったです
T_{i,j,k,l }=Σ_m U_{i,j,m} V_{k,l,m}W_m
です

164 名前:132人目の素数さん mailto:sage [2022/01/24(月) 19:20:39.32 ID:94pwReUH.net]
>>162
この形の分解の解説が見つかった訳ではないのですが
{i,j}を一つの添字、{k,l }を一つの添字とみなして、Tを行列と思って特異値分解



165 名前:キるということだろうなと一応自分の中で解決しました []
[ここ壊れてます]

166 名前:132人目の素数さん [2022/01/25(火) 18:17:59.18 ID:608rLxi9.net]
>>163
もちそれよ

167 名前:132人目の素数さん mailto:sage [2022/01/25(火) 19:56:12.31 ID:Pnsv2z2P.net]
https://www.youtube.com/watch?v=CaCsEpvPESs&t=561s

ヨビノリの複素関数論で、arg z に多価性もたせない定義してるっぽいんだけど、
こういう流儀って割とあるんでしょうか?

arg z は、極座標表示の θ の事を arg z と書くというような説明で、
動画中で厳密な定義などは話してなかったと思います。

log z = log |z| + i arg z + 2 n Pi i

という書き方で、arg z に多価性があるとすると、
2 n Pi i は冗長な表記になりますよね。

ちなみに、 arg z の主値 Arg z は arg z とは別に定義しています。

168 名前:132人目の素数さん mailto:sage [2022/01/25(火) 20:41:07.33 ID:laio/Hsv.net]
そもそも多価関数って考え方自体いにしえの概念だと思う
リーマン面導入する前段階の話じゃないの

169 名前:132人目の素数さん mailto:sage [2022/01/25(火) 20:44:43.30 ID:laio/Hsv.net]
議論したい範囲で使えればこだわらなくていいと思う
時間の無駄だし

170 名前:132人目の素数さん [2022/01/25(火) 22:30:28.14 ID:608rLxi9.net]
>>165
多かろうが少なかろうが別にどうでんよか

171 名前:132人目の素数さん mailto:sage [2022/01/26(水) 20:39:57.26 ID:h/uMv5LT.net]
(1)しか解けませんでした
解答教えて下さい
https://i.imgur.com/FRjVQ2X.jpg

172 名前:132人目の素数さん [2022/01/28(金) 07:53:26.28 ID:Ug3UlzNf.net]
>>33
これってみんな解析接続とか使って回答してるけど複素数使わないと証明できないの?

173 名前:132人目の素数さん [2022/01/28(金) 08:51:57.79 ID:WCHxZV0z.net]
カルタンの「複素函数論」のやりかたなら
複素数を使わなくてもできる。

174 名前:132人目の素数さん [2022/01/28(金) 12:13:17.41 ID:OutyWaGG.net]
>>170
a中心で収束半径無限大だからほぼ自明



175 名前:132人目の素数さん mailto:sage [2022/01/28(金) 12:16:06.32 ID:ZAPkf+5g.net]
>>54

176 名前:132人目の素数さん [2022/01/28(金) 12:25:37.46 ID:OutyWaGG.net]
>>173
どうやっても収束するからに決まってんじゃん

177 名前:132人目の素数さん [2022/01/28(金) 12:27:58.90 ID:OutyWaGG.net]
整級数について勉強すれば?

178 名前:132人目の素数さん mailto:sage [2022/01/28(金) 14:17:27.44 ID:ZAPkf+5g.net]
>>175
勉強不足ですまない
f:R→Rがすべてのxに対して点aを中心としてテイラー展開可能で、
∞ = sup{|x-a| | Σf^(n)(a)(x-a)^n/n!が収束する}だから、
∞ = sup{|x-b| | Σf^(n)(b)(x-b)^n/n!が収束する}
であって、
fのbでのテイラー級数が収束するというのは自明として、
fのbでのテイラー級数が元の関数fに一致することはどう証明するんだろう

179 名前:132人目の素数さん [2022/01/28(金) 14:22:48.93 ID:xZ7ulfde.net]
OutyWaGGは無頓着に和の変形しちゃいけないって知らないんだよ

180 名前:132人目の素数さん [2022/01/28(金) 15:12:36.16 ID:OUYsDMrD.net]
フーリエ級数だと(収束しても)元の関数に収束するとは限らないことは誰でも知ってるし注意深く扱うのに、何故テイラーだと(収束すれば)無条件に元の関数に一致すると思い込むのか

181 名前:132人目の素数さん [2022/01/28(金) 17:20:10.77 ID:OutyWaGG.net]
>>177
噴飯
どこもかしこも絶対収束するのにね

182 名前:132人目の素数さん [2022/01/28(金) 17:21:28.57 ID:OutyWaGG.net]
>>176
>勉強不足ですまない
自明に感じるまで勉強するんだ

183 名前:132人目の素数さん [2022/01/28(金) 18:10:30.31 ID:4e7+btyS.net]
>>180
なるほど、こいつは“勉強”した結果、何が自明で何が非自明か判断できなくなっちゃったんだな

184 名前:132人目の素数さん [2022/01/28(金) 19:07:11.93 ID:OutyWaGG.net]
>>178
だから
整級数の理論を学べよ



185 名前:132人目の素数さん [2022/01/28(金) 19:08:51.74 ID:OutyWaGG.net]
>>181
揚げ足取ってやったって悦に入ってどうするんだろうね

186 名前:132人目の素数さん mailto:sage [2022/01/28(金) 19:35:51.73 ID:c67AN9z0.net]
理論を学ぶ必要があるとすれば自明じゃないんだが

187 名前:132人目の素数さん mailto:sage [2022/01/28(金) 19:57:56.40 ID:9K70lIw2.net]
ここで質問する人は概ね理論を学んでる最中の人だと思うが、
理論を学んでようやく分かる自明なら「自明」と返されても理解できる可能性はあまりない

188 名前:132人目の素数さん [2022/01/30(日) 20:22:48.97 ID:7/fDKnK3.net]
トーラスと断面がメビウスの輪になるトーラス(*1)は位相同型ですか?
断面がメビウスの輪になるトーラスとアンビリックトーラス(*2)は位相同型ですか?

*1
x = (R + r * cos(θ / 2)) * cos(θ)
y = (R + r * cos(θ / 2)) * sin(θ)
z = r * sin(θ / 2)
r < R
web1.kcn.jp/hp28ah77/fig18_49.gif

*2
x = (7 + cos((u / 2) - 2v) + 2 * cos((u / 3) + v)) * sin(u)
y = (7 + cos((u / 2) - 2v) + 2 * cos((u / 3) + v)) * cos(u)
z = sin((u / 3) - 2v) + 2 * sin((u / 3) * v)
https://upload.wikimedia.org/wikipedia/commons/thumb/a/ad/Umbilic_torus.png/250px-Umbilic_torus.png

189 名前:132人目の素数さん [2022/01/30(日) 20:56:05.51 ID:AebVW8ek.net]
>>186
>断面がメビウスの輪になるトーラス
断面とは?

190 名前:132人目の素数さん [2022/01/30(日) 21:09:29.49 ID:4cbMh+2L.net]
知らんけどトーラスって名前を付けるなら同相じゃないとおかしいんじゃないの?

191 名前:132人目の素数さん [2022/01/30(日) 21:34:45.56 ID:X/LjiTq7.net]
エスパーすると、ファイバー束 T^2->S^1 の同型のことを言ってる気がしないでもない

192 名前:132人目の素数さん [2022/01/30(日) 21:41:32.12 ID:7/fDKnK3.net]
>>187
すみません、分かりません。
メビウスの輪を回転させて作ったねじれたトーラスと
円を回転させて作ったトーラスは同相じゃないんじゃないかと思ったんです。

>>188
同相じゃない図形でも回転させると同相になるってことですか?

>>189
多分それです。

193 名前:132人目の素数さん [2022/01/30(日) 21:48:19.78 ID:AebVW8ek.net]
>>190
>メビウスの輪を回転させて作ったねじれたトーラ
メビウスの輪って?2次元のメビウスの帯とは違うよね?

194 名前:132人目の素数さん [2022/01/30(日) 21:49:02.67 ID:7/fDKnK3.net]
>>191
メビウスの帯です。



195 名前:132人目の素数さん [2022/01/30(日) 22:23:39.24 ID:AebVW8ek.net]
>>192
メビウスの帯をどう回転させるの?
2次元の曲面を回転させたら3次元じゃ無いの?

196 名前:132人目の素数さん [2022/01/30(日) 22:29:15.81 ID:AebVW8ek.net]
あと
回転ってのはR^3の中にあるメビウスの帯をR^3の何らかの回転軸の直線に関して回転させるってこと?
R^3でそんなことしてもトーラスにはならないから違うと思うけど回転ってどういうことをするんだろ

197 名前:132人目の素数さん [2022/01/30(日) 22:30:55.82 ID:7/fDKnK3.net]
>>193
穴が開いてるのをy軸だとするとy軸を中心に回転、と思いましたけどそれだと緯線が円になるので
y軸を中心とした円を軸にめくれるように回転、ですかね。
すると緯線が元の場所にたどりつくまでに同じ経線を2度通るので
同相ではないのではないかと思いました。

198 名前:132人目の素数さん [2022/01/30(日) 22:34:24.92 ID:7/fDKnK3.net]
>>194
これって少なくともy軸に関して回転させたらトーラスにならないですか?なると思ってました。
https://ja.wikipedia.org/wiki/%E3%83%A1%E3%83%93%E3%82%A6%E3%82%B9%E3%81%AE%E5%B8%AF#/media/%E3%83%95%E3%82%A1%E3%82%A4%E3%83%AB:M%C3%B6biusband.png

199 名前:132人目の素数さん [2022/01/30(日) 22:35:17.01 ID:7/fDKnK3.net]
URL訂正
https://upload.wikimedia.org/wikipedia/commons/c/ce/M%C3%B6biusband.png

200 名前:132人目の素数さん [2022/01/30(日) 22:37:57.39 ID:AebVW8ek.net]
>>195
y軸を中心とした円を軸にめくれるように回転ってどういうこと?
トーラスならそれ(円を軸にめくれるように回転)分かるけど
メビウスの帯はトーラスの表面の一部にはならないんだけど

201 名前:132人目の素数さん [2022/01/30(日) 22:39:38.31 ID:AebVW8ek.net]
>>196
>y軸
どれがy軸?下から上に穴の中を通る直線?
これに関して回転させても3次元の立体になるだけでは?

202 名前:132人目の素数さん [2022/01/30(日) 22:41:49.36 ID:7/fDKnK3.net]
>>199
意味がわかりました。確かに立体になって表面にはならないですね。
トーラスと中身が詰まったトーラスは違うんですね。

203 名前:132人目の素数さん [2022/01/30(日) 23:00:54.27 ID:MleoNhVD.net]
中身詰まってるのはソリッドトーラスという

204 名前:132人目の素数さん [2022/01/30(日) 23:06:38.73 ID:AebVW8ek.net]
メビウスの帯をR^3の中じゃ無くて回転させるというかスライド?させてクラインの壺とか射影平面にはできるけどね



205 名前:132人目の素数さん [2022/01/30(日) 23:21:36.19 ID:AebVW8ek.net]
射影平面はできないわ
メビウスの帯と円だった

206 名前:132人目の素数さん mailto:sage [2022/01/31(月) 00:12:05.15 ID:BTa9xoAM.net]
口と尻…人はクラインの壺なのかな?鼻の穴も有るから同相ではないのかな?
鼓膜が抜けてしまった人も更に位相化するのかな?

207 名前:132人目の素数さん [2022/02/02(水) 01:02:19.19 ID:Z5CXm6Kz.net]
ピアスで貫通させれば同相ではない

208 名前:132人目の素数さん [2022/02/02(水) 22:40:48.54 ID:Z5CXm6Kz.net]
SMプレイでひらめいたのが結び目理論

209 名前:132人目の素数さん [2022/02/05(土) 11:03:42.66 ID:udbflrBw.net]
T_0で局所コンパクトな位相群Gは位相空間として見た時に距離付け可能ですか?
反例が存在するなら教えて頂けないでしょうか

210 名前:132人目の素数さん [2022/02/08(火) 20:39:12.04 ID:oxroEDOE.net]
>>207
複素数体の乗法を入れた位相群 S^1 の非可算無限個の直積が反例になりそうですね.

211 名前:132人目の素数さん [2022/02/08(火) 22:10:42.12 ID:VCtZeaFC.net]
集合からコンパクト距離空間への写像全体の集合は
距離空間になるのでは?

212 名前:132人目の素数さん [2022/02/08(火) 23:25:14.39 ID:dVGxjT5N.net]
なったとしてそれがなんなの?

213 名前:132人目の素数さん [2022/02/09(水) 09:09:43.67 ID:k5OOz8m1.net]
>>210
208でなきゃ黙っとけ

214 名前:132人目の素数さん [2022/02/09(水) 09:13:58.02 ID:HxJTIiJt.net]
距離空間→第一可算公理を満たす



215 名前:132人目の素数さん [2022/02/09(水) 11:02:31.85 ID:k5OOz8m1.net]
可分性よりはずっと弱い

216 名前:132人目の素数さん [2022/02/09(水) 11:15:52.58 ID:M5TRq19L.net]
そういや無限直積のポントリャーギン双対ってなんになるんやろ
例えば>>208の双対ってなんやろ?

217 名前:207 mailto:sage [2022/02/09(水) 18:27:17.39 ID:Xfx3Ldkb.net]
>>208
GはT_0かつコンパクトな位相群になり、
また第一可算公理を満たさないから距離付けは不可能
反例ありがとうございます

218 名前:132人目の素数さん [2022/02/12(土) 17:11:07.12 ID:Jhxmm3sY.net]
有限群Gのシロー部分群が、すべて巡回群のとき、剰余群G/D(G)のシロー部分群も
巡回群ですか?(D(G)はGの交換子群とする)
よろしくお願いします。

219 名前:132人目の素数さん [2022/02/12(土) 17:16:46.96 ID:FQFEDxwe.net]
π:G→G/Dを自然な射影、PをGのpシロー群とするときG/Dのpシロー群はπ(P)
Pが巡回群ならπ(P)も巡回群

220 名前:132人目の素数さん [2022/02/12(土) 17:27:56.06 ID:Jhxmm3sY.net]
>>217
ありがとうございました。

221 名前:132人目の素数さん [2022/02/13(日) 15:22:41.43 ID:wXviuiRD.net]
有限群Gの位数がn=Πpi,piは異なる素数でi≠jならpiはpjを法として1と合同でない
という条件を満たすとするとGは巡回群となりますか?
よろしくお願いします。

222 名前:132人目の素数さん [2022/02/13(日) 17:04:34.15 ID:M6HN3dPC.net]
位数2の元が有れば他の素因子pがあればp≡1(mod2)で仮定に反する
よって位数が偶数ならC2
位数が奇数、素数位数でなければFeit-Thompsonにより素数位数の正規部分群Cpを持つ
Cpに属さない素数位数qの元xのCへの自己共役作用が非自明なら単射<x>→AutCが誘導されるがこの時q | p-1 によりp≡1 (mod q)
よってCはGの中心
以下ry

223 名前:132人目の素数さん [2022/02/13(日) 17:31:13.89 ID:uhuvFRS8.net]
Dickson
Definitions of a group and a field by independent postulates,
https://www.ams.org/journals/tran/1905-006-02/home.html
のセクション5の定理の十分条件
とその位数のアーベル群は巡回群しかないことからわかる

224 名前:132人目の素数さん [2022/02/13(日) 18:04:55.48 ID:M6HN3dPC.net]
え?コレ論文になるほどの話?
>>220間違ってる?
それともFeit-Thompson使わない初等的証明って話?



225 名前:132人目の素数さん [2022/02/13(日) 18:09:44.60 ID:M6HN3dPC.net]
>>220は正規部分群の取り方が逆だな
しかしそれでもindexが素数の正規部分群取れるんだから話一緒になるよな?

226 名前:132人目の素数さん [2022/02/13(日) 18:22:45.03 ID:uhuvFRS8.net]
1905年の論文だし、まだ群論のことをよく知らない数学者もいた時代

227 名前:132人目の素数さん [2022/02/13(日) 18:31:28.89 ID:M6HN3dPC.net]
納得
Feit-Tompson 1960年らしいからな

228 名前:219 [2022/02/13(日) 21:36:01.70 ID:wXviuiRD.net]
>>220-225
ありがとうございました。勉強します。

229 名前:132人目の素数さん [2022/02/14(月) 00:03:19.92 ID:IbRhtz/L.net]
Feit-Tompsonとか持ち出すアホの言うことは聞かない方がいいぞ

230 名前:132人目の素数さん [2022/02/14(月) 00:23:53.87 ID:z8q70iXP.net]
>>227
じゃあお前とけや

231 名前:132人目の素数さん [2022/02/14(月) 00:26:54.23 ID:z8q70iXP.net]
そもそもコイツスペルミスしたレスの方コピペしとるww
アホやんwwwww

232 名前:132人目の素数さん [2022/02/14(月) 02:30:26.26 ID:IbRhtz/L.net]
そういうことではなくて証明に何百ページも費やすような大定理を使っていることがアホと言っている

233 名前:132人目の素数さん mailto:sage [2022/02/14(月) 09:30:06.73 ID:owPpBcJR.net]
でも証明できてるなら良いじゃん

234 名前:132人目の素数さん [2022/02/14(月) 11:10:01.54 ID:z8q70iXP.net]
>>230
ではその大定理を使わない証明をどうぞ



235 名前:132人目の素数さん [2022/02/14(月) 12:49:39.45 ID:IbRhtz/L.net]
それが>>221
全部読む必要はない
セクション5の十分条件のところだけ

236 名前:132人目の素数さん [2022/02/14(月) 13:28:05.30 ID:z8q70iXP.net]
>>233
ではその部分“だけ”というのを上げて下さい
Feit-Thompsonを使えば5、6行のレスで済む話をそんな“大定理”を使わずともアッサリ初等的に証明できるのか書いてみて下さい
もちろんそれが可能だから人の事“アホ”呼ばわりしたんでしょ?

237 名前:132人目の素数さん [2022/02/14(月) 15:20:20.51 ID:IbRhtz/L.net]
>>234
Dicksonの定理より(n,φ(n))=1ならば位数nの群は同型を除いてただ1つ
以上

238 名前:132人目の素数さん [2022/02/14(月) 15:38:52.21 ID:z8q70iXP.net]
出ました自分が勉強したとこまでは常識でそれ以上は大定理病wwwww

239 名前:132人目の素数さん mailto:sage [2022/02/14(月) 15:58:28.99 ID:DsAR9be/.net]
「Dicksonの定理」が初等的な定理かどうかが問題。

240 名前:132人目の素数さん [2022/02/14(月) 16:43:06.54 ID:IbRhtz/L.net]
フェイトトンプソンが大定理なのは衆目の一致するところ

241 名前:132人目の素数さん [2022/02/14(月) 16:43:44.73 ID:IbRhtz/L.net]
>>237
原論文のリンクを貼ってあるが

242 名前:132人目の素数さん [2022/02/14(月) 16:54:18.32 ID:z8q70iXP.net]
まぁ何言ってもダメやな
現代数学なんぞ全てのジャンル全ての大定理の証明に目を通しておくなんて不可能
当然査読論文誌で保証されてるから結果だけ使わしてもらうと言う部分が出てくるのは当然
もちろんそのジャンルの専門家ならダメやけどな
Ducksonの定理は初等的で全ての数学者が目を通しておくべき論文なんて言えるはずなかろうに
だったらFeit-ThompsonだろうがDicksonだろうが目クソハナクソにしかならん
結局自分が勉強したジャンルが数学の全てで自分が読んだ事ある論文追えてないやつはバカとか思ってる能無しの戯言

243 名前:132人目の素数さん [2022/02/15(火) 04:36:10.64 ID:ScQ3xlkV.net]
「堀田良之著 代数入門ー群と加群ーのp68の定理12.1にて単因子の一意性が
示されているが、定理12.3(PID上の有限生成加群の構造定理)の一意性の証明に、
定理12.1の一意性ではなく、別の証明方法で一意性が示されている。
定理12.1の一意性が定理12.3の一意性の証明に使えない理由は何ですか?」
という質問がありました。

これに対して
「定理12.1の一意性は行列Fが与えられた時のもの。
一方、定理12.3の証明中に
「f:R^n → R^mを自然な基底で表示した行列をF∈Mm,n(R)とおく。」とあるが、
このFが一つに決まることは示されてない。
証明でMの生成系をx1,x2,..,xm としているが、生成系の取り方は1つだけではなく、
生成系が変わればgも変わる。Kergの生成系の取り方も1つだけでなく、
それによってfも変わり、それによってFも変わるから。」という回答がありました。
この回答は基本的に正しいと考えてよいでしょうか?

244 名前:132人目の素数さん [2022/02/15(火) 12:26:17.62 ID:y4udoMBu.net]
どこにあったQ&A?



245 名前:132人目の素数さん mailto:sage [2022/02/15(火) 12:52:54.68 ID:OJO2d6qJ.net]
数学の本スレって終わったのか?
どっか代替スレで続いては居ないのか?

246 名前:132人目の素数さん [2022/02/15(火) 13:09:50.70 ID:dCOANLno.net]
ちゃんとあるよ
前後にへんなのつけるようになったけど

247 名前:132人目の素数さん mailto:sage [2022/02/15(火) 13:11:24.88 ID:4nfrzJjw.net]
>>241
そもそもPID上の有限生成加群の構造定理より前に単因子が出る本って近年あるんだろうか
間違ってはいなくとも、あまり古い本を読むと困ったところで躓くな

248 名前:132人目の素数さん [2022/02/15(火) 13:17:31.41 ID:zaM3b9cK.net]
「杉浦光夫著 解析入門II」の補助定理9.3について。
留数定理を使った実積分の計算に関する補助定理です。
以下が原文の主張です。

f(z) が扇型 D = {re^{it} ; 0 ≦r≦ a, α≦t≦β} 上で
lim_{|z| → ∞, z ∈ D} z f(z) = 0
をみたし円弧 A(ρ) : z(t) = ρe^{it} (α≦t≦β) 上で f は連続のとき次の式が成り立つ。
lim_{ρ → ∞} ∫_{A(ρ)} f(z) dz = 0

D の定義の a は ρ の間違いだと思われますが、それだけではなく、D は有界なので lim_{|z| → ∞, z ∈ D} z f(z) = 0 という仮定は常に満たされると思います。

証明を見るに、 |f(z)| の A(ρ) 上での最大値を M(ρ) としたとき lim_{ρ → ∞} ρ M(ρ) = 0 が成立すれば主張は正しいですが、どのように修正するのがよろしいでしょうか?

249 名前:132人目の素数さん [2022/02/15(火) 14:54:45.18 ID:yRj4zjCc.net]
Feit-ThompsonとかDicksonとかいうのは何ページくらいで証明できるの?学部程度の知識を仮定して

250 名前:132人目の素数さん [2022/02/15(火) 15:38:39.80 ID:zxR7Un2e.net]
>>247
Feit-Thompsonは数百ページなんじゃないか?
相当長い間正しい事の確認が取れたと数学界でコンセンサスが得られなかった定理だからな
Dicksonの定理は上のレスでは数行だけど正直あの数行で理解するのは難しい
学部生向けの教科書に書くなら2,3ページはいるんじゃない?

251 名前:132人目の素数さん [2022/02/15(火) 21:31:00.85 ID:FooM0eyk.net]
>>248
サンクス
上のバトルはDicksonの勝ちだな

252 名前:132人目の素数さん [2022/02/15(火) 22:06:56.46 ID:zxR7Un2e.net]
まぁそれでDicksonの勝ちってんならそれでいいよ
そんなバカ数学の世界にいらん
はよ出てってくれ

253 名前:132人目の素数さん mailto:sage [2022/02/15(火) 22:10:35.58 ID:lmpt/4Co.net]
どっちの定理使おうが証明できてるならどっちも勝ちだよ

254 名前:132人目の素数さん [2022/02/15(火) 22:22:33.06 ID:zxR7Un2e.net]
まぁ勝ちでも負けでもいいわ
質問に答えて話かも分からずいきなりバカ呼ばわりしてきたカスのかた持つようなやつ数学の世界にいてほしないわ
早よでてけ



255 名前:132人目の素数さん [2022/02/15(火) 23:45:57.31 ID:tOnX6n5y.net]
>>239
>原論文のリンク
群の定義の1_1,1_2,1_3の意図がよく分からないや
直後に1に集約してるし
ところで
右単位元(xe=x)と右逆元(xy=e)の存在だけで両側単位元と両側逆元であることが示せるんだな
知らなかった(または忘れてた)
xに対しy,zを
xy=yz=e
となるものと定義し
yx=yxe=yxyz=yez=yz=e
ex=exe=xyxyz=xyez=xyz=xe=x


256 名前:132人目の素数さん [2022/02/16(水) 01:30:18.47 ID:6yvTmTUr.net]
論文たしかにわけわからんな
専門家が見たら当たり前に飛ばしてる行間があるんやろ
とりあえずSL(2,F) (FはchF=p<∞の代数的閉体)の部分群Gの極大部分群の分類とかいうのがあってそれ見てたらできそうだけど、またバカだのなんだの言われそうだからロムしとくわ

257 名前:132人目の素数さん [2022/02/16(水) 01:44:41.57 ID:6yvTmTUr.net]
ちなみにコレ
lup.lub.lu.se/student-papers/record/8998907/file/8998908.pdf
この分類定理使えばいけそうやけどな
まぁそんな解法はバカなんやろな
賢い解答上がってくるの期待しとくわ

258 名前:132人目の素数さん [2022/02/16(水) 05:48:03.85 ID:gikwIy9G.net]
ID:z8q70iXP
ID:zxR7Un2e
ID:6yvTmTUr
言葉使いに知性の低さが現れている
数学の能力は低くなさそうなので残念だ

259 名前:132人目の素数さん [2022/02/17(木) 20:11:49.54 ID:zk0NUaki.net]
線型空間(ベクトル空間)について質問なんだけど、
例えば「偶数全体」っていう集合は和とスカラー倍に関して閉じているから、R上の線型空間だといえる ってことであってる?

260 名前:132人目の素数さん mailto:sage [2022/02/17(木) 20:18:29.51 ID:pdt2mYRm.net]
>>257
スカラー倍で閉じてないだろ

261 名前:132人目の素数さん [2022/02/17(木) 20:18:37.42 ID:8MqyaEz4.net]
(1/2)*2 が偶数に見えるのならあってるかもしれない

262 名前:132人目の素数さん mailto:sage [2022/02/18(金) 00:25:59.22 ID:E9ImNuSN.net]
整数絡めたいならZ加群とかそういうの考えた方が良さそう

263 名前:132人目の素数さん [2022/02/18(金) 01:02:44.93 ID:c3AMEtnC.net]
ただのアーベル群という罠

264 名前:132人目の素数さん [2022/02/18(金) 01:14:10.13 ID:punXMhyZ.net]
>>258 >>259
あ、そっか、1\2倍もスカラー倍か
不得手なもので...ありがとうございました!



265 名前:132人目の素数さん mailto:sage [2022/02/26(土) 14:39:32.27 ID:J+8ZV7dP.net]
雪江先生の代数学に
(1)Qの有限次拡大体を代数体という
(2)Lを代数体、Ωを代数的整数環(CのうちZ上整なものの集合)とするとき、L∩ΩをLの整数環という
とありますが、代数体Lは常にCの部分体とみなせて、そうみなした場合のL∩ΩをLの整数環というという意味でよいでしょうか?

266 名前:132人目の素数さん mailto:sage [2022/02/26(土) 20:35:48.19 ID:RXaUygON.net]
よい

267 名前:132人目の素数さん mailto:sage [2022/02/27(日) 01:49:02.46 ID:S9NDN6oL.net]
>>264
ありがとうございます

268 名前:132人目の素数さん [2022/03/03(木) 01:59:15.68 ID:ANlVnbXn.net]
小林昭七「接続の微分幾何とゲージ理論の」P45の(2.15)式ってなんでD^2φ=R∧φになるんですか?
(2.5)によれば、と書いてあるんですが、(2.5)の説明から(2.15)までの行間が分かりません。

この本を持っていなければ、
https://www.google.co.jp/url?sa=t&rct=j&q=&esrc=s&source=web&cd=&cad=rja&uact=8&ved=2ahUKEwjmn4Ov0af2AhVFs1YBHSC-DscQFnoECAkQAQ&url=http%3A%2F%2Fwww.math.tsukuba.ac.jp%2F~tasaki%2Flecture%2Fln2000%2F2000t.pdf&usg=AOvVaw0GTWFs1cuMA2tSmdd8OKji
このリンク(田崎博之さんの微分幾何のpdf)のpdfのP67(サイトのページ数では69)の真ん中くらいの
d^∇d^∇φ = d^∇d^∇(φ1φ2) = (d^∇d^∇φ1) ∧ φ2 = (R^∇φ1) ∧ φ2 = R^∇ ∧ φ1φ2 = R^∇ ∧ φ
この式の (R^∇φ1) ∧ φ2 = R^∇ ∧ φ1φ2この部分の質問です。

269 名前:132人目の素数さん [2022/03/03(木) 02:03:19.65 ID:ANlVnbXn.net]
www.math.tsukuba.ac.jp/~tasaki/lecture/ln2017/diffgeo2017.pdf
リンクがちゃんと貼れてなかったので再送信します。
また送れていない場合は、「田崎博之 微分幾何学 pdf」でググると多分1番上に出てきます。

270 名前:132人目の素数さん mailto:sage [2022/03/03(木) 09:03:13.96 ID:g6h29mqj.net]
αを任意の実数とする。
この時、任意の0〜9の有限列はαの十進小数展開の中に見出すことができる

これは真ですか?

271 名前:132人目の素数さん [2022/03/03(木) 09:45:16.32 ID:Xs/1esYn.net]
1/3=0.333... には(0と)3以外出てこないよ

272 名前:132人目の素数さん mailto:sage [2022/03/04(金) 04:02:35.11 ID:Y+W5uF8D.net]
>>268
訂正
「αを任意の無理数とする。」

273 名前:132人目の素数さん mailto:sage [2022/03/04(金) 04:03:15.06 ID:Y+W5uF8D.net]
>>270
訂正します
「αを任意の超越数とする。」

274 名前:132人目の素数さん mailto:sage [2022/03/04(金) 04:28:50.15 ID:Vcg2+F3M.net]
つhttps://ja.m.wikipedia.org/wiki/%E3%83%AA%E3%82%A6%E3%83%B4%E3%82%A3%E3%83%AB%E6%95%B0



275 名前:132人目の素数さん [2022/03/07(月) 13:57:18.46 ID:Va66C910.net]
>>266
詳しいことは分からないが、直前に
「これより R∇ = d∇d∇ ∈ Ω2(M ; EndE) とみなすことができる。」
と書いてあるのは見ているのですか?

276 名前:132人目の素数さん [2022/03/08(火) 19:27:47.80 ID:onvGb6T0.net]
群の表示を機械的に得る方法ってないですか?ガロア群の計算で、生成元に課される関係式を説明無しにポンと出して、「この関係式から二面体群D4に同型であることが分かる」と書かれているんですが、すぐに分かる方法とかあるんでしょうか?

277 名前:132人目の素数さん [2022/03/08(火) 19:29:43.49 ID:FlzLgAId.net]
自明だから

278 名前:132人目の素数さん mailto:sage [2022/03/08(火) 19:33:28.69 ID:JGNVKvhn.net]
>>274
良く分かんないけどティーツェ変換とかとかその辺の話?
https://en.wikipedia.org/wiki/Tietze_transformations

279 名前:132人目の素数さん [2022/03/08(火) 19:39:17.47 ID:yTD6aKsd.net]
雪江明彦著『代数学1群論入門』

K = <x, y | x^3 = y^2 = 1, y*x*y = x^{-1}> とすると、

K = S_3 = D_3 であることを証明せよ。


これって証明する必要があることですか?
自明ではないですか?

280 名前:132人目の素数さん mailto:sage [2022/03/08(火) 19:56:23.85 ID:UxsRhYfi.net]
コレを自明で切って捨てていいと思ってるなら群論の教科書に手を出せるレベルにはない

281 名前:132人目の素数さん [2022/03/08(火) 20:21:47.30 ID:yTD6aKsd.net]
>>277

σ := (1, 2, 3)
τ := (1, 2)

とすると、

σ^3 = ()
τ`^2 = ()
τ*σ*τ = σ^{-1}

が成り立ちます。

そして、この3つの関係式だけを使って、 S_3 のCayley Tableを完成させることができます。

確かに、 y^i * x^j ((i, j) ∈ {0, 1{ × {0, 1, 2}) がすべて異なるかと言われると証明が必要な気もしますが。

K を考える意味って何ですか?

282 名前:132人目の素数さん [2022/03/08(火) 20:23:54.99 ID:yTD6aKsd.net]
松坂和夫さんの本には、自由群、生成元と関係式について全く記述がないですね。

松坂さんは、綺麗に説明しにくいことは書きたがらないように思います。

283 名前:132人目の素数さん [2022/03/08(火) 20:32:29.92 ID:yTD6aKsd.net]
K = F_n / N

F_n をつくるときにまず、同値関係で類別しています。
K をつくるときにさらに同値関係で類別しています。

分かりにくくないですか?

雪江さんは、語とその語の同値類を区別しないなどと宣言しています。
さらに、F_n / N の元は、 x*N (x は語の同値類)ですが、これも語と区別しないなどと宣言しています。

宣言すれば何でもOKなどと思っているのではないでしょうか?

284 名前:132人目の素数さん [2022/03/08(火) 20:36:22.13 ID:D0IqaTd6.net]
さすが松坂くん、今日も冴えまくってますね!



285 名前:132人目の素数さん mailto:sage [2022/03/08(火) 20:37:30.02 ID:UxsRhYfi.net]
>>279
ほら、わかってない
そうやって相手に言われた毎にいちいちカチンときて間抜けな話書いてるレベルの人間性しかないからいつまで経ってもわからないんだよ
そしてお前が今問題にしてる事こそ圏論におけるユニバーサリティの話なんだよ
その1番大切な重要ポイントを“自明”と言って切り捨ててしまってるから何やってもだめなんだよ
その大元がお前がいつまでも抜け出せない小学生みたいな人間性や
諦めろ
お前に学問を修める才覚はない

286 名前:132人目の素数さん [2022/03/08(火) 20:46:29.83 ID:qoofTIPc.net]
>>274
有限群のプログラムあるやン
何て言ったっけ

287 名前:132人目の素数さん [2022/03/08(火) 20:46:55.96 ID:UhWa7JZA.net]
>>274
有限群のプログラムあるやン
何て言ったっけ

288 名前:132人目の素数さん [2022/03/09(水) 16:34:51.63 ID:tzkVTZvd.net]
群Gの極大正規部分群Nと組成部分群HでHがNに含まれないとき、NH=Gだと思うの
ですが、どう示せばよいのでしょうか?
よろしくお願いします。

289 名前:132人目の素数さん mailto:sage [2022/03/09(水) 16:55:58.71 ID:QohnJgNI.net]
GじゃないならH極大ちゃうやん

290 名前:132人目の素数さん mailto:sage [2022/03/09(水) 17:00:01.73 ID:tzkVTZvd.net]
>>287
すみません。HがNに含まれないときは、極大ですか?

291 名前:132人目の素数さん [2022/03/09(水) 17:11:49.44 ID:tzkVTZvd.net]
G⊃G1⊃G2⊃・・・⊃H
G⊃A


292 名前: でAはHを含まないという場合はないのですか? []
[ここ壊れてます]

293 名前:132人目の素数さん mailto:sage [2022/03/09(水) 17:14:01.00 ID:zKx5QXYQ.net]
まだ群論に手出すの早すぎるんじゃないの?

294 名前:132人目の素数さん [2022/03/09(水) 17:19:17.41 ID:tzkVTZvd.net]
わかりました。考え直します。



295 名前:132人目の素数さん [2022/03/09(水) 17:52:57.89 ID:DEX6kRo8.net]
>>286
NH={nh : n∈N⊂G, h∈H⊂G}でGは積で閉じているので、NHの任意の要素はGの元であり、NH⊂Gであることが示せた
HはGの部分群であり、単位元を持つので、N={n1 : n∈N, 1∈H}⊂NHであり、更に⊂Gである
Nは極大なので、
N=NHまたはNH=Gである…@
HはNに含まれないので、HにありNにない元hを取ると、h∈NHだがh∈Nではなく、ゆえにN≠NH
これと@から、NH=Gである

296 名前:132人目の素数さん [2022/03/09(水) 18:23:02.85 ID:q5VBbH12.net]
>>286
G⊃NH⊃N
NH=G or NH=N
NH⊃H
NH¥N⊃H¥N≠φ
NH≠N
G=NH

297 名前:132人目の素数さん [2022/03/09(水) 19:00:21.95 ID:NMYN85ET.net]
Michael Artin著『Algebra 2nd Edition』

この本では、二面体群 D_n が <x, y | x^n, y^2, xyxy> と同形であるということは明らかであるという考えですね。

<x_1, …, x_n | r_1, …, r_k> の定義の直後に、

「Thus the dihedral group D_n is isomorphic to the group <x, y | x^n, y^2, x*y*x*y>.」

と書いてあります。

298 名前:286 [2022/03/09(水) 19:07:32.34 ID:tzkVTZvd.net]
Gの極大正規部分群は正規部分群の中で極大なもので、Gの極大部分群とは
限らないと思います。
それから、自己解決しました。

299 名前:132人目の素数さん mailto:sage [2022/03/09(水) 20:28:27.55 ID:NtKKk1/f.net]
>>294
生成元とリレーションの考え方がまるでわかってない、そしてまるでわかってないという指摘を受けて悔し紛れに本を読んで反論っぽい事を試みる
全てがクズ
お前に学問の話されるだけで虫唾が走る

300 名前:132人目の素数さん [2022/03/09(水) 21:02:14.57 ID:AZvQOM9b.net]
>>294
その前のページに
For example, the dihedral group Dn of symmetries of a regular n-sided polygon is generated by the rotation x with angle 2π/n and a reflection y, and these generators satisfy relations that were listed in (6.4.3):
(7.10.2) x^n =1,y^2 =1,xyxy= 1.
と書いてある。

お粗末にも程がある。

301 名前:132人目の素数さん [2022/03/09(水) 21:35:14.52 ID:NMYN85ET.net]
>>297

それだけでは、 D_n と <x, y | x^n, y^2, xyxy> が同形であることの証明にはならないというのが正しいのではないでしょうか?

例えば、寺田至・原田耕一郎著『群論』では、 D_n と <x, y | x^n, y^2, xyxy> は同形であることを証明しています。

302 名前:132人目の素数さん [2022/03/09(水) 21:36:07.21 ID:NMYN85ET.net]
>>297

証明には半ページくらい使っています。

303 名前:132人目の素数さん [2022/03/09(水) 21:54:54.65 ID:G3/jqlUu.net]
えっ

304 名前:132人目の素数さん [2022/03/09(水) 22:01:23.63 ID:AZvQOM9b.net]
proposition 6.4.3を見てないでしょ
有限群にしか通用しない荒っぽい論証だとは思うが



305 名前:132人目の素数さん mailto:sage [2022/03/09(水) 22:04:05.12 ID:4k8d6X/3.net]
あれ?
でも大先生によると積分表示までは間違いない

∫1/√(u(1-u)(1-x^2u))du from 0 to 1
= 2K(x)
(ただし一般的なK(x) = 大先生ではK(x^2))

https://www.wolframalpha.com/input?i=%E2%88%AB1%2F%E2%88%9A%28u%281-u%29%281-x%5E2u%29%29du+from+0+to+1&lang=ja

x∫(1-u)^(-1/2)(1-x^2u)^(-1/2)du from 0 to 1
= atanh(x)

https://www.wolframalpha.com/input?i=x%E2%88%AB%281-u%29%5E%28-1%2F2%29%281-x%5E2u%29%5E%28-1%2F2%29du+from+0+to+1&lang=ja

306 名前:132人目の素数さん mailto:sage [2022/03/09(水) 22:04:22.94 ID:4k8d6X/3.net]
誤爆
orz

307 名前:132人目の素数さん [2022/03/09(水) 22:23:13.93 ID:NMYN85ET.net]
>>301

その命題は単に、 D_n = {x^i * y^j | (i, j) ∈ {0, 1, …, n-1} × {0, 1}} と書けるということを言っているだけですよね。

Artinさんは、 D_n が <x, y | x^n = y^2 = 1, y*x = x^{n-1}*y> と同形であることは証明していません。

308 名前:132人目の素数さん [2022/03/09(水) 22:32:53.18 ID:AZvQOM9b.net]
いや、ちゃんと証明になっている

309 名前:132人目の素数さん [2022/03/09(水) 22:33:25.87 ID:NMYN85ET.net]
F を集合 S = {x, y} 上の自由群とする。
R = {x^n, y^2, x*y*x*y} ⊂ F とする。
N を R を含む最小の F の正規部分群とする。

Artinさんは、 F/N が D_n と同形であることを証明していません。

310 名前:132人目の素数さん [2022/03/09(水) 22:36:38.99 ID:NMYN85ET.net]
>>305

F を集合 S = {x, y} 上の自由群とする。
R = {x^3, y^2, x*y*x*y} ⊂ F とする。
N を R を含む最小の F の正規部分群とする。

雪江明彦さんは、 F/N が D_3 と同形であることを証明しています。

雪江明彦著『代数学1群論入門』のp.113の例題4.6.6を見てください。

311 名前:132人目の素数さん [2022/03/09(水) 22:44:44.30 ID:AZvQOM9b.net]
 <x, y | x^n, y^2, xyxy>からD_nへの全射準同型があって位数が同じだから単射。
どこがわからない?

312 名前:132人目の素数さん mailto:sage [2022/03/09(水) 22:47:54.96 ID:NMYN85ET.net]
>>308

Artinさんはその全射準同型について述べていません。

313 名前:132人目の素数さん [2022/03/09(水) 22:51:11.85 ID:NMYN85ET.net]
雪江明彦さんの本では、そのような全射準同型の存在は定理として書いてあります。

314 名前:132人目の素数さん [2022/03/09(水) 22:52:16.62 ID:NMYN85ET.net]
Artinさんはそのようなことを論じず、単に、明らかだと言っているようなものです。



315 名前:132人目の素数さん [2022/03/09(水) 23:01:05.28 ID:AZvQOM9b.net]
>>309
>>297に書いてあるじゃん

316 名前:132人目の素数さん [2022/03/09(水) 23:02:07.31 ID:ZPbyjoTC.net]
Artinさんの本を読むレベルに達していないということです
安心してください

317 名前:132人目の素数さん mailto:sage [2022/03/09(水) 23:09:58.72 ID:eXqQmm7/.net]
その半ページ位の証明の中にその群の位数が2nって書いてあるって話じゃないの?

318 名前:132人目の素数さん [2022/03/10(木) 22:04:31.63 ID:dx5fHmpW.net]
森田康夫著『代数概論』

G を群とする。
S を G の部分集合とする。
F(S) を S で生成される自由群とする。
W(S) を S 上の語の集合とする。

W(S) ∋ (s_1, s_2, …, s_n) → s_1 * s_2 * … * s_n ∈ G

は、 (s_1, s_2, …, s_n) の同値類のとり方によらずに定まり、

i : F(S) → G

なる群の準同型を与える。

特に R ⊂ F(S) とするとき、 R の類を含む最小の F(S) の正規部分群が Ker(i) となるなら、 R の元を = 1 とおいたものの集合を、 G の生成元 S に
関する基本関係とよぶ。


「R の類」って何ですか?

319 名前:132人目の素数さん [2022/03/10(木) 23:04:02.90 ID:tEwUO+1Q.net]
松坂くんは今日も通常運行

320 名前:132人目の素数さん [2022/03/11(金) 10:37:53.84 ID:IEqHSrBq.net]
結局、「free groups, generators, relations」について一番分かりやすく丁寧に説明している本は何ですか?

321 名前:132人目の素数さん [2022/03/11(金) 10:58:40.82 ID:IEqHSrBq.net]
こういう当たり前のような当たり前じゃないような話って面倒ですね。

322 名前:132人目の素数さん [2022/03/11(金) 11:07:29.88 ID:IEqHSrBq.net]
Todd - Coxeter Algorithmってどうですか?

323 名前:132人目の素数さん [2022/03/11(金) 11:23:07.45 ID:IEqHSrBq.net]
Michael Artin著『Algebra 1st Edition』のほうがMichael Artin著『Algebra 2nd Edition』よりも説明が分かりやすいですね。

324 名前:132人目の素数さん [2022/03/11(金) 11:28:32.88 ID:IEqHSrBq.net]
Michael Artin著『Algebra 1st Edition』の説明は雪江明彦さんの本での説明と同様ですね。



325 名前:132人目の素数さん [2022/03/11(金) 14:31:37.93 ID:IEqHSrBq.net]
あ、やっぱり、Artinさんの本の説明も分かりにくいところがあります。

---------------------------------------------------------------------------------
命題8.1:
F を集合 S = {a, b, …} 上の自由群とする。
G を群とする。
f : S → G とする。
f は F から G への準同型 φ に一意的に拡張される。


S 上の自由群から G への準同型 φ が全射であるとき、 S は G を生成するという。

---------------------------------------------------------------------------------

S の各元は自由群の元であって、 G の元でないにもかかわらず、 S が G を生成するというのはおかしくないですか?
そのことについての説明が全くありません。

326 名前:132人目の素数さん [2022/03/11(金) 14:48:44.68 ID:kdaWbqk+.net]
昨日より悪化している様子ですが、至って通常運行です

327 名前:132人目の素数さん [2022/03/11(金) 18:08:00.13 ID:IEqHSrBq.net]
自由群の部分群が自由群であることの証明って難しいんですか?

328 名前:132人目の素数さん [2022/03/11(金) 18:59:22.33 ID:jJTaHo17.net]
自明そうに見えて実は証明が難しいので有名な定理の一つだね。
樹状グラフへの群作用の一般論からすぐに出るが、その一般論の展開に手間がかかる。

329 名前:132人目の素数さん [2022/03/11(金) 19:56:48.71 ID:IEqHSrBq.net]
山内恭彦さんが、「問題を解くな」という題で、

「本文を十分に理解しないで、問題によって定理の内容がはじめてわかるというのは邪道である。いくつかの例題を解かなければ、
わかったかどうか自信の持てないようないい加減な読み方をしていたのでは数学は自分のものにならない。」

などと書いていますが、合っていますか?

ちなみに、

「私はもちろん鈍才ではなかった。」

などとも書いています。

330 名前:132人目の素数さん mailto:sage [2022/03/11(金) 20:00:53.56 ID:NyqtH1lx.net]
>>326
出典を示してもらいたい。

331 名前:132人目の素数さん mailto:sage [2022/03/11(金) 20:32:50.59 ID:Bf3/1QXe.net]
https://en.m.wikipedia.org/wiki/Nielsen%E2%80%93Schreier_theorem

自由群はグラフの基本群、グラフの被覆空間はまたグラフ

332 名前:132人目の素数さん [2022/03/11(金) 20:44:31.57 ID:IEqHSrBq.net]
>>327

吉田洋一・矢野健太郎編『私の数学勉強法』(ダイヤモンド社)です。

333 名前:132人目の素数さん [2022/03/11(金) 20:52:40.49 ID:jJTaHo17.net]
それはとても良い本だ
資格の勉強のために数学を勉強しているうちに数学が好きになって応用数学の研究者になった人の話が感動的だった

334 名前:132人目の素数さん [2022/03/12(土) 07:10:32.51 ID:M+ctUvN/.net]
>>322
「S 上の自由群から G への準同型 φ が全射であるとき、φ(S) は G を生成するという。」に書き換えたらいいでしょ
本文を補いながら読めないのは駄目だよ



335 名前:132人目の素数さん mailto:sage [2022/03/12(土) 10:56:06.27 ID:AVAZDub7.net]
>>329
全体を読んでみないと、山内さんの真意はわからんな。
当然、逆説的な意味合いで書いたものと想像するけど。
この本、古い本で、ネット書店、近くの公立図書館(含:出身大学の附属図書館)にも無くて、状態の悪い古本しかない。
問題を解く解かないの以前に、本文をきちんと読んで十分理解する(そのように最善の努力をする)のは当然。
しかし、手を動かして問題を解いてみて、初めて定理が腑に落ちることがあるのも事実。

もう少し手に入りやすい本で、
新・数学の学び方
著者 小平 邦彦 (編),深谷 賢治 (ほか著)
がある。手元にあるから、この機会に読み直してみようっと。

336 名前:132人目の素数さん mailto:sage [2022/03/12(土) 11:07:00.59 ID:2n9q4Fwf.net]
逆説的ではなく素直に「問題を解くのは邪道、本文で理解すべき」と読んだけど

337 名前:132人目の素数さん mailto:sage [2022/03/12(土) 11:16:33.02 ID:2n9q4Fwf.net]
>>331
粗探しし続ける人をかばうわけではな

338 名前:「が、本のギャップは人に聞くものだと思うけどな []
[ここ壊れてます]

339 名前:132人目の素数さん mailto:sage [2022/03/12(土) 11:32:26.80 ID:EqUMV0Hv.net]
そもそも射がある対称を生成するという言い回しは圏論では普通に出てくる
それが不自然だなんだとバカな事ばっかり言ってるからいつまで経っても前に進まん
元々素頭悪すぎるから性格直しても前になど進まないだろうがね

340 名前:132人目の素数さん [2022/03/12(土) 12:48:01.17 ID:M+ctUvN/.net]
>>332
いや、文字通りの意味。二冊演習問題のついてない名著の実例を挙げていた。一冊は吉田耕作の本だと記憶している。
和算の難問の出し合いは日本数学の正常な発展を阻害した悪習である、と言っていたはずだが、ID:IEqHSrBqよ、この記憶合ってる?

341 名前:132人目の素数さん [2022/03/12(土) 14:14:09.64 ID:SYo+2ymZ.net]
>>336

「このごろ数学の本を書くと問題を付けないと本屋で出版してくれない。ところがワイルの本(Classical Groupsその他)でも吉田先生(Differential and Integral Equations)の本でも、
数学の本当に優れた書物には、問題なんていうものは付いていない。だいたい問題というものは、本文をよく読んでいれば当然解けるものか、あるいは本文とは別な独創的(?)な
アイデアがなければ解けないものである。後の場合、ほかの本を読んで別の知識を持っていればわけなく解けるというような人を馬鹿にしたものもある。」

「こんなのは、日本の旧い和算の伝統で、数学の正常の発達を阻害した悪習である。」

などと書いています。

また、

「ほかの自然科学の学科も同じように勉強しなかったが、このほうは96±2ぐらいに安定していた。」

「とくに一番でいながらお行儀の悪かったのには諸先生を悩ましたものだった。」

などとも書いています。

342 名前:132人目の素数さん [2022/03/12(土) 14:29:20.48 ID:SYo+2ymZ.net]
ファン・デル・ヴェルデン著『現代代数学1』

ですが、この本って何がいいんですか?

群論なんて初歩の初歩しか書いていないですよね。シローの定理も書いてありません。

343 名前:132人目の素数さん [2022/03/12(土) 14:30:15.11 ID:SYo+2ymZ.net]
準同型定理までしか書いてありません。

レベルが低すぎやしないでしょうか?

344 名前:132人目の素数さん [2022/03/12(土) 15:33:08.99 ID:sw88OIef.net]
今日はいつもに増して一段と冴えている様子です



345 名前:132人目の素数さん [2022/03/12(土) 15:54:34.63 ID:M+ctUvN/.net]
>>337
ああ、結構合ってた。

346 名前:132人目の素数さん [2022/03/12(土) 16:19:17.77 ID:YJEEu3Hb.net]
>>336
>和算の難問の出し合いは日本数学の正常な発展を阻害した悪習である
遺題継承ってやつでしょ?んな西洋でも同じやン
てゆーかそれと
>>337
>「このごろ数学の本を書くと問題を付けないと本屋で出版してくれない。ところがワイルの本(Classical Groupsその他)でも吉田先生(Differential and Integral Equations)の本でも、
>数学の本当に優れた書物には、問題なんていうものは付いていない。だいたい問題というものは、本文をよく読んでいれば当然解けるものか、あるいは本文とは別な独創的(?)な
>アイデアがなければ解けないものである。後の場合、ほかの本を読んで別の知識を持っていればわけなく解けるというような人を馬鹿にしたものもある。」
の本書くときに付けないと出版してくれない問題って遺題継承のようなのじゃなくて例題・演習問題のことよね
書いてる本のレベルにも依らないかな
微積とか線形代数とか各種解析学や代数学位相幾何の初歩とかの本のことジャね?
専門書にもあると読者は嬉しいけど無くてもカマワンでしょ

347 名前:132人目の素数さん [2022/03/12(土) 16:24:33.24 ID:M+ctUvN/.net]
「遺題継承ってやつでしょ?んな西洋でも同じやン」
高次方程式の解法とかね。あれも明らかに悪習と言える。
20世紀ではハンガリーが問題の出し合いが数学みたいな変な方向に行っていた(今はどうなのか知らない)

348 名前:132人目の素数さん [2022/03/12(土) 16:30:59.44 ID:SYo+2ymZ.net]
彌永昌吉・小平邦彦著『現代数学概説1』

この本って本当に小平邦彦さんが書いた部分もあるんですか?
自由群について読もうとしましたが、半群とかから書いてある本なので面倒なので読むのをやめました。

Nathan Jacobson著『Basic Algebra I Second Edition』ってどうですか?

「Ocassionally, we shall find it convenient to develop some of the applications in exercises.
For this reason, as well as others, the working of a substantial number of the exercises is
essential for a thorough understanding of the material.」

などと書いてあったので、不便な本だなと思いましたが。

349 名前:132人目の素数さん [2022/03/12(土) 16:32:59.77 ID:SYo+2ymZ.net]
今まで調べた本の中では、自由群についての記述は、Michael Artinさんの本が一番ましだと思います。

350 名前:132人目の素数さん mailto:sage [2022/03/12(土) 17:03:01.66 ID:SYo+2ymZ.net]
テンソル代数と表現論
www.utp.or.jp/book/b598957.html

この本、売れそうな気がしますが、内容は、佐武一郎さんの本に書いてあるようなことですよね?

いつも内容を見ずに期待だけで買って失敗しているので、今回は書店でチェックしてから良さそうな本だったら買おうと思います。

351 名前:132人目の素数さん mailto:sage [2022/03/12(土) 17:19:09.81 ID:J71hwjQ5.net]
まぁ何読んでも無理やろ
またどうせ途中で投げ出して無かった事にするオチ
前の代数学の基本定理も投げ出したままやろ
ちょっとかじっては分からなくなったら本にガタガタイチャモンつけて終わりを繰り返すだけ
いつまで経っても何にもできないまま

352 名前:132人目の素数さん [2022/03/12(土) 17:25:07.73 ID:SYo+2ymZ.net]
>>346

第5章 群の表現論、主に有限群の場合
第6章 対称群の表現

この辺りが興味あるのですが、線形代数を使わないと解明されない有限群の性質は何ですか?

353 名前:132人目の素数さん mailto:sage [2022/03/12(土) 17:56:32.96 ID:SYo+2ymZ.net]
Michael Artin著『Algebra 2nd Edition』ですが、

2面体群 D_n は 2*π/n の回転 x と折り返し y によって生成される。これらの生成元は、

x^n = 1
y^2 = 1
x*y*x*y = 1

を満たす。

これらの関係を使って、 D_n の元たちを x^i * y^j (0 ≦ i < n, 0 ≦ j < 2)の形に書くことができる。

---------------------------------------------------------------------------------

こういう内容が書いてあります。

ですが、 D_n の元たちを x^i * y^j (0 ≦ i < n, 0 ≦ j < 2)の形に書くのに、

x^n = 1
y^2 = 1
x*y*x*y = 1

を使って書く人なんていますか?

何が言いたいのか分かりません。

x^i * y^j (0 ≦ i < n, 0 ≦ j < 2)の形で表わした D_n の元たちの間の積を求めるのに、幾何学的に考えずに、

x^n = 1
y^2 = 1
x*y*x*y = 1

の関係式を使って代数的に求めるという話なら分かるのですが。

354 名前:132人目の素数さん [2022/03/12(土) 17:59:37.13 ID:SYo+2ymZ.net]
S = {x, y, x^{-1}, y^{-1}} をアルファベットとする任意の語を x^i * y^j (0 ≦ i < n, 0 ≦ j < 2)の形に変形するのに、

x^n = 1
y^2 = 1
x*y*x*y = 1

を使うという話なら分かるのですが。



355 名前:132人目の素数さん [2022/03/12(土) 18:01:15.24 ID:SYo+2ymZ.net]
D_n = {x^i * y^j | 0 ≦ i < n, 0 ≦ j < 2} であることは関係式とか持ち出さなくても D_n の定義から明らかですよね。

356 名前:132人目の素数さん [2022/03/12(土) 18:12:08.55 ID:SYo+2ymZ.net]
生成元と関係式についてなんですが、どんな関係式が良い関係式なのかといった話がいままで調べてきた本には一切書かれていないのですが、なぜ
そのような重要な話を書かないのでしょうか?

357 名前:132人目の素数さん mailto:sage [2022/03/12(土) 18:29:59.44 ID:J71hwjQ5.net]
お前そのクソみたいな能力でなんで何が重要でなにが重要でないか分かると思ってんの?
何様?
ひとつも何もやり遂げた事ないクセに
何にもできん能無しのくせになんでそんな上から目線で講釈垂れる事ができるん?
お前が文句垂れてる本の作者はみんなお前なんか足元にも及ばんような功績を上げてきた人たちなのがなんで分からんの?

358 名前:132人目の素数さん [2022/03/12(土) 19:40:47.53 ID:SYo+2ymZ.net]
#D_n ≦ #<x, y | x^n, y^2, x*y*x*y> の証明ですが、Artinさんや雪江さんの本での証明のような抽象的な証明方法しかないんですか?

D_n とかを持ち出してそれとの関係によって証明するのではなく、 <x, y | x^n, y^2, x*y*x*y> の中で完結している証明はないんですか?

359 名前:132人目の素数さん [2022/03/12(土) 19:57:56.79 ID:NT+/FWh4.net]
時代の流れに逆らって引きこもり証明に拘るオレかっけー

360 名前:132人目の素数さん [2022/03/12(土) 20:23:12.59 ID:YJEEu3Hb.net]
>>352
重要と思うなら自分でその主張に基づいた発表でもするのが数学徒でしょ?

361 名前:132人目の素数さん [2022/03/12(土) 21:00:48.27 ID:M+ctUvN/.net]
一般論として与えられた関係からその群がどのような性質を持つかを判定するのは極めて難しい
例えば有限群であるかどうかを判定するアルゴリズムは存在しない

362 名前:132人目の素数さん [2022/03/13(日) 11:03:10.66 ID:BGO5j9mA.net]
群 <x, y, z | x^3, y^3, z^2, x*y*z> を考える。
この群の元 y*x*y*x は 1 に等しいことを示せ。

363 名前:132人目の素数さん [2022/03/13(日) 11:13:17.68 ID:BGO5j9mA.net]
>>358

これは、Michael Artin著『Algebra 2nd Edition』にある例題ですが、良い例題ですね。

364 名前:132人目の素数さん [2022/03/13(日) 12:28:23.30 ID:oVhRSUwo.net]
-1乗を'で表すとして、群<略>の中で
xyz=1よりz=y'x'
1=z^2=y'x'y'x'
よってxyxy=1
右からy^2かけてxyx=y^2
左からyかけてyxyx=y^3=1

ただの計算練習でしかないと思うんだけど、どこらへんが良い例題なのか詳しく教えて



365 名前:132人目の素数さん mailto:sage [2022/03/13(日) 13:45:11.15 ID:BGO5j9mA.net]
あ、簡単でしたね。

でも、Artinさんは必要以上に複雑なやり方で示しているんです。

「The next example shows that computation in R can become complicated, even in a relatively simple case.」

などと書いているので、複雑に見せたいんだと思います。

あるいは高齢のArtinさんにとっては複雑な問題だったのかもしれませんね。

366 名前:132人目の素数さん [2022/03/13(日) 13:46:22.85 ID:e7P6IMU4.net]
>>359
>>361
手のひら返しが酷い

367 名前:132人目の素数さん mailto:sage [2022/03/13(日) 15:29:32.93 ID:BGO5j9mA.net]
Michael Artin著『Algebra 2nd Edition』

free groups, generators and relationsについて大分理解が進みました。
これから、Todd - Coxeter Algorithmのセクションを読もうと思います。

Todd - Coxeter Algorithmについて書いてある日本語の代数学の本はおそらくないと思いますが、なぜでしょうか?
非常に重要な話だと思います。

368 名前:132人目の素数さん mailto:sage [2022/03/13(日) 15:37:13.14 ID:BjRkWk92.net]
能無しのクセにどこまでも上から目線のクソ

369 名前:132人目の素数さん [2022/03/13(日) 16:34:16.11 ID:BGO5j9mA.net]
Michael Artin著『Algebra 2nd Edition』のTodd - Coxeter Algorithmのセクションですが、完全な記述をしているわけじゃないんですね。

370 名前:132人目の素数さん [2022/03/13(日) 16:46:44.17 ID:9Ip71OTU.net]
>>361
その感想は正しいと思えないよ

371 名前:132人目の素数さん [2022/03/13(日) 17:08:15.98 ID:BGO5j9mA.net]
>>365
不完全な記述を読むのは止して、雪江明彦著『代数学1群論入門』の「位数12の群の分類」のセクションを読もうと思います。

雪江さんの本の参考文献のところを見ると、鈴木通夫さんの本にTodd - Coxeter Algorithmの解説があるみたいなのですが、
鈴木さんの解説は分かりやすいですか?

372 名前:132人目の素数さん [2022/03/13(日) 17:14:10.67 ID:sUAGfNGR.net]
>>367
おまえには分からん

373 名前:132人目の素数さん mailto:sage [2022/03/13(日) 17:15:58.55 ID:sUAGfNGR.net]
どうせ鈴木さんて大丈夫な人ですか

て来るんだろう

374 名前:132人目の素数さん mailto:sage [2022/03/13(日) 17:16:29.20 ID:sUAGfNGR.net]
どうせ鈴木さんて大丈夫な人ですか

て来るんだろう



375 名前:132人目の素数さん [2022/03/13(日) 19:02:16.33 ID:9Ip71OTU.net]
>>362
おそらく彼が何も理解してないからだろうなとも思いますね

376 名前:132人目の素数さん [2022/03/13(日) 20:00:39.74 ID:gu4Ou024.net]
>>361
そんなことは聞いてないです
どこらへんが良い例題だと感じたのか?それを聞いています

この人は日本語が読めないのでしょうか?
大丈夫な人なのでしょうか?

377 名前:132人目の素数さん mailto:sage [2022/03/13(日) 20:08:06.65 ID:OMYBLP8F.net]
まぁ変なやつではある
そもそも何のために数学やってるのか全くわからん
まず能力的に将来数学で飯が食える見込みはまるでない
本人もそれは自覚してるだろう
そして多くの数学者が数学を志す理由の一つである“数学という学問”に対する畏敬の念もまるでない
おそらくは”化け物級の自己愛性の人格障害者”なんだろうとは思うが
まぁほっとけばいいんだがともかく敬愛する偉大な数学者たちを散々にコケおろすのが我慢ならん

378 名前:132人目の素数さん [2022/03/15(火) 20:12:36.39 ID:pvcvqVHf.net]
近藤武著『群論』ってどうですか?

自由群についても書いてあるので図書館から借りてきました。

貸し出し履歴の紙が本に貼ってありますが、1件も貸し出し記録がありませんでした。

なぜか、月報が本に挟んでありました。

岩波講座基礎数学月報4に三村征雄さんが、

「私はいま、学士入学したつもりで、この講座を読もうかと思っている。私のような老書生にもわかるように、親切でやさしいものを
書いて下さるよう、諸先生にお願いしたい。」

などと書いています。

379 名前:132人目の素数さん [2022/03/15(火) 20:20:49.47 ID:lOra4tQp.net]
>>374
岩波の基礎数学は親切で優しくて為になるよね

380 名前:132人目の素数さん [2022/03/15(火) 23:10:55.13 ID:KDNuwgA2.net]
>>近藤武著『群論』ってどうですか?

昔それを読みかけたとき
ミスプリではない誤りを見つけて
岩波にハガキを出したことがある。
意外なことに
著者が感謝していたという返事が来た。
それでも読んでみようと思ったくらいだから
よい本だったに違いない。

381 名前:132人目の素数さん [2022/03/16(水) 13:14:59.87 ID:BqiFGRDi.net]
松坂和夫著『代数系入門』

この本での「環」は乗法に関する単位元をもちます。

環 R の部分環の定義ですが、それ自身環であり、 R の乗法に関する単位元を含むものという定義です。

群のときには、部分群の定義はそれ自身群であるようなものという定義でした。

ですので、環 R の部分環の定義をそれ自身環であるようなものと定義すると、 R の乗法に関する単位元を含まない可能性がある
ということを注意書きは必要だと思うのですが、『代数系入門』には一切そのことについて触れていません。

これはOKですか?

382 名前:132人目の素数さん [2022/03/16(水) 13:51:46.71 ID:x/iBQDje.net]
OKです

383 名前:132人目の素数さん [2022/03/16(水) 14:11:54.92 ID:BqiFGRDi.net]
松坂和夫著『代数系入門』

環の準同型写像の定義ですが、

f(x + y) = f(x) + f(y)
f(x * y) = f(x) * f(y)
f(1) = 1

と定義しています。

群の場合には、

f(0) = 0

は導かれるため、定義に含まれていませんでした。

環の準同型写像の定義を

f(x + y) = f(x) + f(y)
f(x * y) = f(x) * f(y)

とすると、

f(1) = 1

は成り立たないかもしれないという注意が必要だと思うのですが、『代数系入門』には一切そのことについて書いてありません。

これはOKですか?

384 名前:132人目の素数さん [2022/03/16(水) 14:16:17.93 ID:x/iBQDje.net]
OKです



385 名前:132人目の素数さん [2022/03/16(水) 15:57:43.72 ID:BqiFGRDi.net]
予想通り、この本は売れそうですね。
現在、ランキング1377位です。

佐武一郎さんの本より分かりやすいですかね?

テンソル代数と表現論: 線型代数続論 単行本 ? 2022/3/28
池田 岳 (著)

出版社 ? : ? 東京大学出版会 (2022/3/28)
発売日 ? : ? 2022/3/28
言語 ? : ? 日本語
単行本 ? : ? 304ページ
ISBN-10 ? : ? 4130629298
ISBN-13 ? : ? 978-4130629294
Amazon 売れ筋ランキング: - 1,377位本 (の売れ筋ランキングを見る本)

386 名前:132人目の素数さん [2022/03/16(水) 15:58:40.95 ID:BqiFGRDi.net]
>>381

アマゾンですが、

「本 (の売れ筋ランキングを見る本)」っておかしくないですか?

387 名前:132人目の素数さん [2022/03/16(水) 16:45:18.55 ID:BqiFGRDi.net]
本が売れるかどうかということでいうと、斎藤毅さんが代数学の本を書けば売れると思います。

388 名前:132人目の素数さん [2022/03/16(水) 16:47:35.02 ID:BqiFGRDi.net]
なんで桂さんが代数の本を書いたんですかね。

389 名前:132人目の素数さん mailto:sage [2022/03/16(水) 17:00:32.02 ID:Avy90WpM.net]
ピーターショルツが代数学の教科書を書いたらかなり他の人にとっては有益だろうなと思う
代数の先の造詣も深いし証明も非常に分かりやすく、イノベーターとしてショルツ個人の観点も価値が大きい

390 名前:132人目の素数さん [2022/03/16(水) 17:07:42.03 ID:x/iBQDje.net]
>>385
IUT批判してる人か

391 名前:132人目の素数さん mailto:sage [2022/03/16(水) 17:37:21.10 ID:Avy90WpM.net]
>>386
それはおまけでパーフェクトイド空間の人

392 名前:132人目の素数さん [2022/03/17(木) 08:41:34.00 ID:YP6mgV26.net]
松坂和夫著『代数系入門』


R を環とし、 J を R に等しくない R の左イデアルとする。もし J を含む R の左イデアルが R と J 自身のほかに存在しないならば、 J を R の極大左イデアル
という。同様にして極大右イデアルも定義される。 R が可換の場合にはもちろんこれらの概念は一致し、単に極大イデアルとよばれる。


などと書かれています。

この書き方だと「極大イデアル」という用語は可換環の場合にしか使われないというようにとれます。


R を環とし、 J を R に等しくない R の左イデアルとする。もし J を含む R の左イデアルが R と J 自身のほかに存在しないならば、 J を R の極大左イデアル
という。同様にして極大右イデアルも定義される。 J が極大左イデアルかつ極大右イデアルであるとき、単に極大イデアルとよばれる。 R が可換の場合にはもちろんこれらの概念は一致する。


と書くべきですよね?

393 名前:132人目の素数さん mailto:sage [2022/03/17(木) 09:36:52.73 ID:DPjNswdB.net]
いいえ

394 名前:132人目の素数さん [2022/03/17(木) 14:10:56.65 ID:YP6mgV26.net]
成田正雄著『初等代数学』

多項式の積について結合法則が成り立つことの証明ですが、「積の定義からあまりにも明らかであろう」などと書いて証明を書いていません。
一方、 (1, 0, 0, …) が積に関する単位元であることは証明しています。

明らかに、結合法則のほうが証明を記述するのが面倒です。

一体何を考えていたんですかね?



395 名前:132人目の素数さん [2022/03/17(木) 14:13:11.96 ID:YP6mgV26.net]
松坂和夫著『代数系入門』

でも、routine workだなどとして、証明を読者の練習問題にしています。

こういうきちんと書くのが大変なものは、松坂さんの常套手段ですが、読者に押し付けますね。

396 名前:132人目の素数さん [2022/03/17(木) 14:19:38.91 ID:YP6mgV26.net]
結合法則が成り立つことの証明ですが、

f = (a_0, a_1, …)
g = (b_0, b_1, …)
h = (c_0, c_1, …)

(f * g) * h
f * (g * h)

のどちらも l 次の項の係数は、添字の和が l になるような a_i * b_j * c_k をすべて足したものになる。

よって、

(f * g) * h = f * (g * h)

が成り立つ。

これでOKですか?

397 名前:132人目の素数さん [2022/03/17(木) 15:37:08.83 ID:xbmzOGB5.net]
>>392
OKです
もそっと厳密に言うなら
Σが和に関して分配則
Σ同士の積(作用の合成)に関して結合則を満たすことを示してからということになりましょうが
ほぼ自明ですし

398 名前:132人目の素数さん [2022/03/17(木) 15:58:02.70 ID:YP6mgV26.net]
>>393

ありがとうございました。

399 名前:132人目の素数さん [2022/03/17(木) 18:22:54.47 ID:YP6mgV26.net]
単項イデアル整域というのがあります。

なぜ、単なる「単項イデアル環」というのは考えないのでしょうか?

400 名前:132人目の素数さん [2022/03/17(木) 18:25:22.44 ID:YP6mgV26.net]
>>395

松坂和夫著『代数系入門』では、そういう説明が全くありません。
そして、突然、考える環を整域に限定して単項イデアル整域などというものを持ち出してきます。

401 名前:132人目の素数さん mailto:sage [2022/03/17(木) 18:29:29.00 ID:zkopiL4v.net]
黙って読め能無し

402 名前:132人目の素数さん [2022/03/17(木) 19:04:20.17 ID:YP6mgV26.net]
整除の関係ってもっとすっきりと記述できないんですか?

403 名前:132人目の素数さん [2022/03/17(木) 19:24:45.90 ID:RQYWBCG3.net]
>>395
整域ではない環の素因数分解について考えてみて下さい

404 名前:132人目の素数さん mailto:sage [2022/03/17(木) 19:51:32.35 ID:PLBzgQ22.net]
>>396
単項イデアル環を定義する本もあるので、他の本を読んでみたらどうでしょうか?
レスポンスがあれば知ってる本勧めますが



405 名前:132人目の素数さん [2022/03/17(木) 20:10:11.75 ID:YP6mgV26.net]
>>399
「素元分解とその一意性」というセクションを読んだら、整域だと、左辺と右辺の共通の約元で約すことができたり、色々便利なのは分かりました。

>>400
ブルバキとかですかね?
「素元分解とその一意性」というセクションを読んだら、単項イデアル整域だと色々と都合よく物事が進むことが分かったので、本の紹介は結構です。

406 名前:132人目の素数さん mailto:sage [2022/03/17(木) 21:11:17.89 ID:PLBzgQ22.net]
>>401
そうでしたか
一息ついて先を読み、それでも分からなければ質問したほうが効率が良いかもしれませんね

407 名前:132人目の素数さん [2022/03/18(金) 12:53:40.66 ID:05C5zaa7.net]
松坂和夫著『代数系入門』

この本では、他の本で既約元と言っているものを素元と言っています。

こういうのはありですか?

408 名前:132人目の素数さん [2022/03/18(金) 12:55:16.57 ID:05C5zaa7.net]
他の本では、素元の定義は既約元とは一致しません。

UFD上では素元 = 既約元だそうですが、一応定義は別々にするのがいいのではないでしょうか?

409 名前:132人目の素数さん mailto:sage [2022/03/18(金) 14:11:47.31 ID:nlnyHFhD.net]
わかってるじゃんキミ

410 名前:132人目の素数さん [2022/03/18(金) 14:19:08.22 ID:TD2d3DEB.net]
いつもの調子なら少なくとも

この本では、他の本で既約元と言っているものを素元と言っています。
松坂和夫さんは大丈夫な人なのでしょうか。

なはずなので、実はあんまりわかってないと思う

411 名前:132人目の素数さん [2022/03/19(土) 10:54:04.60 ID:xrmmaAwA.net]
R がUFDなら R[x] もUFDであるという定理の証明ってなんかスッキリしていませんね。

412 名前:132人目の素数さん [2022/03/19(土) 12:14:29.84 ID:NSEHmUfJ.net]
スッキリさせる必要も無いでしょ?
分かればそれで納得して
自分の基盤にするだけ

413 名前:132人目の素数さん [2022/03/19(土) 12:58:14.52 ID:iiyXIICj.net]
スッキリしないと思う、文句を言うだけなら誰でもできる
そこで止まって自分でスッキリ証明してみようとしない、できないかどうか思うことすらないのが松坂くんクオリティ

414 名前:132人目の素数さん [2022/03/19(土) 18:35:43.65 ID:xrmmaAwA.net]
R をUFDとする。
K を R の分数体とする。

q ∈ R[x] が K において既約な原始多項式ならば、 q は R[x] の素元であることを証明せよ。



415 名前:132人目の素数さん [2022/03/19(土) 18:38:17.81 ID:xrmmaAwA.net]
>>410
松坂和夫著『代数系入門』では、「素元」の定義は、

p が単元ではなく、 a | p ⇒ 「a は単元または p と同伴」が成り立つ

です。

416 名前:132人目の素数さん [2022/03/19(土) 18:49:32.12 ID:xrmmaAwA.net]
>>410
deg(q) ≧ 1 とする。

417 名前:132人目の素数さん mailto:sage [2022/03/19(土) 18:52:50.03 ID:1DHE0GmN.net]
ガウスの定理

418 名前:132人目の素数さん [2022/03/19(土) 18:55:52.92 ID:xrmmaAwA.net]
d ∈ R[x] が d | q を満たすとする。

q = d * d' (d' ∈ R[x])と書ける。

以下で、 d, d' がともに単元でないとして矛盾を導く。

(1) d, d' の次数がともに1以上の場合。

K[x] の単元全体の集合は K - {0} であるから、 d, d' は K[x] の単元ではない。
よって、 q = d * d' は K[x] において既約ではない。これは q についての仮定に反する。

(2) d, d' のどちらかが R の元である場合。

q は 1次以上の多項式であるから、 d, d' の片方は R の元であり、他方は1次以上の多項式である。
d, d' のうち R の元であるほうを a とし、1次以上の多項式であるほうを b とする。

q は原始多項式であるから、 a は単元でなければならない。
これは矛盾である。

419 名前:132人目の素数さん [2022/03/19(土) 18:58:38.82 ID:xrmmaAwA.net]
なんか色々面倒ですね。

420 名前:132人目の素数さん [2022/03/19(土) 19:04:34.94 ID:NSEHmUfJ.net]
>>410
規約なら当たり前ってコトでは

421 名前:132人目の素数さん [2022/03/19(土) 19:08:01.58 ID:RVqkzSM6.net]
超関数と測度論と関数解析と微分幾何の関係性がいまいち分からないのですが?
ちょっとずつかぶってるとおもうのですが
ぜんぶまとめた一般論、もしくはそれに近いのはありますか

422 名前:132人目の素数さん [2022/03/19(土) 19:15:03.70 ID:RVqkzSM6.net]
単語で検索してこれでました

カレント (数学) - Wikipedia
数学、特に函数解析、微分幾何学や幾何学的測度論では、ジョルジュ・ド・ラームの意味でk-カレント(k-current)は、滑らかな多様体M のコンパクトな台を持つ微分形式 k-形式の空間上の汎函数である。
形式的なカレントの振る舞いは、微分形式上シュワルツの超函数に似ている。
幾何学的な設定では、ディラックのデルタ函数や、より一般的な M の部分集合に沿ったデルタ函数の方向微分も、一般化した部分多様体上の積分で表わすことができる。

423 名前:132人目の素数さん [2022/03/19(土) 19:24:06.65 ID:xrmmaAwA.net]
>>416

2 * (x + 1) は Q[x] の既約な多項式ですが、 Z[x] の素元ではありません。

424 名前:132人目の素数さん [2022/03/19(土) 19:35:17.68 ID:NSEHmUfJ.net]
>>419
じゃあそれが>>410の反例?



425 名前:132人目の素数さん [2022/03/19(土) 19:39:49.43 ID:xrmmaAwA.net]
>>420

>>410

では、 q には既約であるだけでなく、原始多項式であるという条件も課されています。

426 名前:132人目の素数さん [2022/03/19(土) 21:04:07.83 ID:NSEHmUfJ.net]
>>421
じゃあモニックだから自明?

427 名前:132人目の素数さん [2022/03/20(日) 11:27:59.70 ID:rsnHfVcP.net]
Eisensteinの規準って単なる十分条件ですけど、なんでどの代数の本にも書いてあるんですか?

428 名前:132人目の素数さん [2022/03/20(日) 12:04:30.39 ID:CZjaoWGX.net]
>>Eisensteinの規準って単なる十分条件ですけど

大方の初学者にとっては非常に非自明だと思う。

429 名前:132人目の素数さん mailto:sage [2022/03/20(日) 12:14:25.91 ID:6UfoMWWO.net]
フレネ・セレの公式
https://ja.m.wikipedia.org/wiki/%E3%83%95

430 名前:%E3%83%AC%E3%83%8D%E3%83%BB%E3%82%BB%E3%83%AC%E3%81%AE%E5%85%AC%E5%BC%8F
になになにベクトル場って用語が色々出てくるんですけど曲線の上だけで定まっていても場って言うのが普通なんですか?
ベクトル場に導ベクトル場があるってのもなんか気持ち悪いんですが
[]
[ここ壊れてます]

431 名前:132人目の素数さん [2022/03/20(日) 12:41:35.32 ID:Ohy7GLVZ.net]
>>414
ああわかった
自明なところを細かく説明したってコトね

432 名前:132人目の素数さん mailto:sage [2022/03/20(日) 12:54:21.47 ID:fuxFqhXl.net]
某所によるとcor 3.12は自明とのこと

433 名前:132人目の素数さん [2022/03/20(日) 12:55:48.46 ID:CZjaoWGX.net]
0と1だけの体があっても気持ち悪くないの?

434 名前:132人目の素数さん [2022/03/20(日) 13:02:11.80 ID:Ohy7GLVZ.net]
>>425
もしかして場って用語を物理サイドから認識したんじゃない?



435 名前:132人目の素数さん [2022/03/20(日) 13:03:46.38 ID:Ohy7GLVZ.net]
>>428
F2のこと?じゃないよね

436 名前:132人目の素数さん [2022/03/20(日) 17:59:01.19 ID:rsnHfVcP.net]
吉田洋一・矢野健太郎編『私の数学勉強法』

赤攝也さんが以下のように書いています:


たしか岩村聯教授からだったと思うが(記憶違いだったらおわびします)、「数学の本は、おしまいのほうから逆に読むものだ」
という警句を聞いたことがある。ちょっと誤解を招きやすいが、一種の名言だと思う。


既に内容が分かっている本なら、そのような読み方もいいかもしれませんが、初めての分野の本では、無理な読み方ですよね?

437 名前:132人目の素数さん [2022/03/20(日) 18:02:35.50 ID:rsnHfVcP.net]
赤攝也さんの本を読んで、赤攝也さんは↓のような人なんだろうなあと想像していたのですが、正解だったようです:


ところで、「厳密な推論によって組み立てられなければならない」という言葉は、必然的に、すべての定理の証明を、
「完全に論理的に」理解できなくてはいけないのだ、という一種の強迫観念を人々にいだかせる。ところが、いつも
すらすらとことがはこぶとは限らない。証明をよんでいると、その途中で、とうもよくわからない、というところが
でてくる。すると私は、どうしてもわかろうと努力する。一時間か二時間で解決がつけば問題はない。しかし、場合
によっては、一日かかってもまだわからないことがある。もちろん、たまりかねて、他の本を参考にしてみたりもする。
それでもなおかつわからないことがある。ところが、「理論は正しい推論によって組み立てられなくてはならない」
ものである。だから、その証明をとばすことは、その理論を正しく理解することにはならない。そこで、結局その本
をなげ出してしまい、かなりの劣等感だけが残るという結果になる。― 自分は、数学に向いていないのではあるまいか?

ときによっては、苦労の甲斐あって、めでたく読了という段階にこぎつけるのに成功することもないではない。しかし、
読んだ内容の大部分は、一週間もすれば忘れてしまう。
「何か、こういうような定理があったっけ。」
「うん、そういえば、そんな言葉もでてきたなあ。だけど、定義は忘れてしまった。」
……………………………………

そこで私はこう思う。いったいなんのためにあの本を読んだんだろう。こう、みんな忘れてしまうのでは、時間を空費
したとしか、いいようがない。私は数学者になりたいんだ。そのためには、やはり、いろいろの知識を確実に集積して
いかなくてはならないのだろう。しかし、この調子では、どうも絶望のようだ。

― 結局、ここでも残るのは、劣等感だけである。

438 名前:132人目の素数さん [2022/03/20(日) 18:39:29.57 ID:Ohy7GLVZ.net]
>>431
>既に内容が分かっている本なら、そのような読み方もいいかもしれませんが、初めての分野の本では、無理な読み方ですよね?
逆じゃないの?そういう意味の警句だと思うけどね
でも最後の方は細かい話に分裂していくことが多いから
一概にそれが良いとは思えないけどなあ

439 名前:132人目の素数さん [2022/03/20(日) 19:22:54.72 ID:rsnHfVcP.net]
吉田耕作さんが、以下の文章を書いています:

ここで、告白(?)致しますと、ヴァン・カンペンの論文でビコンパクト(現在コンパクトといって学部一年生に教えている概念)が
出てきたとき、フレッシェの(点列収束の意味の)コンパクト ― これならわれわれもすでに知っていた ― と同じ概念なのかどうか
誰にもわからずに、大議論になったような時代でした。篤学の角谷君がいろいろ文献を調べて、結局ウリゾーンの有名な論文を
見つけたので落着しましたけれども。


こんなの演習問題レベルの話ですよね。
かなり有名な数学者でもこの程度というのが悲しいですね。

440 名前:132人目の素数さん [2022/03/20(日) 19:35:34.95 ID:rsnHfVcP.net]
吉田耕作さんが以下のように書いていますが、頭の回転がはやければ、

>>434

の話など一瞬で解決していたでしょうね。


振りかえって見ますと、私はむしろ頭の回転がおそいほうですが、記憶力はまずまずのほうらしい。だからいろいろ乱読して得た
雑然とした知識を、年を経てから自己流の拙ない体系に整理する段階になってみると、それらの相互関係もわかってきて、何か
に応用することなどもできる。わずかでも自ら動かせるようになってはじめて自分のものになったような気がするわけです。強いていえば、
これが「私の数学勉強法」です。

441 名前:132人目の素数さん mailto:sage [2022/03/20(日) 19:36:20.22 ID:pWvS1Hy0.net]
>>434
吉田耕作以上に引用されるような本や論文書いてからホザケ。クズ

442 名前:132人目の素数さん [2022/03/20(日) 19:57:20.26 ID:Ohy7GLVZ.net]
>>434
そりゃ後付の感想だろうね

443 名前:132人目の素数さん mailto:sage [2022/03/20(日) 20:32:46.63 ID:Nz6zl8Ev.net]
まともな人間の書くレスじゃないわな

444 名前:132人目の素数さん [2022/03/21(月) 12:55:26.08 ID:8glVzNG6.net]
V を体 K 上のベクトル空間とする。

加法群 V から V への準同型全体からなる環を End(V) とする
ベクトル空間 V から V への線形写像全体からなる環を End_K(V) とする。

End(V) ≠ End_K(V)

となる例を教えて下さい。



445 名前:132人目の素数さん [2022/03/21(月) 13:02:39.01 ID:Pees2SKR.net]
>>439
V=K=R

446 名前:132人目の素数さん [2022/03/21(月) 13:38:31.14 ID:8glVzNG6.net]
>>440

End(V) ∋ f であり、 f ∈ End_K(V) でないような f はなんですか?

447 名前:132人目の素数さん [2022/03/21(月) 13:59:24.09 ID:8glVzNG6.net]
f ∈ End(R) とする。
q ∈ Q とする。
f(1) = a とする。

f(q) = a * q

ですね。

448 名前:132人目の素数さん [2022/03/21(月) 19:00:39.74 ID:8glVzNG6.net]
有限次元ベクトル空間 V と V の再双対空間の間には、基底のとり方には依存しない同型写像を定義することができる。

↑が何を言いたいのか V = R^2 の場合に説明してください。

449 名前:132人目の素数さん [2022/03/21(月) 19:03:39.05 ID:8glVzNG6.net]
1. R^2 からその双対空間の間には、基底のとり方に依存しない同型写像は存在しない。

まず、これを示してください。

450 名前:132人目の素数さん [2022/03/21(月) 19:04:06.45 ID:8glVzNG6.net]
訂正します:

1. R^2 からその双対空間への基底のとり方に依存しない同型写像は存在しない。

まず、これを示してください。

451 名前:132人目の素数さん [2022/03/21(月) 19:15:23.84 ID:Pees2SKR.net]
ガンバってね
前にどっかで書いたと思う

452 名前:132人目の素数さん [2022/03/21(月) 19:19:03.86 ID:8glVzNG6.net]
>>446

K = V = C

これなら容易に示せますね。

複素共役をとる写像は加法群 C から C への準同型写像です。

conj(i * z) = -i * z ≠ i * z if z ≠ 0

なので、線形写像ではありません。

453 名前:132人目の素数さん [2022/03/21(月) 19:20:14.23 ID:8glVzNG6.net]
訂正します:

>>446

K = V = C

これなら容易に示せますね。

複素共役をとる写像は加法群 C から C への準同型写像です。

conj(i * z) = -i * conj(z) ≠ i * conj(z) if z ≠ 0

なので、線形写像ではありません。

454 名前:132人目の素数さん [2022/03/21(月) 19:59:28.20 ID:8glVzNG6.net]
「基底のとり方に依存する」というのは、↓こういうことが言いたいんですか?

V = R^2 の標準基底 <e1, e2> を考える。
<e1, e2> の双対基底を <f1, f2> とする。

e1 を f1 に写し、 e2 を f2 に写すような V から V^* への同型写像 φ1 を考える。

この同型写像により、 (1, 1) ∈ V は

(1, 1) = 1*e1 + 1*e2 → 1*f1 + 1*f2

に写される。

1*f1 + 1*f2 は (x, y) ∈ V を x + y に写す。

------------------------------------------------------------------

V = R^2 の基底 <e2, -e1> を考える。
<e2, -e1> の双対基底を <g1, g2> とする。

e2 を g1 に写し、 -e1 を g2 に写すような V から V^* への同型写像 φ2 を考える。

この同型写像により、 (1, 1) ∈ V は

(1, 1) = 1*e2 + (-1)*(-e1) → 1*g1 + (-1)*g2

に写される。

1*g1 + (-1)*g2 は V ∋ (x, y) = y*e2 + (-x)*(-e1) を y - x に写す。



よって、 φ1 と φ2 は異なる。



455 名前:132人目の素数さん [2022/03/21(月) 20:28:05.11 ID:Pees2SKR.net]
>>447
>K = V = C
そうですね
それが簡単ですね

456 名前:132人目の素数さん [2022/03/22(火) 18:23:40.20 ID:Ze60qfmJ.net]
K を体、 V, W を K 上のベクトル空間とする。

Hom_K(V, W) の特殊な場合に過ぎない V の双対空間 Hom_K(V, K) はなぜ重要なんですか?

457 名前:132人目の素数さん [2022/03/22(火) 18:41:26.64 ID:UEFCkGHM.net]
デュアルだからね

458 名前:132人目の素数さん mailto:sage [2022/03/22(火) 18:53:16.96 ID:wUQov1o5.net]
Vでなくなんで双対考えるかって聞くならわかるけど
一般のhomと比べようって感覚はよく分からない

459 名前:132人目の素数さん [2022/03/22(火) 19:06:03.30 ID:Ze60qfmJ.net]
K を体、 V を K 上の n 次元ベクトル空間とする。

V^* の元って、要は、ドット積 V ∋ v → a・v ∈ K (a ∈ K^n)のことですよね?

V^* とはドット積の集合であるとなぜ平たく書かないんですか?

460 名前:132人目の素数さん [2022/03/22(火) 19:07:54.65 ID:Ze60qfmJ.net]
>>454

訂正します:

K を体、 V を K 上の n 次元ベクトル空間とする。

V^* の元って、要は、ドット積 K^n ∋ x → a・x ∈ K (a ∈ K^n)みたいなものですよね?

なぜ、平たくそう書かないんですか?

461 名前:132人目の素数さん [2022/03/22(火) 19:15:38.57 ID:UEFCkGHM.net]
>>455
君がそれで書いたら?

462 名前:132人目の素数さん [2022/03/22(火) 19:18:08.71 ID:Ze60qfmJ.net]
K を体、 V を K 上の n 次元ベクトル空間、 {v_1, …, v_n} を V の基底とする。

V^* の元って、要は、 a を K^n の元として、 V の元 v に、その座標ベクトルと a とのドット積を対応させる写像のことですよね。

なぜ、平たくそう書かないんですか?

463 名前:132人目の素数さん mailto:sage [2022/03/22(火) 19:18:48.89 ID:qdVyNpi9.net]
>>455
「みたいなもの」って何?
「みたいな」って何?

464 名前:132人目の素数さん [2022/03/22(火) 19:25:28.61 ID:Ze60qfmJ.net]
K を体、 V を K 上の n 次元ベクトル空間とする。

V^** の元って、要は、 x を V の固定元として、 V^* の元 φ に x での φ の値を対応させるような写像のことですよね。

こんなものを考えることがなぜ重要なんですか?



465 名前:132人目の素数さん mailto:sage [2022/03/22(火) 19:28:24.43 ID:qdVyNpi9.net]
>>457
V^* の元って、…
写像のことですよね。
なぜ、平たくそう書かないんですか?


違うからだろうな(笑)

466 名前:132人目の素数さん mailto:sage [2022/03/22(火) 19:29:51.97 ID:qdVyNpi9.net]
>>459
どこの大学出身ですか?

467 名前:132人目の素数さん [2022/03/22(火) 19:30:02.73 ID:UEFCkGHM.net]
>>454
>V^* の元って、要は、ドット積 V ∋ v → a・v ∈ K (a ∈ K^n)のことですよね?
ここで言ってるドット積って内積みたいな双線形形式のこと?ではないよねa∈K^nだし

468 名前:132人目の素数さん [2022/03/22(火) 19:30:46.23 ID:Ze60qfmJ.net]
V ∋ x と x^{^} ∈ V^** を同一視するというのは、 x そのものを、 V^* の元 φ に x での φ の値を対応させる写像だと考えるということですよね?

そんなことして何が嬉しいんですか?

469 名前:132人目の素数さん [2022/03/22(火) 19:32:50.96 ID:UEFCkGHM.net]
>>457
>V^* の元って、要は、 a を K^n の元として、 V の元 v に、その座標ベクトルと a とのドット積を対応させる写像のことですよね。
V^*の定義に座標も基底も関係ないよ

470 名前:132人目の素数さん mailto:sage [2022/03/22(火) 19:34:22.87 ID:qdVyNpi9.net]
>>459
ある概念を学んでいるときにその重要性を教えて欲しいんですか(笑)

定理3-2は重要。定理3-3は重要ではない
とか書いてある教科書がほしいんですか(笑)
今更ですが馬鹿ですか?

471 名前:132人目の素数さん [2022/03/22(火) 19:37:57.21 ID:Ze60qfmJ.net]
>>463

利点としては、

φ は写像で、 x はただの V の元だったのが、 φ も x もどちらも線形形式になって、非対称だったのが対称になって、すこし気分がいいくらいのものですか?

φ(x) = x(φ) みたいに書けてうれしいみたいな?

472 名前:132人目の素数さん [2022/03/22(火) 19:42:02.57 ID:UEFCkGHM.net]
>>463
f(x)をxに対してfをf(x)に対応させる写像を対応させるみたいな

473 名前:132人目の素数さん [2022/03/22(火) 19:43:18.77 ID:UEFCkGHM.net]
>>466
>非対称だったのが対称になって
というかデュアルね

474 名前:132人目の素数さん [2022/03/23(水) 08:01:44.52 ID:Nl4goO46.net]
斎藤毅著『線形代数の世界』

写像が全単射であることを示すのに、全射かつ単射であることを示すのではなく、逆写像を構成して、可逆であることを示すことにより示しています。

こっちのほうが分かりやすいですね。



475 名前:132人目の素数さん [2022/03/23(水) 08:05:43.15 ID:ZyHgXq9L.net]
>>469
場合によるとしか

476 名前:132人目の素数さん mailto:sage [2022/03/23(水) 12:31:28.43 ID:ZTNB7Ifl.net]
>>469
圏論が好きだと言いたいの

477 名前:132人目の素数さん [2022/03/23(水) 13:46:43.71 ID:Nl4goO46.net]
斎藤毅著『線形代数の世界』

証明が独特で巧みな証明が多いですけど、どうやって思いつくんですか?

478 名前:132人目の素数さん [2022/03/23(水) 14:44:17.10 ID:Nl4goO46.net]
斎藤毅著『線形代数の世界』

m×n 行列の集合 M_{mn}(K) m ≧ 0, n ≧ 0 として定義しています。

行列を {x ∈ N | x < m} × {y ∈ N | y < n} から K への写像と考えているからでしょうか?

m = 0 または n = 0 のときには、 {x ∈ N | x < m} × {y ∈ N | y < n} は空集合ですから、

M_{mn}(K) = {空写像} ということですか?

こういうところが嫌いです。

479 名前:132人目の素数さん [2022/03/23(水) 14:44:50.60 ID:Nl4goO46.net]
>>473

訂正します:

斎藤毅著『線形代数の世界』

m×n 行列の集合 M_{mn}(K) を m ≧ 0, n ≧ 0 に対して定義しています。

行列を {x ∈ N | x < m} × {y ∈ N | y < n} から K への写像と考えているからでしょうか?

m = 0 または n = 0 のときには、 {x ∈ N | x < m} × {y ∈ N | y < n} は空集合ですから、

M_{mn}(K) = {空写像} ということですか?

こういうところが嫌いです。

480 名前:132人目の素数さん [2022/03/23(水) 14:48:59.32 ID:Nl4goO46.net]
そして、 m = 0 または n = 0 のときに、 M_{mn}(K) が一体何なのかについて全く説明がありません。

その一方で、「余談」などとして、説明の必要がないことを書いていたりします。

481 名前:132人目の素数さん [2022/03/23(水) 15:10:32.39 ID:TzgUHyAD.net]
>>475
自分で書いてるぞ
>m = 0 または n = 0 のときには、 {x ∈ N | x < m} × {y ∈ N | y < n} は空集合ですから、M_{mn}(K) = {空写像}

自分も定義からすぐ分かってるのに、なんの説明を求めてるのか

482 名前:132人目の素数さん [2022/03/23(水) 18:47:07.88 ID:Nl4goO46.net]
>>476

m ≧ 0, n ≧ 0 に対して m × n 行列が定義されるというのは、誤植だと考える人も多いのではないでしょうか?

なにか説明が要ると思います。

あるいは、ほとんどすべての著者と同じように、 m ≧ 1, n ≧ 1 とすべきではないでしょうか?

483 名前:132人目の素数さん [2022/03/23(水) 18:51:38.99 ID:Nl4goO46.net]
↓こんなくだらない余談を書かずに、 m = 0 または n = 0 のときにはどう考えるか説明を書くべきです。



余談15

記号 M_{mn}(K) や、 a_{ij} の中で、 mn や ij は積 m×n や i×j ではなく、数 m と n や、 i と j をただならべて書いたものである。


484 名前:132人目の素数さん mailto:sage [2022/03/23(水) 18:59:20.14 ID:TzgUHyAD.net]
>>477
著者の思う定義ではなかった誤植と、数学的に通らない誤植があると思うが、
これは数学的には定義に何も問題はなく後者ではないので、前者の誤植を疑う意味はないと思う
この定義を受け入れて他の本を読んだときに、m,nが1以上であっても同じ議論が出来るだろう



485 名前:132人目の素数さん [2022/03/23(水) 20:26:25.21 ID:ZyHgXq9L.net]
>>478
え?それ重要よ

486 名前:132人目の素数さん [2022/03/23(水) 20:31:52.71 ID:Nl4goO46.net]
斎藤毅著『線形代数の世界』

定義1.4.1

V を K 線形空間とする。 W が V の K 部分空間(subspace)であるとは、 W が V の部分集合であって、次の条件をみたすことである。

(1) W の任意の元 x, y に対し、 x + y も W の元である。
(2) K の任意の元 a と W の任意の元 x に対し、 a*x も W の元である。
(3) V の零元 0 は W の元である。

空集合は条件(1)と(2)をみたすが、(3)をみたさない。

-----------------------------------------------------------------------------------------------------

「{} の任意の元 x, y に対し、 x + y も {} の元である。」という文があったとします。

これだけ見ると、「+」って何?という話になると思います。


{} ⊂ V と考えると、

「{} の任意の元 x, y に対し、 x + y も {} の元である。」の「+」は V での加法演算のことなので、問題ないと思います。


空集合 {} は一つしかないわけですが、それを V の部分集合と考えると「{} の任意の元 x, y に対し、 x + y も {} の元である。」が意味をなしますが、
空集合 {} を {バナナ, りんご, いちど} の部分集合と考えると「{} の任意の元 x, y に対し、 x + y も {} の元である。」は意味をなしませんよね?

このあたりはどのように考えたら良いのでしょうか?

487 名前:132人目の素数さん [2022/03/23(水) 20:33:11.73 ID:Nl4goO46.net]
>>481

訂正します:

斎藤毅著『線形代数の世界』

定義1.4.1

V を K 線形空間とする。 W が V の K 部分空間(subspace)であるとは、 W が V の部分集合であって、次の条件をみたすことである。

(1) W の任意の元 x, y に対し、 x + y も W の元である。
(2) K の任意の元 a と W の任意の元 x に対し、 a*x も W の元である。
(3) V の零元 0 は W の元である。

空集合は条件(1)と(2)をみたすが、(3)をみたさない。

-----------------------------------------------------------------------------------------------------

「{} の任意の元 x, y に対し、 x + y も {} の元である。」という文があったとします。

これだけ見ると、「+」って何?という話になると思います。


{} ⊂ V と考えると、

「{} の任意の元 x, y に対し、 x + y も {} の元である。」の「+」は V での加法演算のことなので、問題ないと思います。


空集合 {} は一つしかないわけですが、それを V の部分集合と考えると「{} の任意の元 x, y に対し、 x + y も {} の元である。」が意味をなしますが、
空集合 {} を {バナナ, りんご, いちご} の部分集合と考えると「{} の任意の元 x, y に対し、 x + y も {} の元である。」は意味をなしませんよね?

このあたりはどのように考えたら良いのでしょうか?

488 名前:132人目の素数さん [2022/03/23(水) 20:39:45.44 ID:ZyHgXq9L.net]
>>474
>行列を {x ∈ N | x < m} × {y ∈ N | y < n} から K への写像と考えているからでしょうか?
その本ではそう定義してるの?

489 名前:132人目の素数さん [2022/03/23(水) 20:41:14.08 ID:Nl4goO46.net]
>>482

このあたりの話はどの本を読めばいいですか?

前原昭二さんの『記号論理入門』とか数学基礎論の本とか持っていますが、全く役に立ちません。
新井敏康さんの『数学基礎論増補版』も持っていますが、このようなことは書いていますか?

490 名前:132人目の素数さん [2022/03/23(水) 20:42:22.08 ID:Nl4goO46.net]
>>483

いや、定義していません。ただ、表形式に体 K の元を並べたものが行列という説明です。

491 名前:132人目の素数さん mailto:sage [2022/03/23(水) 20:42:22.70 ID:jfyiGhbx.net]
Hom(0次元,3次元)もHom(4次元,0次元)もHon(0次元,0次元)もベクトル空間としては全部0次元ベクトル空間
underlying set は{0}一元だけからなる一元集合

492 名前:132人目の素数さん [2022/03/23(水) 20:51:40.31 ID:ZyHgXq9L.net]
>>482
>「{} の任意の元 x, y に対し、 x + y も {} の元である。」という文があったとします。
>これだけ見ると、「+」って何?という話になると思います。
(中略)
>空集合 {} を {バナナ, りんご, いちご} の部分集合と考えると「{} の任意の元 x, y に対し、 x + y も {} の元である。」は意味をなしませんよね?
空集合の積集合(これも空集合)から空集合への写像は一つだけ存在するので
その意味で「意味」はあるわけです
気にしているのはある集合からある集合へのある種の性質を持つ写像が定義されている場合とそうで無い場合とでその部分集合の間にこの写像の制限として定義される写像となっている場合となっていない場合があるという状況で後者においてもある種の性質を持つ写像が存在すると考えて良いのかということでしょうがこの場合は特に問題はありません

493 名前:132人目の素数さん [2022/03/23(水) 20:52:52.31 ID:ZyHgXq9L.net]
>>485
では適当に推測するのがよろしいですね

494 名前:132人目の素数さん mailto:sage [2022/03/23(水) 21:17:41.49 ID:jfyiGhbx.net]
そもそも

> (3) V の零元 0 は W の元である。

0を含まなければ部分空間とみなさんと言ってるのに空集合の場合を考える意味がない



495 名前:132人目の素数さん [2022/03/24(木) 10:22:38.77 ID:tOm+hK4d.net]
佐武一郎著『線型代数学』

中への写像って何ですか?

単なる写像のことのようにみえますが、もしそうなら不要な用語ですよね。

そして、不要な用語だから、使われなくなったんですか?

496 名前:132人目の素数さん [2022/03/24(木) 14:29:12.81 ID:tOm+hK4d.net]
佐武一郎著『線型代数学』

↓が成り立つから、 V と V^* の間には標準的な同形が存在しないということですが、
V の基底をその双対基底に写すような同型写像の中には標準的な同形が存在しないと言っているだけですよね?


V を体 K 上の n 次元ベクトル空間とする。

{v_1, …, v_n} を V の基底とする。
{λ_1, …, λ_n} を {v_1, …, v_n} の双対基底とする。

{u_1, …, u_n} を V の基底とする。
{μ_1, …, μ_n} を {u_1, …, u_n} の双対基底とする。

v_i → λ_i (i = 1, …, n) と写すような V から V^* への同型写像を f とする。
u_i → μ_i (i = 1, …, n) と写すような V から V^* への同型写像を g とする。

f = g となるための必要十分条件は、 {v_1, …, v_n} から {u_1, …, u_n} への基底変換行列が直交行列であることである。

497 名前:132人目の素数さん [2022/03/24(木) 14:32:03.69 ID:tOm+hK4d.net]
>>491

V の基底をその双対基底に写すような同型写像という条件をはずせば、
V から V^* への同形の中には標準的な同形が存在するかもしれませんよね?

498 名前:132人目の素数さん mailto:sage [2022/03/24(木) 16:03:06.27 ID:aPpOj8Hk.net]
これって揚げ足取りをするために、過去に読んだことのある沢山の本のメモか何かを見ながら毎日書き込みをしてんのか?

疑問内容のレベルが低すぎるのに反して自分の読み取りに怖いぐらい自信を持ってる上から目線なのだが、
アスペの実例を見れてその点だけは興味深い。

499 名前:132人目の素数さん [2022/03/24(木) 16:17:19.92 ID:tOm+hK4d.net]
斎藤毅著『線形代数の世界』

V を体 K 上のベクトル空間とする。
W, W' を V の部分空間とする。
W × W' に自明な仕方で加法をスカラー倍を定義したベクトル空間を W (+) W' と書く。

W ∩ W' = {0} であるとする。

このとき、

W (+) W' ∋ (u, v) → u + v ∈ W + W' は全単射である。

この写像により、ベクトル空間 W (+) W' と部分空間 W + W' を同一視すると書いています。

この写像はもちろん線形写像なので同型写像ですが、この本では、まだ線形写像が登場しません。
ですので、単なる全単射です。

それにもかかわらず、集合として同一視するのではなく、ベクトル空間として同一視すると書いています。

これは明らかにまずいですよね?

500 名前:132人目の素数さん [2022/03/24(木) 16:23:52.11 ID:tOm+hK4d.net]
池田岳著『テンソル代数と表現論』

公式ページでは今日3月24日発売になっていますが、ネットショップでは3月28日になっていますね。

どちらが正しいのでしょうか?

早く、書店で内容を確認したいです。

501 名前:132人目の素数さん [2022/03/24(木) 17:15:23.86 ID:2MbnhoYz.net]
>>492
無いよ
まあ頑張って作ろうとしてみて

502 名前:132人目の素数さん mailto:sage [2022/03/24(木) 17:23:40.84 ID:aPpOj8Hk.net]
>>478
ちょうどよい余談だな。

それを書いてなければお前がそれにツッコミを知れるような種類の余談。お前の疑問と同レベル。

それに気付かない所がお前の異常性を示してして興味深い。

503 名前:132人目の素数さん mailto:sage [2022/03/24(木) 17:24:58.75 ID:aPpOj8Hk.net]
ツッコミを知れる
→ツッコミを入れる

504 名前:132人目の素数さん mailto:sage [2022/03/24(木) 19:26:36.28 ID:WIh4V48l.net]
ここは英文解釈のスレじゃないけど、数学に関連することなので、了解してね。
DieudonneのFoundations on Modern Analysis vol. I を読んでいます。
such that をどう訳すかなんだけど、例えば、
(3.13.9) In order that a= \lim_{n\to \infty} x_n, a necessary
and sufficient condition is that, for every \varepsilon > 0,
there exist an integer n_0 such that the relation n\geq n_0
implies d(a, x_n) < \varepsilon .

A)森毅訳:現代解析の基礎
a=\lim_{n\to \infty} x_n とは、任意の \varepsilon>0 にたいして、n\geqq n_0
なら d(a,x_n)<\varepsilon となるような整数 n_0 が存在すること。

となってます。慌て者の俺は、
"a=\lim_{n\to \infty} x_n とは、任意の \varepsilon>0 にたいして、n\geqq n_0なら"
と、ここまで読んで、出し抜けにn_0が出てくるので「えっ!? n_0 って何?」となってしまいます。もちろん、最後まで読めば、「あー、n_0は整数で、それが存在するんだ」とわかるけど、なんか「おっとっと」と躓いた感じがして好きじゃないです。

そう思って、これを:
B)(前略)任意の \varepsilon>0 にたいして、整数 n_0 で、
n\geqq n_0 なら d(a,x_n)<\varepsilon となるものが存在
すること。

とすると、「整数 n_0 が存在する」の中間に、
"n\geqq n_0 なら d(a,x_n)<\varepsilon となるようなものが"が割り込んで、主語と述語が離れてしまい、いわゆる悪文になってしまいます。

そうかといって、これを:
C)(前略)任意の \varepsilon>0 にたいして、整数 n_0 が存在して、n\geqq n_0 なら d(a,x_n)<\varepsilon となること。

とすると、英文と微妙にニュアンスが変わってしまうような気がするんだが、
どう訳せばベターでしょうか?
(森毅の上の翻訳も、他に色々突っ込みどころがあるけど、今回はsuch thatに限定します)



505 名前:132人目の素数さん mailto:sage [2022/03/24(木) 19:48:43.37 ID:Wjszltp2.net]
日本語に訳すときはあんまり困らない気がする
英語はなんか後ろから制限かけるようなところがあってそれがすごい困ることあるけどな
日本語なら「任意のaに対してあるbが存在してそれが任意のcに対して〜」と本来の束縛記号を解釈していく順番に前から書いていって自然な日本語になるけど英語はそういう“条件”がガタガタ前に来るのを嫌う文化があるようで、実際綺麗な文章にならなくて困ったりする
「でもこの語順にしないと誤解生む可能性あるやん?」とか思ってすげぇ悩むけどな
そこは日本語の方が有利な希ガス

506 名前:132人目の素数さん [2022/03/24(木) 20:58:30.20 ID:enz8BlOY.net]
>>499が慌て者なのが原因
人の話は最後まで聞け、と言うじゃないか。

507 名前:132人目の素数さん [2022/03/24(木) 22:58:26.57 ID:mJy3uBzM.net]
>>500
>前から書いていって自然な日本語になるけど英語はそういう“条件”がガタガタ前に来るのを嫌う文化
え?逆じゃん

508 名前:132人目の素数さん [2022/03/24(木) 23:01:14.97 ID:mJy3uBzM.net]
>>499
>どう訳せばベターでしょうか?
訳さないのがベスト

509 名前:132人目の素数さん mailto:sage [2022/03/24(木) 23:33:26.92 ID:9A8rlpcN.net]
>>502
そうか?
まず何か芯になってるpredicateについて述べてそのあとの修飾でforall 〜とかいうのが英語としては自然な気がする
つまり英語文化だと“前提条件的なもの”をゴタゴタ述べるのを後回しにして後から後から説明を加えていく構造になってる
実際文法上もSVが来てそれを修飾するMが後からくる
でも数学だとそうはいかない
まず命題に出てくる変数のうち束縛すべきものがforall なのかthere existsなのかを前に持ってこないといけない
でも英語には“後から修飾”が自然でそっちを取らないとおかしい文がかなりできてしまう、だいたいのto 不定詞で修飾するタイプは間違いなく後ろ修飾だしな

510 名前:132人目の素数さん mailto:sage [2022/03/24(木) 23:46:55.43 ID:9A8rlpcN.net]
>>502

例えばこれなんかそうだよ

for every \varepsilon > 0,
there exist an integer n_0 such that the relation n\geq n_0
implies d(a, x_n) < \varepsilon .

これfor every epsilon,って言う前置詞句が先行してる形だけどこれは英語としてはかなり汚いハズ
本来普通の日常会話ではfor〜みたいな前置詞句は後ろに

511 名前:後ろに回すのが英語の通例“for three years"とか"at all times"とか
でもそれだと流石に数学的にはまずいのでやむを得ず前に持ってきてる、しかし可能なら後に置きたいと言うのが本音のようでどのくらいまでなら“やむを得ず前に出す”べきなのか“これくらいなら後ろに置いた方が綺麗”なのかの見極めがつかん
流石にこの例ではこの位置しか置けるところないけどな
[]
[ここ壊れてます]

512 名前:132人目の素数さん mailto:sage [2022/03/25(金) 01:11:40.93 ID:lDOZNO/W.net]
>>500
such that は∃で束縛された変数の満たす条件部分を「」とかで全部括って、
”「」となるような(変数)が存在する”と書きたい(自然な語順)
が、誤解を招かないようにと指導された結果逐語訳的に書いてる、私は苦しい

513 名前:132人目の素数さん [2022/03/25(金) 06:03:05.03 ID:8wyF0lGg.net]
>>504
日本語問題にあんまり関わりたくないが
「すべての自然数nに対して実数xが存在してn<xが成り立つ」
より
「すべての自然数nに対してn<xとなる実数xが存在する」
の方が日本語として自然でしかも
「実数xが存在してすべての自然数nに対してn<xが成り立つ」
とも解釈できる良くない文章
「存在する」が動詞であり日本語は動詞が後置されるのが普通というのがこの混乱を生む
条件が後置されるのは数式がまさにそれだよ
「∀n∈N∃x∈Rn<x」
∀よりもn∈N
∃よりもx∈R
これらすべてよりn<x
の方が後ろに置かれる
これ英語の語順なんだよ

514 名前:132人目の素数さん [2022/03/25(金) 06:08:26.00 ID:qRLdHOH9.net]
>>506
慣れれば苦しくなくなる



515 名前:132人目の素数さん mailto:sage [2022/03/25(金) 07:28:03.77 ID:6AJFI+9P.net]
>>507
その例は確かにその方がいいね
しかしもう日本語の方は存在するを前置するのに慣れてしまったからしゃあなしと思えるようになった
それで「日本語の方が数学表現に向いていない」と日本人が思うのは我々日本人が“日常会話で使う自然な日本語”と“数学的に求められる語順”の差を強く感じるからだと思う
思うにそれはどこの国のどんな言語でも同じで多少は致し方ないのだと思うよ
ただ俺はその問題は英語の方がでかいと思ってる
なぜかと言うと日本語には“格助詞”があって出てくる語がどんな役割を果たしているのかを語順が変わっても「意味は通じる」状態にはできる、しかし英語は語順が重要で語順が変われば意味すら変わってしまう言語、我々日本人には“どっちでも意味わかるやん”と思える語順でも英会話の教科書にはこの副詞節は文中には入れられませんとかなんとか山のように出てくるしね
まぁ意見に過ぎないんだけどな

516 名前:132人目の素数さん [2022/03/25(金) 16:33:09.50 ID:oI+zCtYu.net]
松坂和夫著『代数系入門』

p.209 補題F

M, M' を R-加群、 f : M → M' を全射準同型とし、 P を M の部分加群、 Ker f = N とする。
もし f の P への縮小 f_P : P → M' が P から M' への同型写像ならば、

M = P (+) N

である。

---------------------------------------------------------------------------------

G, G' を 加法群、 f : G → G' を全射準同型とし、 P を G の部分群、 Ker f = N とする。
もし f の P への縮小 f_P : P → G' が P から G' への同型写像ならば、

G = P (+) N

である。

この命題から補題Fは明らかに成り立ちますよね。

517 名前:132人目の素数さん [2022/03/25(金) 16:37:48.94 ID:oI+zCtYu.net]
g を G の任意の元とする。

f_P は全射だから、 f(g) = f_P(p) となるような P の元 p が存在する。

よって、 f(g) = f_P(p) = f(p) より、 f(g - p) = 0 だから、

g - p ∈ N

g- p = n for some n ∈ N

以上より、 G = P + N が成り立つことが分かった。

x ∈ P ∩ N とする。

f_P(x) = f(x) = 0

f_P は単射だから、 x = 0

∴ P ∩ N = {0}

∴ G = P (+) N

518 名前:132人目の素数さん mailto:sage [2022/03/25(金) 16:39:26.70 ID:6iTniafV.net]
Rの作用は?

519 名前:132人目の素数さん [2022/03/25(金) 16:44:12.32 ID:oI+zCtYu.net]
補題F:

M, M' を R-加群、 f : M → M' を全射準同型とし、 P を M の部分加群、 Ker f = N とする。
もし f の P への縮小 f_P : P → M' が P から M' への同型写像ならば、

M = P (+) N

である。

証明:

M, M' は加法群である。
f は全射群準同型である。
P は加法群 M の部分群である。
Ker f = N は全射群準同型 f の核と一致する。
f_P は 加法群 M の部分群 P から加法群 M' への群同型写像である。

>>512
の加法群に関する命題より、

M = P (+) N

である。

520 名前:132人目の素数さん [2022/03/25(金) 16:45:10.05 ID:oI+zCtYu.net]
>>512

R の作用は
>>511
の証明を見れば分かるように全く使われません。

521 名前:132人目の素数さん [2022/03/25(金) 16:46:25.53 ID:oI+zCtYu.net]
松坂和夫さんは、補題FをR-加群に関する命題として書いていますが、実際には、加法群についての命題です。

522 名前:132人目の素数さん [2022/03/25(金) 16:52:51.17 ID:oI+zCtYu.net]
p.202

M を R-加群とし、 N1, N2 を M の部分加群とする。もし任意の z ∈ M が(順序を除き)一意的に

z = z1 + z2; z1 ∈ N1, z2 ∈ N2

と表わされるならば、 M は N1 と N2 の直和に分解されるという。

--------------------------------------------------------------------------------

↑この定義も気に入りません。
↓このように定義すべきです。

M を R-加群とし、 N1, N2 を M の部分加群とする。
M, N1, N2 を加法群と考えたときに、 M が N1 と N2 の直和に分解されるとき、
R-加群 M は R-加群 N1, N2 の直和に分解されるという。

523 名前:132人目の素数さん [2022/03/25(金) 17:42:45.36 ID:oI+zCtYu.net]
R-加群の理論ってなんか嫌ですね。

R をできるだけ制約のない環にしたいけど、それだと何も面白いことを証明できない。
R を体にすると、色々なことを証明できるけど、制約が強すぎる。

R に強すぎず弱すぎない制約を色々課して、その制約のもとで何が証明できるかを考える。

面倒くさすぎます。

524 名前:132人目の素数さん [2022/03/25(金) 18:10:19.28 ID:oI+zCtYu.net]
池田岳著『テンソル代数と表現論: 線型代数続論』を書店で立ち読みした人はいませんか?



525 名前:132人目の素数さん [2022/03/25(金) 18:23:58.96 ID:ehb7IM2+.net]
>>517
知識ないが、印象としてはいろいろできない、違いがない気はするが
具体的には体上のベクトル空間とかだろ
元の体、環をいれかえてもベクトル空間の一般論になるだけの感じがするが

526 名前:132人目の素数さん [2022/03/25(金) 18:38:03.58 ID:6iTniafV.net]
>>514
使わないというか示さないとダメでしょ

527 名前:132人目の素数さん [2022/03/25(金) 19:01:40.39 ID:01IBWIYl.net]
>>517
やりたくなかったらやらなくていいんだぞ

528 名前:132人目の素数さん [2022/03/25(金) 19:54:05.71 ID:6iTniafV.net]
>>514
環準同形R[x]->>R[x]/(x^2)=L->>R[x]/(x)=RでL,RはR[x]加群
R[x]>->>(x)>->R[x]:1->xはR[x]加群の準同形
(x)>->R[x]->>R[x]/(x^2)で(x)>->>(x)/(x^2)=Rx
ただの加法群としてL=R+RxだがR[x]加群としてはL≠R+Rx

529 名前:132人目の素数さん mailto:sage [2022/03/26(土) 10:03:39.37 ID:AsHTalk2.net]
>>517
これは面白い。
数学は論理だけではない。
とはいうもののその実例が分かりにくかったが、このアスペのお陰でまた1つ数学に対する理解が進んだ。

数学者の創造が、アスペにとっての「極めて面倒な領域(R加群)」を作ってアスペを振り落としていくんだな。中途半端に見えることが苦手なんだな。

このアスペにとっての「自分にとっての分かりにくさ、腑に落ちなさ」は一貫している(もちろん本当に「普通に分からない所」はここに書き込むことは無いが)。あくまでも著者の書き方が悪いということにしていないと、このアスペはプライドが保てないのだろう。

530 名前:132人目の素数さん [2022/03/26(土) 10:33:04.50 ID:rrJbIV56.net]
R加群をやるモチベーションがないならベクトル空間でいいじゃん
お前にはまだ早い

531 名前:132人目の素数さん mailto:sage [2022/03/26(土) 10:44:25.91 ID:qqnNSUQN.net]
コイツの場合、環論以前にもうとっくに群論のレベルで振り落とされてるんだけどな

532 名前:132人目の素数さん [2022/03/26(土) 12:12:30.73 ID:bzbf/eIs.net]
線型代数でも振り落とされてたよ

533 名前:132人目の素数さん mailto:sage [2022/03/26(土) 16:04:00.04 ID:8hhO9Nzz.net]
>>522

よくわからないのですが、どういうことでしょうか?

534 名前:132人目の素数さん [2022/03/26(土) 16:05:37.46 ID:8hhO9Nzz.net]
松坂和夫著『代数系入門』

pp. 213-214
(最初に M の生成元 v_1, …, v_n をその個数が最小となるようにとっておけば、明らかにむだな (a_i) は現われない。)

これってどうしてですか?



535 名前:132人目の素数さん [2022/03/26(土) 16:26:18.16 ID:sHi3iAkq.net]
>>527
加法群として直和でもR加群として直和とは限らないという例ですがどこまで分かりましたか?
R[x]
->>
(x^2)
R[x]/(x^2)
(x)
R[x]/(x)
環準同形は加群構造となる
>->>
>->
:1->x
(x)/(x^2)
加法群としてL=R+Rx
R[x]加群としてL≠R+Rx

一部間違えました
(x)->>(x)/(x^2)=Rx
です
ここが分からなかったですか?すいません

536 名前:132人目の素数さん [2022/03/26(土) 16:50:14.66 ID:8hhO9Nzz.net]
>>529

例えば、

R[x]
->>
(x^2)
R[x]/(x^2)
(x)
R[x]/(x)

この記号列は何ですか?

537 名前:132人目の素数さん [2022/03/26(土) 16:53:59.47 ID:8hhO9Nzz.net]
松坂和夫著『代数系入門』

の説明があまりにも拙いので、

Nathan Jacobson著『Basic Algebra I Second Edition』の第3章「Modules over a Principal Ideal Domain」を読もうと思います。

他の日本語の本も見てみましたが、PID上の加群の構造定理を扱っている本は少ないようですね。
例えば、桂さんの本はZ上の加群の構造定理しか扱っていないようですね。

538 名前:132人目の素数さん [2022/03/26(土) 17:11:38.30 ID:sHi3iAkq.net]
>>530
>R[x]
多項式環
>->>
全射
>(x^2)
x^2の生成するイデアル
>R[x]/(x^2)
(x^2)で割った商環
>(x)
xの生成するイデアル
>R[x]/(x)
(x)で割った商環

539 名前:132人目の素数さん mailto:sage [2022/03/26(土) 17:22:53.40 ID:EGlRSlrq.net]
何読んでも同じ
また本に文句言って投げ出すだけ
そもそも可換環論ある程度わかるためには整域を代数閉体に埋め込んで議論とかせんといかんけど前の代数閉体の存在とか代数学の基本定理もギブアップしてほったらかしたままやろ?
お前自分が理解してない代数閉体の存在とか仮定して本読めるん?
お前の論ではそんないい加減な事は許されんのとちゃうの?

540 名前:132人目の素数さん [2022/03/27(日) 01:51:49.21 ID:DyfrzigE.net]
A 可換環, f:A^n→A^m ,A加群の準同型写像
この時f全射ならばn≧mである事を示しなさい。

この問題の解答はAの極大イデアルと何かのテンソル積を考えれば良いと書いてあったのですが分かりません。証明の方法を教えて頂けないでしょうか。

541 名前:132人目の素数さん mailto:sage [2022/03/27(日) 03:30:57.57 ID:NRLe5ax/.net]
A^m→A^nが全射ならAの極大イデアルIをとって
A^m⊗A/I→A^n⊗A/Iが全射なA/I加群の準同型を引き起こす
ここでA^m⊗A/IはA/I^m、A^n⊗A/IはA/I^nとA/I加群として同型
さらにA/Iは体なのですなわちこの引き起こされた全射A/I^m→A/I^nはA/Iベクトル空間としての写像でありベクトル空間の次元に関する議論によりm≧n

542 名前:132人目の素数さん [2022/03/27(日) 04:34:58.34 ID:emeMsIsa.net]
ありがとうございます!

543 名前:132人目の素数さん [2022/03/27(日) 05:26:54.02 ID:emeMsIsa.net]
わからなくなったので質問なのですが

環準同型 A→B 、M,N :A加群とする。
A加群準同型M→Nが単射ならばM ⊗B→N ⊗BはA加群単射準同型でさらにB加群単射準同型である

という認識はあってますでしょうか

544 名前:132人目の素数さん mailto:sage [2022/03/27(日) 08:29:28.81 ID:PbpFHhnO.net]
あってる



545 名前:132人目の素数さん [2022/03/27(日) 08:32:46.96 ID:0wH8EOV6.net]
>>537
一般に単射はダメ
>>535もこちらも⊗は⊗_Aね

546 名前:132人目の素数さん mailto:sage [2022/03/27(日) 08:51:41.45 ID:PbpFHhnO.net]
この場合はええやろ
X⊗BにはB加群構造もそのB構造をA→Bを通して得られるA加群構造も両方入ってる
⊗Bを作用させたから意地でもmodBのobjectとして見なければならないとか言う決まりはない

547 名前:132人目の素数さん [2022/03/27(日) 08:58:02.49 ID:0wH8EOV6.net]
>>540
>この場合はええやろ
ええというのは単射のこと?⊗のこと?

548 名前:132人目の素数さん mailto:sage [2022/03/27(日) 09:10:36.00 ID:5Lt49gRN.net]
>>541
A構造もB構造も両方入ってるという事

549 名前:132人目の素数さん [2022/03/27(日) 09:38:26.33 ID:0wH8EOV6.net]
>>542
言いたいのは⊗は⊗_Aではなく
左にA加群構造右にB加群構造が入るということ?
そうではなく⊗は⊗_Aであって
左のA加群構造はA->Bを通じた右のA加群構造と同じということ?

550 名前:132人目の素数さん [2022/03/27(日) 09:42:49.61 ID:0wH8EOV6.net]
>X⊗BにはB加群構造もそのB構造をA→Bを通して得られるA加群構造も両方入ってる
と書いているから
>>543
>そうではなく⊗は⊗_Aであって
>左のA加群構造はA->Bを通じた右のA加群構造と同じということ?
これを意図しているような気がするけれど
それなら
>>540
>⊗Bを作用させたから意地でもmodBのobjectとして見なければならないとか言う決まりはない
modBがよく分からんがまずはB加群と見ていてその上でA->Bを通じてA加群と見ているわけね

551 名前:132人目の素数さん [2022/03/27(日) 09:46:32.13 ID:0wH8EOV6.net]
なら
>>540のええやろがやっぱりよく分からないなあ
⊗は実は⊗_Aであるなら>>539の後半と同じ主張だし
ということはやはり単射の方を
この場合は単射になるという主張?

552 名前:132人目の素数さん [2022/03/27(日) 09:47:42.90 ID:0wH8EOV6.net]
>>544
>modBがよく分からんが
ああこれはB加群の圏のことか
納得

553 名前:132人目の素数さん mailto:sage [2022/03/27(日) 10:18:36.48 ID:w6WygvYx.net]
>>545
お前A→Bという環準同型がある時、B右加群の圏ModBがA右加群の圏ModAの部分圏になってる話し頭に入ってないやろ
これこそ別スレで話題になってたmonado(この場合はかつcominado)の例やろ?
お前多分自分の中で“俺様ルール、俺様定義”が吹き荒れててそういう“ナチュラんな感覚”の理解が阻害されてるよ
無駄に厳密すぎる
松坂君2号にならんように気付や

554 名前:132人目の素数さん [2022/03/27(日) 10:22:23.77 ID:0wH8EOV6.net]
>>547
>お前A→Bという環準同型がある時、B右加群の圏ModBがA右加群の圏ModAの部分圏になってる話し頭に入ってないやろ
なんで?
>>543
>左のA加群構造はA->Bを通じた右のA加群構造と同じということ?
て書いてるけど?



555 名前:132人目の素数さん [2022/03/27(日) 10:23:33.94 ID:0wH8EOV6.net]
>>547
>お前多分自分の中で“俺様ルール、俺様定義”が吹き荒れててそういう“ナチュラんな感覚”の理解が阻害されてるよ
うーん
よく分からないんだけどどこが「俺様」定義か指摘して

556 名前:132人目の素数さん mailto:sage [2022/03/27(日) 10:26:08.70 ID:w6WygvYx.net]
>>548
解説はせん
そもそもModBと書いて右B加群の圏だと即わからない程度ではこの板で圏論絡みの話するのは10年早い、こんな記号何百本の論文や教科書で使われてるか数えきれんくらいやろ
まだそういうもんに十分な数当たれてない証拠

557 名前:132人目の素数さん mailto:sage [2022/03/27(日) 10:27:13.39 ID:w6WygvYx.net]
来たよ、自分のカスみたいな能力棚に上げて逆ギレするアホ
時間無駄にした

558 名前:132人目の素数さん [2022/03/27(日) 10:29:14.20 ID:0wH8EOV6.net]
>>547
>無駄に厳密すぎる
これは次の
>松坂君
に掛かるの?それとも単独の文で俺のことをそう言っているの?
別に無駄に厳密とは思えないんだけどね
>>540
>この場合はええやろ
のなにが「ええ」なのか
その前の私の書いた内容には2つあるから
「単射がダメ」の方か「⊗は⊗_A」の方か
前者の方を言っているような気がしてきたが
そうですか?

559 名前:132人目の素数さん mailto:sage [2022/03/27(日) 10:31:41.75 ID:w6WygvYx.net]
>>552
お前のその上から目線な物言いだよ
ちょっと会話したら自分の能力が相手より遥かに下回ってんのわからんか?
そういうのが実は一番大切なんだよ能無し君

560 名前:132人目の素数さん [2022/03/27(日) 10:32:02.30 ID:0wH8EOV6.net]
>>550
うーんじゃあ君が
>>537
>環準同型 A→B 、M,N :A加群とする。
>A加群準同型M→Nが単射ならばM ⊗B→N ⊗BはA加群単射準同型でさらにB加群単射準同型である
>という認識はあってますでしょうか
>538 名前:132人目の素数さん 投稿日:2022/03/27(日) 08:29:28.81 ID:PbpFHhnO
>あってる
と言ったのは単にB加群でもあA->Bを通じてA加群でもあると言うところしか見て無くて
「単射」という最も重要なところをスルーしてたってことか
アホかな

561 名前:132人目の素数さん [2022/03/27(日) 10:33:17.58 ID:0wH8EOV6.net]
>>553
うーん
単射見落とす奴に言われたくは無いなw

562 名前:132人目の素数さん [2022/03/27(日) 10:39:25.98 ID:/5cshgMa.net]
>>553
>ちょっと会話したら自分の能力が相手より遥かに下回ってんのわからんか?
(sigh)

563 名前:132人目の素数さん [2022/03/27(日) 10:40:10.70 ID:/5cshgMa.net]
あらID変わったか
俺は ID:0wH8EOV6 ね

564 名前:132人目の素数さん mailto:sage [2022/03/27(日) 10:46:10.99 ID:Wfjsc6Cy.net]
>>555
何も見落としとらんは能無し
そもそもお前が“なんで”と聞いてきた話なんぞ超基本中の基本で聞く事自体恥ずかしい話
お前そもそも加群の理論も圏の理論も教科書一冊読み終えたか終わってないか、しかもレスの感じからするにそれすらまともにマスターできてないレベルやろ?
よくその程度のカスみたいな力で大口叩けるな?



565 名前:132人目の素数さん mailto:sage [2022/03/27(日) 10:49:24.83 ID:RE8h4HCp.net]
無駄に厳密すぎる、数学分かってない!
って人は、3.12の証明が分からなかったフィールズ賞受賞者が数学分かってないように見えるんだろうか

566 名前:132人目の素数さん mailto:sage [2022/03/27(日) 10:51:06.57 ID:RE8h4HCp.net]
>>558
アラン・チューリングは微分の記号の意味が分からなくなって「なんで」とメモを残したが、
君の中ではチューリングは能無しらしいな

567 名前:132人目の素数さん [2022/03/27(日) 10:56:04.38 ID:/5cshgMa.net]
>>558
>何も見落としとらんは能無し
じゃあ
>>537
>環準同型 A→B 、M,N :A加群とする。
>A加群準同型M→Nが単射ならばM ⊗B→N ⊗BはA加群単射準同型でさらにB加群単射準同型である

>>538
>あってる
のね?

568 名前:132人目の素数さん mailto:sage [2022/03/27(日) 10:57:56.15 ID:Wfjsc6Cy.net]
>>560
まだ自分の無能さがわからんから?能無し君?
松坂君と一緒だよ
全然努力に裏打ちされてないカスみたいな数学力
それではるかに上の人間になんとか口げんかだけでも勝ちたいというカスみたいな人間性
悔しかったら口げんかで言い返すのではなく教科書相手の100倍読み込んで見返してやるとか言う気持ちにはお前が達する事はないやろ
そのカスみたいな数学力で松坂君レベルの自分より下見下しとけや能無し

569 名前:132人目の素数さん mailto:sage [2022/03/27(日) 10:58:37.74 ID:Wfjsc6Cy.net]
>>561
あっとる言ってるやろ能無し

570 名前:132人目の素数さん mailto:sage [2022/03/27(日) 10:59:40.22 ID:Wfjsc6Cy.net]
おっと
単射かwww
そこは間違っとるな
よかったな口げんか勝てて
アホ〜wwwww
能無しwwwwww

571 名前:132人目の素数さん [2022/03/27(日) 11:00:14.14 ID:/5cshgMa.net]
>>558
>そもそもお前が“なんで”と聞いてきた話なんぞ超基本中の基本で聞く事自体恥ずかしい話
はぁ
「超基本中の基本」はすでに
>>543
>左のA加群構造はA->Bを通じた右のA加群構造と同じということ?
で書いているのだけど

572 名前:132人目の素数さん [2022/03/27(日) 11:01:30.17 ID:/5cshgMa.net]
>>564
>単射かwww
>そこは間違っとるな
そこが最も重要で>>537の聞きたかったことだと思うよ
わざわざ「単射」と書いているからね

573 名前:132人目の素数さん mailto:sage [2022/03/27(日) 11:05:36.17 ID:RE8h4HCp.net]
>>562
彼の一番改善をオススメしたい点は、同じレベルの教科書を読んでずっとマウントを取り、自分が誤ってたとしても反省がない点だよ
そういう意味では、自分は数学をやってるんだとマウントを取り、>>564で自分が誤っててもなお一言二言多い君がアナロジーに見える

574 名前:132人目の素数さん [2022/03/27(日) 11:06:08.83 ID:/5cshgMa.net]
>>537
>環準同型 A→B 、M,N :A加群とする。
>A加群準同型M→Nが単射ならばM ⊗B→N ⊗BはA加群単射準同型でさらにB加群単射準同型である
喚く人も単射性は一般に成り立たないことは認識しているらしいし
テンソル積考えるときは注意すべき点
自分もついうっかり誤解することはある

ここの⊗は⊗_Aでいいのね?



575 名前:132人目の素数さん mailto:sage [2022/03/27(日) 11:07:50.31 ID:8LebE9yN.net]
>>565
もうええ加減にしとけつて
お前自分で自分のしてきた勉強量一番わかってるやろ
お前の能力なんかせいぜいまさに教科書一冊やっとこさ読み終えた段階くらいやろ?
それが自分でわかっててなんで無理くりでも口げんかに勝ちたいんや?
口げんか勝てば満足なんか?
お前が今やらなあかん事はそんな事か?
ここで悔しい思いしたんなら誰にも負けんくらいまた教科書、論文に挑もうと、いつか誰からも一目置かれるくらいの話できるようになろうと思うことちゃうんか?
そういう人間的な部分が数学の勉強で1番大切なんがまだわかってないんか?
オレはそういう数学勉強する上で1番大切な“心の置き方”がわかってないやつは全部能無しとみなしてる、実際そういうやつは大した能力ない、しかもおそらく五年後み十年後も大したことないカスやろと思ってる
お前が今のままカスで終わるか、カスワールドから脱却できるかのちょうどオンラインくらいやろ
お前の自由や
好きに選べ

576 名前:132人目の素数さん [2022/03/27(日) 11:16:42.46 ID:/5cshgMa.net]
>>569
数学の話してね

577 名前:132人目の素数さん mailto:sage [2022/03/27(日) 11:19:58.58 ID:8LebE9yN.net]
カスコース選びましたか

578 名前:132人目の素数さん [2022/03/27(日) 11:31:04.12 ID:/5cshgMa.net]
>>571
ワタシマケマシタワ

579 名前:132人目の素数さん mailto:sage [2022/03/27(日) 11:46:18.67 ID:zLUsPs1I.net]
素直でよろしい
勉強頑張れよ

580 名前:132人目の素数さん [2022/03/27(日) 12:16:43.84 ID:EzGJ9atj.net]
ぶっちゃけID:Wfjsc6Cyは松坂くんにも間違ったこと言ってたし数学力はお察しのレベルでしかない
もちろんミスすること自体は誰でもあるし謝って訂正すれば良いだけなんだけど、指摘しても何も反応なく逃げてたからその程度の人間でしかない

>>553
>>552
>お前のその上から目線な物言いだよ
>ちょっと会話したら自分の能力が相手より遥かに下回ってんのわからんか?
>そういうのが実は一番大切なんだよ能無し君
これまさにお前のことじゃねえかwwwwwwwwwwww

581 名前:132人目の素数さん mailto:sage [2022/03/27(日) 12:19:08.66 ID:jMpjicVj.net]
なんや、結局こういうオチか
そうやろうとは思ったけどね
さよなら〜

582 名前:132人目の素数さん [2022/03/27(日) 15:57:42.74 ID:Qhe293sP.net]
大学院で代数幾何学の分野で双有理幾何学を研究している研究室ってどの大学にありますか?

583 名前:132人目の素数さん [2022/03/27(日) 17:04:48.68 ID:kS/Ba3bU.net]
自分で調べろよ

584 名前:132人目の素数さん mailto:sage [2022/03/27(日) 17:36:44.81 ID:3Eawv5Rd.net]
>>576
そういう「無駄話」はアスペの馬鹿質問よりも下らない。最低だよお前は。



585 名前:132人目の素数さん [2022/03/27(日) 18:44:54.92 ID:/5cshgMa.net]
>>576
東大京大じゃないかな

586 名前:132人目の素数さん [2022/03/27(日) 18:45:11.77 ID:Mv9NyK1u.net]
>>564
恥を知れ愚か者

587 名前:132人目の素数さん [2022/03/27(日) 18:48:11.81 ID:Mv9NyK1u.net]
>>576
京都大学 藤野修教授
https://www.math.kyoto-u.ac.jp/ja/people/profile/fujino

588 名前:132人目の素数さん [2022/03/27(日) 19:20:45.77 ID:EzGJ9atj.net]
>>575
おう二度と来んなよ

589 名前:132人目の素数さん mailto:sage [2022/03/27(日) 19:41:42.83 ID:ptsTk3i6.net]
>>582
なんや能無し
能無しワールドの方にしましたか
まぁあと2、3年せいぜいしょうもない話チラチラ勉強したふりだけしとけやカス
就職頑張れよ

590 名前:132人目の素数さん [2022/03/27(日) 19:53:11.78 ID:pc0w8tcm.net]
山崎圭次郎著『環と加群』ってどうですか?

松坂和夫著『代数系入門』ですが、加群のところから説明が粗雑すぎます。
体のところは簡単そうですが、なぜ、加群のところだけあんな説明になるのか不思議です。

591 名前:132人目の素数さん [2022/03/27(日) 19:55:25.55 ID:EzGJ9atj.net]
>>583
あ、もう研究の世界に入ってますんでお構いなく
それより>>575でさよならしたんじゃねーのか?もう来なくていいぞ愚か者

592 名前:132人目の素数さん [2022/03/27(日) 19:55:53.53 ID:pc0w8tcm.net]
環って、なんかPIDくらいの制約を課しても、はっきりとは見えてこない感じがしませんか?

593 名前:132人目の素数さん mailto:sage [2022/03/27(日) 19:55:58.35 ID:ptsTk3i6.net]
何読んでもおんなじ
そソロの置き方ワーストワンのお前が何読んでも身につく事はないわ

594 名前:132人目の素数さん mailto:sage [2022/03/27(日) 19:56:25.90 ID:ptsTk3i6.net]
>>585
うそこけーwwwwwww



595 名前:132人目の素数さん [2022/03/27(日) 20:00:18.12 ID:EzGJ9atj.net]
>>588
まあ松坂くん2号の愚か者に信じて貰わなくても結構
そこまで高尚な分野でもないしね

で?さよならしたんじゃないの?
さっさと巣に帰れば???

596 名前:132人目の素数さん mailto:sage [2022/03/27(日) 20:00:46.98 ID:ptsTk3i6.net]
>>585
なんや、全然意味わかってなかったんやな
さよならっていうのはお前が自分のクソみたいなプライド守ること優先してコッチの世界に入ってくるのをやめたみたいやからそっちの世界へ旅立っていくアホ〜をお見送りしたんだよ
アホ〜wwwwwwww
さよなら〜wwwwwwwww

597 名前:132人目の素数さん [2022/03/27(日) 20:01:20.25 ID:pc0w8tcm.net]
有限生成のPID上の加群の構造定理って他の代数入門のトピックと比べて難しいように思います。

598 名前:132人目の素数さん mailto:sage [2022/03/27(日) 20:09:46.71 ID:pc0w8tcm.net]
佐武一郎著『リー群の話』に「PID上の加群」という章があるんですね。

それを読んでみようと思います。

599 名前:132人目の素数さん [2022/03/27(日) 21:16:47.31 ID:pc0w8tcm.net]
佐武一郎さんはなぜ、基底のことを「底」と言うのでしょうか?

600 名前:132人目の素数さん [2022/03/27(日) 21:20:00.75 ID:UcyPE5IB.net]
尼寺はダサい

601 名前:132人目の素数さん mailto:sage [20 ]
[ここ壊れてます]

602 名前:22/03/27(日) 21:22:09.46 ID:3Eawv5Rd.net mailto: >>593
質問が見つからなくて無理やり質問を作り出す。
[]
[ここ壊れてます]

603 名前:132人目の素数さん [2022/03/27(日) 21:23:45.93 ID:fC0VHrfW.net]
>>586
そりゃ
体よりはね
でも群よりずっと筋が良さげ

604 名前:132人目の素数さん [2022/03/27(日) 21:24:45.39 ID:fC0VHrfW.net]
ありゃまたID変わった
俺は ID:/5cshgMa



605 名前:132人目の素数さん [2022/03/27(日) 22:21:43.80 ID:fC0VHrfW.net]
というよりか環が一番面白いのではないかな

606 名前:132人目の素数さん [2022/03/27(日) 23:33:19.97 ID:pc0w8tcm.net]
佐武一郎著『リー群の話』

B 「ところで標数2の数学は実際何かの役に立つのですか?面白いだけでただの“遊び”ではないのですか?」
A 「うーむ、その質問には二通りの答がある。もし日常的な意味で役に立つかというのなら、答は多分Noだろう。…」

などと書いています。

符号理論とか応用的な数学を全く知らないんですね。

607 名前:132人目の素数さん mailto:sage [2022/03/27(日) 23:36:20.33 ID:p6VT+KGK.net]
数学のすべての応用先を知ることなんて無理だろ
伊藤清だって確率微分方程式の一番の応用先であるファイナンスなんて全く知らなかったんだから

608 名前:132人目の素数さん [2022/03/27(日) 23:38:18.87 ID:pc0w8tcm.net]
その後の会話で、Aは、数学のなかでは標数pが重要ということを言っているので、標数2だけではなく、標数pの数学は日常的な意味では役に立たない
と佐武一郎さんは思っていたということになりますね。

符号理論や暗号理論について全く知らなかったとしたら、興味の対象があまりにも狭いと言わざるを得ないですよね。

609 名前:132人目の素数さん [2022/03/27(日) 23:46:07.48 ID:FWPitD7Q.net]
なんかこの松坂くん?って理科大夜間の知り合いに似てるんだよな……
10年くらい前だし今は数学続けてないだろうけど

610 名前:132人目の素数さん mailto:sage [2022/03/28(月) 00:12:38.86 ID:JCSPThxz.net]
>>601
今日は勉強しなかったのでネタが無いんだね

611 名前:132人目の素数さん mailto:sage [2022/03/28(月) 07:09:06.16 ID:vnuGdzmY.net]
>>602
いわゆるレスこじきなんじゃないかなぁ
数学の話以外の彼の感想はスルーでいいと思うけどね

612 名前:132人目の素数さん mailto:sage [2022/03/28(月) 11:29:45.96 ID:HBK5fpnq.net]
>>588
5、60代かな〜

613 名前:132人目の素数さん [2022/03/28(月) 13:33:25.80 ID:rM1ipctH.net]
佐武一郎著『リー群の話』

Hom(V, W^*) と Hom(W×V, K) がカノニカルに同形であるということを説明しています。

佐武さんって、「カノニカルに同形」の話が好きですね。

614 名前:132人目の素数さん [2022/03/28(月) 13:34:27.98 ID:rM1ipctH.net]
要するに基底を使わずに定義された同形写像はカノニカルに同形ということですか?

でも、基底を使って定義された同型写像でもカノニカルに同形になることはあるんですか?



615 名前:132人目の素数さん [2022/03/28(月) 13:38:28.88 ID:rM1ipctH.net]
佐武一郎著『リー群の話』

A 「なるほど、それは少し深刻だな。それじゃまずどんなマトリックスを習ったかいってごらん。」

B 「えーと、対角行列、三角行列、巾零行列、巾等行列。それに対称行列、交代行列、ヘルメット行列、…」

A 「おいおい、物騒なことをいっては困るよ。それはエルミット行列の間違いじゃないのか。」


ヘルメットが物騒というのは、学生運動かなんかを連想させるからですか?

616 名前:132人目の素数さん [2022/03/28(月) 13:40:11.40 ID:rM1ipctH.net]
B 「まるで他人事のようですね。一体ヒョウスウ2のタイというのは何ですか?魚の国の選挙でもあったのですか?」

617 名前:132人目の素数さん [2022/03/28(月) 13:47:11.48 ID:rM1ipctH.net]
Hom(V, W) の次元を求めるのに、 M_{m, n}(K) の次元が m × n だからそれと同形な Hom(V, W) の次元も m × n であると求める人が
いますが、なぜこんなことをするのかが分かりません。

別に、直接 Hom(V, W) の基底を求めて、次元が m × n であると結論すればいいだけの話です。

M_{m, n}(K) の次元が m × n であることの明らかさと Hom(V, W) の次元が m × n であることの明らかさは同じだと思います。

618 名前:132人目の素数さん mailto:sage [2022/03/28(月) 13:49:30.61 ID:rM1ipctH.net]
A と同形な代数系 B で議論したほうが分かりやすいということは本当にあるのでしょうか?同形なのだからわかりやすさは同じはずです。

619 名前:132人目の素数さん mailto:sage [2022/03/28(月) 13:52:28.45 ID:vnuGdzmY.net]
>>610
分からないんですね

620 名前:132人目の素数さん [2022/03/28(月) 13:55:53.64 ID:rM1ipctH.net]
佐武一郎著『リー群の話』

A 「今に微分幾何や物理をやればいやでもそういう量に沢山お目にかかるようになるよ。それに一般の場合、テンソルが存在することは
数学的にもちゃんと証明されているんだ。」

B 「それでは一体テンソルはどこにあるのですか?」(机の下をのぞきこむ。)

A 「おいおい、犬や猫じゃあるまいし、テンソルはそんな所にかくれていやしないよ。」

621 名前:132人目の素数さん mailto:sage [2022/03/28(月) 13:59:20.05 ID:vnuGdzmY.net]
>>611
分かりやすい基底を取るのが有効な例は
フィボナッチ数列の漸化式をみたす数列の全体のなす線型空間で一般項を求めるみたいなのとかはどう?
基底をうまく取らないと無理じゃないかしら

622 名前:132人目の素数さん mailto:sage [2022/03/28(月) 14:00:45.64 ID:vnuGdzmY.net]
>>607
基底で定義した後
普遍的なことを示せることもあるよね

623 名前:132人目の素数さん [2022/03/28(月) 18:34:25.55 ID:Mt47r6e7.net]
数学初学者のものです。
群論の教科書の最初の方に出てくる例題すら難しいのですが、
習いはじめの頃は覚えればいいのでしょうか?
それとも自力で解けなければその教科書はまだ早いということでしょうか?
微積線形あたりは躓かず進められたのですが、代数学に入って戸惑ってます。

624 名前:132人目の素数さん mailto:sage [2022/03/28(月) 18:57:47.30 ID:JCSPThxz.net]
>>616
教科書は何を使ってんの?



625 名前:132人目の素数さん [2022/03/28(月) 19:13:39.06 ID:XVIauYBm.net]
>>616
>群論の教科書の最初の方に出てくる例題
書いて

626 名前:132人目の素数さん mailto:sage [2022/03/28(月) 19:20:09.47 ID:JCSPThxz.net]
>>616
微積と線型で躓かず、代数に入ってから急に躓くとか嘘だな。

627 名前:132人目の素数さん [2022/03/28(月) 19:21:40.22 ID:XVIauYBm.net]
>>619
そっかな
あると思うが

628 名前:132人目の素数さん [2022/03/28(月) 19:22:38.43 ID:XVIauYBm.net]
あらまたID変わってた
俺は ID:vnuGdzmY ね

629 名前:132人目の素数さん mailto:sage [2022/03/28(月) 19:22:50.27 ID:JCSPThxz.net]
>>620
無い。まあ見てろよ。

630 名前:132人目の素数さん mailto:sage [2022/03/28(月) 19:24:40.57 ID:JCSPThxz.net]
>>616
習いはじめの頃は覚えればいいのでしょうか?それとも自力で解けなければその教科書はまだ早いということでしょうか?


こんな疑問はおかしい。微積線型はどのようにやってきたのか。

631 名前:132人目の素数さん [2022/03/28(月) 19:37:06.26 ID:GGn1Nobk.net]
少なくとも大学学部レベルの質問じゃないよね

632 名前:616 mailto:sage [2022/03/28(月) 20:07:11.97 ID:Mt47r6e7.net]
>>617
代数学1群論入門(雪江明彦)です。

>>618
群Gの部分集合HがGの部分群になるための必要十分条件は,次の3つの条件が満たされることである.
(1) 1_G(下添字)∈H .
(2) x, y ∈ H なら、xy ∈ H.
(3) x ∈ H なら、x^-1 ∈ H.

>>619, >>620
微積と線型は今の所違和感なく覚えられました(今後死ぬかもですが)。

>>624
学部下級ということで許してください(汗)

633 名前:616 mailto:sage [2022/03/28(月) 20:18:13.46 ID:Mt47r6e7.net]
>>623
微積線型は、「説明を見る→問題を解く→できてないところを復習」で勉強したのですが、
代数学は計算問題ではなくて、なかなか抽象的・論理的に証明できずにいます。

634 名前:132人目の素数さん mailto:sage [2022/03/28(月) 20:30:41.95 ID:JCSPThxz.net]
>>626
代数学は計算問題ではなくて、なかなか抽象的・論理的に証明できずにいます。

来た。微積線型の教科書は何?
証明は完全にとばしたのか。
幼児的なままのいい加減な勉強で終わらせた後に「躓きが無い」と感じられるテキストなんてあるのか。



635 名前:132人目の素数さん mailto:sage [2022/03/28(月) 20:37:11.87 ID:JCSPThxz.net]
>>

636 名前:625
微積線型の勉強がいい加減でもそのテキスト(雪江)を使って代数の勉強は可能。もちろん人にもよるけど。

証明や例題の解答で省略されている所や分からない所はこのスレとかで質問すれば行ける。底辺大学っぽいけど。
[]
[ここ壊れてます]

637 名前:132人目の素数さん [2022/03/28(月) 21:39:21.91 ID:XVIauYBm.net]
>>625
>群Gの部分集合HがGの部分群になるための必要十分条件は,次の3つの条件が満たされることである.
その本で部分群であることの定義である条件がいくつか提示されていると思うけど
それも書いて

638 名前:616 mailto:sage [2022/03/28(月) 23:44:17.09 ID:Mt47r6e7.net]
>>627
大学の講義だけで教科書は使ってないです。重積分解くとか逆行列求めるとかその程度です。
聞いてると2年3年で代数学的になりそうですね。
そして数学的な証明をまず勉強する必要がありそうですね。

>>628
大学は宮廷なので底辺なのはどちらかというと私ですね。

>>629
Gを群, H ⊂ Gを部分集合とする. HがGの演算によって群になるとき, HをGの部分群という.

一瞬、定義から組み立てられるのかなとも思ったのですが、即書けるほど甘くないですね。
証明の部分部分は追えるのですが、書けといわれるとどう構築するかが解らないです。

集合と論理あたりを先に勉強したほうが良さそうですね。

639 名前:132人目の素数さん [2022/03/29(火) 00:49:44.24 ID:1XoDXVdk.net]
>>630
>Gを群, H ⊂ Gを部分集合とする. HがGの演算によって群になるとき, HをGの部分群という.
なら群であるための定義はどう提示されているの?

640 名前:132人目の素数さん mailto:sage [2022/03/29(火) 06:50:43.37 ID:385K01/b.net]
無駄にスレを消費しないで下さい。

641 名前:132人目の素数さん [2022/03/29(火) 12:18:33.96 ID:1XoDXVdk.net]
>>632
じゃ
バッチリ答えてあげなよ

642 名前:132人目の素数さん [2022/03/29(火) 13:40:16.48 ID:uTNYbRGD.net]
Michael Atiyah他著『Introduction to Commutative Algebra』を持っているのですが、松坂和夫さんの本を読むより分かりやすいですか?

643 名前:132人目の素数さん [2022/03/29(火) 13:47:02.96 ID:uTNYbRGD.net]
なんか代数学への入門書で勉強するより、群論なら群論、環論なら環論の本を読んだほうがいいのではないかと思えてきたのですが。

644 名前:132人目の素数さん mailto:sage [2022/03/29(火) 13:50:01.98 ID:Ot/p6OTh.net]
>>634
前者を読め。
お前が読めるとは思えないので途中で挫折したら「問題が解けないし僕には無理でした」とちゃんと報告すること。著者のせいにばかりするのはそろそろやめろ。



645 名前:132人目の素数さん [2022/03/29(火) 16:00:12.28 ID:385K01/b.net]
うるせぇ、はげ

646 名前:132人目の素数さん mailto:sage [2022/03/29(火) 19:29:39.40 ID:THdx4nTq.net]
>>631
お時間空いてすみません。
"""
Gを空集合ではない集合とする. G上の演算が定義されていて次の性質を満たすとき, Gを群という
(1) 単位元と呼ばれる元 e∈G があり, すべてのa∈Gに対し ae = ea = a となる.
(2) すべての a∈G に対し b∈G が存在し, ab = ba = eとなる. この元bはaの逆元とよばれ a^-1 とかく.
(3) すべての a,b,c ∈ G に対し, (ab)c = a(bc) が成り立つ(結合法則).
"""
以上が群の定義です。


>>625について考えたこととして、
1_GはGの単位元, 1_HはHの単位元として,
(2) x, y ∈ H なら、xy ∈ H.→ Gの演算が成立?(によってHが群になればよい)<部分群の定義の言い換え>
(1) 1_G∈H → 1_H∈H (単位元は一意なので) , すべてのa'∈Hに対し a'e = ea' = a' となる.<群の定義(1)の言い換え>
(3) すべての x∈H に対し x^-1∈H が存在し, x x^-1 = x^-1 x = 1_H∈Hとなる. <群の定義(2)の言い換え>
のような形で対応しているとは思うのですが、群の定義の結合法則については言い換えてませんよね?
なぜ>>625の(1), (2), (3)で well defined(用法違いならすみません)なのか、なぜ結合法則を>>625では言い換えてないのか、
HがGの演算をしていることを示せているのかなど飲み込めていないです。何を示せばゴールといった明確な道標がわからないです。
収束の理論などはノルムさえ作れればあとは計算でしっくりきます。

647 名前:132人目の素数さん mailto:sage [2022/03/29(火) 20:07:13.75 ID:Ot/p6OTh.net]
>>638
なるほど。お前全然駄目だな。よく分かった。「たまに質問して大部分は自力で進めて行ける」というようなレベルではない。ここに居るアスペと同じだ。

648 名前:132人目の素数さん mailto:sage [2022/03/29(火) 20:34:51.66 ID:0UX48HUh.net]
>>638
とりあえずwell-definedの意味はまだ慣れていない
その例題が言っているのは
部分群⇔(1)かつ(2)かつ(3)
を示すということだけで、well-definedは関係ない

足を引っ張ろうとする人は気にせず、少しずつ理解していけばいいと思うよ

649 名前:132人目の素数さん [2022/03/29(火) 20:39:28.11 ID:1XoDXVdk.net]
>>638
>群の定義の結合法則については言い換えてませんよね?
積を具体的にμ(x,y)と書くと結合法則は
すべてのx,y,z∈Gについてμ(μ(x,y),z)=μ(x,μ(y,z))が成立することを意味している
ところで
すべてのx,y,z∈Hについてμ(μ(x,y),z)=μ(x,μ(y,z))が成立すればHで結合法則が成り立つことになるんだけど
これ(すべてのx,y,z∈Hについてμ(μ(x,y),z)=μ(x,μ(y,z)))は成立しますかね?

650 名前:132人目の素数さん [2022/03/29(火) 20:40:51.64 ID:1w74Zo3k.net]
>>641
Hの元をGの元と見れば結合律は自明

651 名前:132人目の素数さん [2022/03/29(火) 21:13:47.30 ID:/SNb8XOl.net]
H⊂Gよりx,y,z∈Hならばx,y,z∈G

652 名前:132人目の素数さん [2022/03/29(火) 22:19:07.17 ID:1XoDXVdk.net]
>>642,643
その通り!君元質問者?
違ったら元質問者の人>>642,643で分かったかな?
自明なので確認の必要が無いわけ

653 名前:132人目の素数さん [2022/03/29(火) 22:28:59.66 ID:/SNb8XOl.net]
自明な事を言語化させるための演習だな

654 名前:616 mailto:sage [2022/03/30(水) 01:08:43.47 ID:Dgy1DVL6.net]
>>641-644
(結合法則)の位置だけ文頭にあるのを文末に替えてます。
HをGの部分群というときの条件(の1つ)として
"H(すべてのx,y,z∈H)がGの演算(μ(μ(x,y),z)=μ(x,μ(y,z)))によって群になるとき."
があるので、成立するように定義されていると条件を必要条件として満たす。
逆に十分条件は(2)によってx,y∈H → μ(x∈H, y∈H)∈H → μ(μ(x,y)∈H,z∈H) ∈ H
ということですかね。これだと、確かに定義から必要十分条件で結べてますね。

>>640
ありがとうございます。切り口がわかってきました。
部分群⇔(1)かつ(2)かつ(3)
を示すのところで、群や部分群の定義が疎かだったので、何をどうつなげるかに合点がいっていませんでした。
定義からつなげて必要十分条件を繋げれるように、まずは定義をしっかり覚えることにします。

>>645
そうですね。暗中模索でしたが、少し考え方がわかった気がします。



655 名前:132人目の素数さん mailto:sage [2022/03/30(水) 12:58:18.57 ID:/B5FJpee.net]
なれないうちはμとμ|_Hを書き分けるべきだと思うの

656 名前:132人目の素数さん [2022/03/30(水) 14:47:04.27 ID:t2tndNMS.net]
以下の行列は鏡映をする正方行列です。

{{cos[x], sin[x], 0},
{sin[x], -cos[x], 0},
{0, 0, 1}}

これの単因子を求めると
1, x-1, x^2+1
になると思います。

これをジョルダン標準形に直すと
{{1, 0, 0 }
{0, 1, 0},
{0, 0, -1}}
となりますが、
この単因子はx-1,x-1,x-1だと思います。
単因子が異なるのに2つの行列が相似となるのはなぜでしょうか。

657 名前:132人目の素数さん mailto:sage [2022/03/30(水) 15:45:11.82 ID:vKjK7M3w.net]
>>648
どちらも単因子は1, x-1, x^2-1

658 名前:132人目の素数さん [2022/03/30(水) 16:39:04.91 ID:6qYhvM+D.net]
>>648
>これの単因子
xって数値?

変数xの特性行列の単因子?

659 名前:132人目の素数さん [2022/03/30(水) 17:09:35.01 ID:t2tndNMS.net]
>>649
ありがとうございます。考え直してみます
>>650
そうです。変数xの特性行列の単因子です。

660 名前:132人目の素数さん [2022/03/30(水) 17:24:20.59 ID:t2tndNMS.net]
記述にミスがあったため書き直しました。すみません。

以下の行列は鏡映をする正方行列です。

{{cos[θ], sin[θ], 0},
{sin[θ], -cos[θ], 0},
{0, 0, 1}}

この行列の特性x行列の単因子を求めると
1, x-1, x^2+1
になると思います。

これをジョルダン標準形に直すと
{{1, 0, 0 }
{0, 1, 0},
{0, 0, -1}}
となりますが、
この特性x行列の単因子はx-1,x-1,x+1だと思います。
単因子が異なるのに2つの行列が相似となるのはなぜでしょうか。

661 名前:132人目の素数さん mailto:sage [2022/03/30(水) 18:31:40.81 ID:vKjK7M3w.net]
もう一度言います最初の行列の単因子は
1, x-1, x^2-1=(x-1)(x+1)
単因子は定義により一つ前の多項式は次の多項式の因子です
だから単因子がx-1,x-1,x+1となることはありません
x-1の次は(x-1)*(何か),今の場合(何か)=x+1ですね

もう一度教科書を確かめてください

662 名前:132人目の素数さん [2022/03/30(水) 18:45:23.12 ID:6qYhvM+D.net]
>>652
>この特性x行列の単因子はx-1,x-1,x+1だと思います。
(x-1,0
0.x+1)
の部分多項式成分の基本変形で
(1,0
0,x^2-1)
になるよ
ていうか君が
>{{cos[θ], sin[θ], 0},
>{sin[θ], -cos[θ], 0},
>{0, 0, 1}}
>この行列の特性x行列の単因子を求めると
>1, x-1, x^2+1
>になると思います。
と書いているθ=0のときが後者だけど

663 名前:132人目の素数さん [2022/03/30(水) 19:06:48.95 ID:t2tndNMS.net]
>>653
>>654
ありがとうございます。いろいろ間違っていることがわかりました。
出直してきます。

664 名前:132人目の素数さん [2022/03/30(水) 20:38:22.50 ID:FMgtKCsb.net]
高木貞治著『初等整数論講義第2版』


仮定によって (a, b) = 1 であるから, 任意の整数 k を

a*y + b*x = k

の形に表わすことができる(定理1.7)。

いま法 a*b に関して考察すれば、 x を a の倍数だけ増減しても、または y を b の倍数だけ増減しても、 a*y + b*x は a*b の倍数だけ増減するのであるから、 a*b を法としての一類に属する。

よって a*y + b*x なる式において、 x には a を法としての各類代表の一組である a 個の値を与え、また y には b を法としての代表の一組である b 個の値を与えるときに、この式 a*y + b*x から出る
a*b 個の値はすなわち a*b を法としての各類の代表の一組でなくてはならない.。

--------------------------------------------------------------------------------------------------------------

「よって a*y + b*x なる式において、 x には a を法としての各類代表の一組である a 個の値を与え、また y には b を法としての代表の一組である b 個の値を与えるときに、この式 a*y + b*x から出る
a*b 個の値はすなわち a*b を法としての各類の代表の一組でなくてはならない.。」

これが成り立つ理由を教えて下さい。



665 名前:132人目の素数さん [2022/03/30(水) 20:47:44.82 ID:FMgtKCsb.net]
↓の2つの文を「よって」でつないでいますが、ギャップがありすぎませんか?


いま法 a*b に関して考察すれば、 x を a の倍数だけ増減しても、または y を b の倍数だけ増減しても、
a*y + b*x は a*b の倍数だけ増減するのであるから、 a*b を法としての一類に属する。

よって、

a*y + b*x なる式において、 x には a を法としての各類代表の一組である a 個の値を与え、また y には
b を法としての代表の一組である b 個の値を与えるときに、この式 a*y + b*x から出る a*b 個の値は
すなわち a*b を法としての各類の代表の一組でなくてはならない.。

666 名前:132人目の素数さん [2022/03/30(水) 20:52:17.42 ID:FMgtKCsb.net]
a を法としての各類代表の一組である a 個の値の集合を {x_1, …, x_a} とし、
b を法としての各類代表の一組である b 個の値の集合を {y_1, …, y_b} とするとき、

a*y_j + b*x_i が互いに非合同であることを証明すればいいわけです。

667 名前:132人目の素数さん [2022/03/30(水) 22:59:41.53 ID:hhCzbwGk.net]
>>656
>これが成り立つ理由を教えて下さい。
f(x,y)=ay+bx:Z^2->Zは全射準同形なので
p:Z->Z/(ab)をつなげても全射準同形
ker(pf)=aZ×bZであって
Z^2/(aZ×bZ)の完全代表系を
K={(x,y)∈Z^2|0≦x<a,0≦y<b}とすると
i:K⊂Z^2とつなげたpfi:K->Z/(ab)は全単射

668 名前:132人目の素数さん [2022/03/30(水) 23:01:56.94 ID:hhCzbwGk.net]
>>657
>↓の2つの文を「よって」でつないでいますが、ギャップがありすぎませんか?
全然?

669 名前:132人目の素数さん [2022/03/31(木) 07:37:27.13 ID:RyhsBaxO.net]
やはり「よって」で上の文と下の文をつなぐのはおかしいですよね。
「よって」と書いているということは、上の文に下の文の理由が書いてあるはずです。
ですが、上の文のどこを探しても下の文が成り立つ理由は書いてありません。

高木貞治さんは大丈夫な人だったのでしょうか?


いま法 a*b に関して考察すれば、 x を a の倍数だけ増減しても、または y を b の倍数だけ増減しても、
a*y + b*x は a*b の倍数だけ増減するのであるから、 a*b を法としての一類に属する。

よって、

a*y + b*x なる式において、 x には a を法としての各類代表の一組である a 個の値を与え、また y には
b を法としての代表の一組である b 個の値を与えるときに、この式 a*y + b*x から出る a*b 個の値は
すなわち a*b を法としての各類の代表の一組でなくてはならない.。

670 名前:132人目の素数さん [2022/03/31(木) 07:47:26.73 ID:RyhsBaxO.net]
Hardy & Wrightの有名な本に同じ命題(定理59)が書いてありました。

非常に分かりやすい証明です。

671 名前:132人目の素数さん [2022/03/31(木) 07:50:08.98 ID:RyhsBaxO.net]
a*y + b*x = a*y' + b*x' (mod a*b)



b*x = b*x' (mod a)
よって、 x = x' (mod a)

a*y = a*y' (mod b)
よって、 y = y' (mod b)

672 名前:132人目の素数さん [2022/03/31(木) 07:57:34.62 ID:RyhsBaxO.net]
高木貞治さんが「よって、」の上の文で言っているのは、要するに以下のことです:

(1)
x = x' (mod a)



a*y + b*x = a*y + b*x' (mod a*b)


(2)
y = y' (mod b)



a*y + b*x = a*y' + b*x (mod a*b)

673 名前:132人目の素数さん [2022/03/31(木) 08:00:45.28 ID:RyhsBaxO.net]
高木貞治さんの文章を数式で書くと以下になります。
「よって、」のおかしさは明白ですよね。


x = x' (mod a) ⇒ a*y + b*x = a*y + b*x' (mod a*b)
y = y' (mod b) ⇒ a*y + b*x = a*y' + b*x (mod a*b)

よって、

a*y + b*x = a*y' + b*x' (mod a*b) ⇒ x = x' (mod a) かつ y = y' (mod b)

674 名前:132人目の素数さん [2022/03/31(木) 08:03:30.97 ID:RyhsBaxO.net]
Hardy & Wrightの証明

>>663

に類するようなことを書くべきだったわけです。



675 名前:132人目の素数さん [2022/03/31(木) 08:48:56.55 ID:RyhsBaxO.net]
>>665

試験でこんな答案を書いたとしたら零点ですよね。

676 名前:132人目の素数さん [2022/03/31(木) 09:19:45.70 ID:rC8zEOK8.net]
「ぼくでもすっきりわかるさいきょうのしょうめい」以外は認めない松坂くんからしたら、そりゃまあ零点でしょうね

普通の人からすれば零点ではないし、そもそも紙面の限られた教科書にある全ての証明一つ一つに対してそのままテストで満点取れる(笑)レベルの細かさを要求するのが間違い

677 名前:132人目の素数さん mailto:sage [2022/03/31(木) 09:28:14.80 ID:wrKDUxeZ.net]
そもそもテストで求められる丁寧さもだれ対象かで変わってくる
学部の一回生のための試験と大学院入試とでは採点基準も変わる
そんな当たり前の事数学勉強始めて遅くとも最初の1年以内くらいには気づいてないといけない事
それがもう何年も何年も数学の教科書読んでるのに気がつかない能無しぶり
全く見込みがない
元々の地頭も悪いんだろうけど、数学という学問に対しての心構えそのものができてない、そしてそういうのが学問極めていくのに一番大切で数学の勉強のキモである事が一部の能無しには永遠に分からんのやろ

678 名前:132人目の素数さん mailto:sage [2022/03/31(木) 09:36:07.51 ID:DOhF98a2.net]
単射見逃したところで少し牙が折れたかと思ったけど反省ゼロだったか

679 名前:132人目の素数さん mailto:sage [2022/03/31(木) 09:39:12.16 ID:wrKDUxeZ.net]
>>670
まさにお前の話だよ、能無し

680 名前:132人目の素数さん [2022/03/31(木) 09:57:24.20 ID:dUjH0WlN.net]
>>666
>に類するようなことを書くべきだったわけです。
言葉で説明していてアレで十分よ

681 名前:132人目の素数さん [2022/03/31(木) 09:59:02.13 ID:RyhsBaxO.net]
>>672

>>665
のどこが十分なのでしょうか?

682 名前:132人目の素数さん [2022/03/31(木) 09:59:06.85 ID:dUjH0WlN.net]
>>670
だったみたいね

683 名前:132人目の素数さん [2022/03/31(木) 09:59:27.47 ID:dUjH0WlN.net]
>>673
ガンバってね

684 名前:132人目の素数さん mailto:sage [2022/03/31(木) 10:44:24.21 ID:N1ew4tno.net]
>>663
これは自明なので著者はとばした。付いてこれない低能は読む資格が無いということ。
お前の質問は全て同じ。

普通の読者は、著者が自明とみなして省略した部分を自力で補いながら読む。「金返せ」と言わんばかりの勢いだが、お前は数学の本を読むのをやめろ。早く死ね。



685 名前:132人目の素数さん mailto:sage [2022/03/31(木) 11:03:06.54 ID:N1ew4tno.net]
>>667
0点は無い。

しかしお前みたいな奴は面接で0点を取る可能性はあるな。しっかり見抜いて0点をつけてもらいたい。

一見細部にまで注意が行き届くように見えて実際には単なるアスペだからな。数学をやる能力が無い。

686 名前:132人目の素数さん [2022/03/31(木) 11:08:22.30 ID:RyhsBaxO.net]
>>676

これが自明というのなら、自明だからという理由で飛ばさなければならない箇所は非常に多いと思います。
初等整数論講義第2版は薄っぺらい本になっていなければなりませんが、実際にはそうではありません。

687 名前:132人目の素数さん mailto:sage [2022/03/31(木) 11:12:12.69 ID:N1ew4tno.net]
>>678
だから、お前には読む資格が無い本なんだよ。読むのをやめろ。お前の批判は的外れで低レベルなので共感を呼ばないのは分かるか?

688 名前:132人目の素数さん mailto:sage [2022/03/31(木) 11:15:22.84 ID:N1ew4tno.net]
>>678
とばすか書くかはお前が決めるのではない。著者が決めること。薄くするのもありだがそれしかあり得ないという思考がお前がアスペの証拠。

お前はここに書き込む時に「自分がアスペでつまらない細かいことだけに目が向いてしまう」ということを自覚しろ。

689 名前:132人目の素数さん mailto:sage [2022/03/31(木) 11:22:41.41 ID:N1ew4tno.net]
この種のアスペは

この本にはこの大事な定理が載っていません。著者は大丈夫な人でしょうか
とか、この本にはこんな無駄な定理が載っています。もっと他に書くことがあるのではないてしようか
とか、アスペ丸出しのことを書き込んてしまう。

690 名前:132人目の素数さん [2022/03/31(木) 13:24:57.95 ID:RyhsBaxO.net]
石田信著『代数学入門』

メビウスの反転公式の証明ですが、以下のように書いています:


Σ_{d | m} μ(m/d) * F(d) = Σ_{d | m} μ(m/d) * Σ_{k | d} f(k) であるが、 k | d なら k | m, m/d | m/k だから、
これは Σ_{k | m} (Σ_{l | m/k} μ(l)) * f(k) にひとしい。


「k | d なら k | m, m/d | m/k だから、これは Σ_{k | m} (Σ_{l | m/k} μ(l)) * f(k) にひとしい。」

何が言いたいのか分かりません。

自分なりに証明すると以下のようになります:

関数 I を I(n) = 1 for all n ∈ {1, 2, 3, …} と定義する。

Σ_{d | m} μ(m/d) * F(d) = Σ_{d | m} μ(m/d) * F(d) = Σ_{d | m} μ(m/d) * Σ_{k | d} f(k) = Σ_{d | m} μ(m/d) * Σ_{k | d} f(k) * I(d/k)
= Σ_{d1 * d2 * d3 = m} μ(d1) * f(d2) * I(d3) = Σ_{d1 * d3 * d2 = m} μ(d1) * I(d3) * f(d2)
= Σ_{k | m} (Σ_{l | m/k} μ(l) * I((m/k)/l)) * f(k)
= Σ_{k | m} (Σ_{l | m/k} μ(l)) * f(k)

691 名前:132人目の素数さん [2022/03/31(木) 13:25:47.35 ID:RyhsBaxO.net]
>>682

ダミーの関数 I を考えたところがうまいですね。

692 名前:132人目の素数さん [2022/03/31(木) 13:44:16.14 ID:jhVzzh6/.net]
>>682
>何が言いたいのか分かりません。
割と分かりやすい部分だよ

693 名前:132人目の素数さん [2022/03/31(木) 14:25:35.05 ID:RyhsBaxO.net]
石田信著『代数学入門』


しかし、 R が単位元をもつ環であっても、部分環 S は必ずしも単位元をもつとはかぎらない(例3参照)。
また部分環 S が単位元(≠ 0)をもっていても、それが R の単位元であるとはかぎらない(問5)。


この注意は必要ですよね。

松坂和夫著『代数系入門』では、単位元をもつ環のことを環と定義しています。
『代数系入門』での群 G の部分群の定義は、それ自身群になるような G の部分集合というものです。
部分環は、それ自身環になるような R の部分集合のこととは定義していません。
部分環とは、それ自身環になるような R の部分集合で、 R の単位元を含むものという定義です。

この定義は、

「また部分環 S が単位元(≠ 0)をもっていても、それが R の単位元であるとはかぎらない(問5)。」

↑のような S を部分環から排除したいためだと思いますが、このような例について『代数系入門』には記述がありません。

松坂和夫さんは一体何を考えていたのしょうか?

このような例は必ず書かなければならないものだと思います。

694 名前:132人目の素数さん [2022/03/31(木) 14:34:37.92 ID:RyhsBaxO.net]
部分群の場合には、それ自身群になるような G の部分集合でありさえすれば、 1_G を必然的に含みますが、
環の場合にはそうではありません。

こういう違いがあるという注意は、いかにも松坂和夫さんが書きたがりそうな注意ですが、書いていません。

環の定義はやはり、加法について可換群であり、乗法について結合法則が成り立ち、分配法則が成り立つものという定義がいいと思います。

これだと環の場合にも、

部分環とは、それ自身環になるような R の部分集合のこと

と定義できるからです。



695 名前:132人目の素数さん [2022/03/31(木) 14:54:58.35 ID:RyhsBaxO.net]
石田信著『代数学入門』

この本での部分体の定義はやはり

それ自身体になるような F の部分集合のこと

というものです。


ここでつぎの注意をしておこう。 S を環 R の部分環とする。このとき、 S は加法群としての R の部分群だから、 S の零元は R の零元 0 と一致し、
また S の元 c の S での(加法の)逆元は c の R での逆元 -c と一致する(1-7節参照)。さらに K が体 F の部分体のときは、 K^* = K - {0} は
乗法群としての F^* = F - {0} の部分群だから、 K の単位元は F の単位元 e と一致し、また K の元 c ≠ 0 の K での(乗法の)逆元は c の F での
逆元 c^{-1} と一致する(1-7節参照)。


統一感があって、気持ちがいいですね。

696 名前:132人目の素数さん [2022/03/31(木) 16:50:45.25 ID:RyhsBaxO.net]
現在、1591位ですね。
誰か、買った人、書店で見た人いますか?

テンソル代数と表現論: 線型代数続論 単行本 ? 2022/3/26
池田 岳 (著)
出版社 ? : ? 東京大学出版会 (2022/3/26)
発売日 ? : ? 2022/3/26
言語 ? : ? 日本語
単行本 ? : ? 304ページ
ISBN-10 ? : ? 4130629298
ISBN-13 ? : ? 978-4130629294
寸法 ? : ? 1

697 名前:5 x 2 x 21 cm
Amazon 売れ筋ランキング: - 1,591位本 (の売れ筋ランキングを見る本)
- 2位代数・幾何
[]
[ここ壊れてます]

698 名前:132人目の素数さん [2022/03/31(木) 18:47:22.29 ID:dUjH0WlN.net]
>>685
>しかし、 R が単位元をもつ環であっても、部分環 S は必ずしも単位元をもつとはかぎらない(例3参照)。
>また部分環 S が単位元(≠ 0)をもっていても、それが R の単位元であるとはかぎらない(問5)。
普通の定義だと0と1は共通よ
あんまり広げてもつまらないし

699 名前:132人目の素数さん [2022/03/31(木) 18:54:17.52 ID:RyhsBaxO.net]
石田信著『代数学入門』

Five Lemmaって何の役に立つんですか?

この命題を見ても、「だから何?」という感想しか持てませんよね。

700 名前:132人目の素数さん [2022/03/31(木) 19:14:31.66 ID:dUjH0WlN.net]
>>690
誰かの言葉を借りれば
超基礎中の基礎

701 名前:132人目の素数さん [2022/03/31(木) 19:40:24.12 ID:RyhsBaxO.net]
>>691

石田信著『代数学入門』

Five Lemmaで証明することが2つあります。
1つは本文で証明されています。
もう一方をノーヒントで証明しました。
証明の最後までの流れは見渡せない感じですが、次に何をすべきかは各段階で自ずと分かりますね。
各段階ですべきことをするといつの間にか最後の結論を導いているという感じですね。

センスありますか?

702 名前:132人目の素数さん [2022/03/31(木) 19:55:45.22 ID:dUjH0WlN.net]
>>692
>いつの間にか最後の結論を導いているという感じ
つまり
一見意味不明に見えて当たり前の結果だってことなんだよね

703 名前:132人目の素数さん [2022/04/02(土) 18:22:55.98 ID:at4qHNQh.net]
池田岳著『テンソル代数と表現論』

書店で見てきました。

ぱらぱらと見た感じでは、特に分かりやすく書かれているわけでもない普通の本という感じでした。

704 名前:132人目の素数さん [2022/04/02(土) 19:46:10.09 ID:CFY9yb0C.net]
>>692
>センスありますか?

自分の中では「歴史に名を残す大天才レベル」だと思ってそう



705 名前:132人目の素数さん mailto:sage [2022/04/02(土) 22:21:42.19 ID:qXvt9j2y.net]
どこで質問したらよいのかわからないのでここで質問させてください。
より相応しい場所があれば教えていただけると助かります。

確率の問題です。
それぞれ異なる確率x1, x2, ..., xm で成功する独立した試行がm個存在するとき、
これらの試行のうちちょうどn個(0 <= n <= m)が成功する確率の求め方を教えてください。

n=0の時は(1- x1) * (1 - x2) * ...で、n=mの時は単純に全部かければよいとわかるのですが、
それ以外のパターンは一般化できるのでしょうか?

706 名前:132人目の素数さん [2022/04/02(土) 22:48:04.28 ID:at4qHNQh.net]
リーマン・スティルチェス積分は普通のリーマン積分と難易度は少しも変わりませんが、なぜ一部の微分積分の教科書しかリーマン・スティルチェス積分について書かれていないのでしょうか?

707 名前:132人目の素数さん [2022/04/03(日) 10:57:33.16 ID:hj1bT/iI.net]
>>696
二項分布

708 名前:132人目の素数さん [2022/04/03(日) 11:46:00.63 ID:LwomPzda.net]
>>696
p1,p2,…,pmをそれぞれの生起確立とする
x1,x2,…,xmをそれぞれが起これば1起こらなければ0の確率変数とする
P(x1,x2,…,xm)
=p1^x1(1-p1)^(1-x1)p2^x2(1-p2)^(1-x2)…pm^xm(1-pm)^(1-xm)
Σ_{x1,x2,…,xm}P(x1,x2,…,xm)t^(x1+x2+…+xm)
=Σ_{x1}p1^x1(1-p1)^(1-x1)t^x1Σ_{x2}p2^x2(1-p2)^(1-x2)t^x2…Σ_{xm}pm^xm(1-pm)^(1-xm)t^xm
=(p1t+(1-p1))(p2t+(1-p2))…(pmt+(1-pm))=F(t)
Σ_{x1+x2+…+xm=n}P(x1,x2,…,xm)
=F^(n)(0)/n!

709 名前:132人目の素数さん [2022/04/03(日) 11:51:19.12 ID:qnTq7OrA.net]
吉田伸生著『複素関数の基礎』

昨日、本屋でぱらぱらと見ました。

参考文献に「松阪和夫」などと書かれていました。

雪江明彦さんもYouTubeの講義動画で黒板に「松阪」などと書いていました。

https://youtu.be/pZMusy4HJjI?t=142

710 名前:132人目の素数さん [2022/04/03(日) 11:54:02.19 ID:LwomPzda.net]
m=4,n=2なら
p1p2(1-p3)(1-p4)+p1(1-p2)p3(1-p4)+p1(1-p2)(1-p3)p4+(1-p1)p2p3(1-p4)+(1-p1)p2(1-p3)p4+(1-p1)(1-p2)p3p4
=(p1p2+p1p3+p1p4+p2p3+p2p4+p3p4)-3(p1p2p3+p1p3p4+p2p3p4)+6p1p2p3p4

711 名前:132人目の素数さん [2022/04/03(日) 12:11:14.36 ID:LwomPzda.net]
>>699
>(p1t+(1-p1))(p2t+(1-p2))…(pmt+(1-pm))=F(t)
F(t+1)=(1+tp1)(1+tp2)…(1+tpm)=Σt^ns_n(p1,p2,…,pm)
ここでs_n(x1,x2,…,xm)はn次基本対称式
F^(n)(t+1)=(F(t+1))^(n)=Σ((n+k)!/k!)t^ks_(n+k)(p1,p2,…,pm)
Σ_{x1+x2+…+xm=n}P(x1,x2,…,xm)
=F^(n)(0)/n!
=Σ(n,k)(-1)^ks_(n+k)(p1,p2,…,pm)
ここで(n,k)=(n+k)!/n!k!=(n+k)Cn

712 名前:132人目の素数さん [2022/04/03(日) 12:56:46.02 ID:qnTq7OrA.net]
>>694

この本ですが、佐武一郎さんの本よりも分かりやすく書いたとか著者が書いていましたが、佐武一郎さんの本はそんなに分かりにくいんですか?

テンソル代数よりも前の部分は証明などが非常に明晰だと思うのですが。

713 名前:132人目の素数さん mailto:sage [2022/04/03(日) 14:51:08.88 ID:PETaFxsk.net]
キミ
前に佐武さんて大丈夫な人なんですか
と書いていたんじゃない

今度は池田さんて大乗な人でしょうか

とかくの?

714 名前:132人目の素数さん [2022/04/03(日) 16:43:19.86 ID:LwomPzda.net]
>>697
あんまり使わないから



715 名前:132人目の素数さん [2022/04/03(日) 16:49:33.29 ID:LwomPzda.net]
でも
確率論やるなら必須

716 名前:132人目の素数さん [2022/04/03(日) 17:13:41.40 ID:qnTq7OrA.net]
>>705-706

Walter Rudin著『Principles of Mathematical Analysis 3rd Edition』

では、 α が区間 [a, b] で単調非減少関数であるときに、

リーマン・スティルチェス積分 ∫_{a}^{b} f dα を定義しています。

岩波数学入門辞典を調べたら、 α は有界変動関数となっていました。

717 名前:132人目の素数さん [2022/04/04(月) 10:09:15.59 ID:3TmVav6Y.net]
F を(可換)体とする。
R を F の部分環で単位元をもつとする。

R の単位元は F の単位元と一致することを示せ。

718 名前:132人目の素数さん [2022/04/04(月) 10:46:08.85 ID:3TmVav6Y.net]
あ、簡単でした。

e_R * e_R = e_R = e_F * e_R
∴ e_R = e_F

719 名前:132人目の素数さん [2022/04/05(火) 15:03:19.63 ID:mjR/NTJt.net]
開区間 I で定義された関数 f で、I 内に不連続な点が至る所稠密に分布しているのと同時に
I 内に微分可能な点が至る所稠密に分布しているようなものの例を挙げよ。

小平邦彦著『解析入門』にこのような例が書いてあります。

小平さんのオリジナルだと思いますが、小平さんとは違うもっと分かりやすい例はありますか?

720 名前:132人目の素数さん [2022/04/05(火) 15:47:47.76 ID:TN8WWiQx.net]
>>710
その例知らんけど
普通はf(m/n)=1/nみたいなのでは?
これじゃ微分可能じゃないかな
まあでも似たようなのでできそう

721 名前:132人目の素数さん mailto:sage [2022/04/05(火) 16:13:36.65 ID:dCfhceFh.net]
(0,1)の実数xに対して関数I(x)をxの十進表示(ある桁から全部9は禁止)x = Σ a(x,n)10^(-n)とする
I(x) = sup{ n | a(x,n) ≠ 0 }
としておく(∞もとりうる)
f(x) = Σ[ y ≦ x ] (1/100)^I(y)H(x - y )
とする、H(x)はH(0)=1/2のヘビサイドの関数

722 名前:132人目の素数さん [2022/04/05(火) 19:03:01.52 ID:mjR/NTJt.net]
>>711-712

ありがとうございました。

>>712

小平さんの例のほうが分かりやすいようです。

723 名前:132人目の素数さん [2022/04/05(火) 20:58:51.77 ID:mjR/NTJt.net]
小平邦彦さんのp.109例3.1の証明を読んでみましたが、証明の一番最後のところの議論が
むちゃくちゃ分かりにくかったです。

もっと議論を分かりやすくできるはずです。

724 名前:132人目の素数さん [2022/04/05(火) 21:51:56.64 ID:zo35/FUy.net]
>>714
キミガヤルノラ



725 名前:132人目の素数さん [2022/04/06(水) 09:39:17.20 ID:NqZL91k4.net]
笠原晧司著『微分積分学』


定理4.25 f(x) が x_0 で解析的なら、 x_0 の適当な近傍の各点で解析的である。

注意 これはおどろくべきことである。「1点で微分可能なら、その近傍の各点で
微分可能」などということはない。これと対照的に、解析性は1点での性質がある
近傍での同じ性質を導くのである。


などと書いています。

f を連続関数とします。
「1点で 0 でないなら、その近傍の各点で 0 でない」という性質が成り立ちます。
これはおどろくべきことでしょうか?

笠原さんは一体どんな数学的センスの持ち主なのでしょうか?

726 名前:132人目の素数さん [2022/04/06(水) 10:19:27.09 ID:jpE5qX2/.net]
>>716
驚かされました
1点で0であるという性質と1点で微分可能であるという性質が同等であるとは
一体どんな数学的センスの持ち主なのでしょうか?

727 名前:132人目の素数さん mailto:sage [2022/04/06(水) 10:26:57.36 ID:XpyYLwVi.net]
>>716
驚くべきことを発見しました。数学的センスの無い読者にかかるとどんな数学書の著者も侮蔑の対象になってしまうのですね。勉強になりました。

あなたのレスは全てそれですね。すごいですね。

728 名前:132人目の素数さん mailto:sage [2022/04/06(水) 10:37:53.16 ID:GMfgtPM7.net]
まぁまともに相手するのも無意味だというのがこのレスひとつでよくわかるよな
この定理こそ人類が解析学の研究で発見した数ある定理の中でも最も重要なものの一つなのに
なぜこの定理がそんなに偉大な定理なのか理解できるのは確かに聞いてすぐ理解できる人間は少ない、しかしそこから「へぇ、そうなんや、なんでやろ」と次の目標を見つけて少しずつ少しずつ階段を上がって行くのが修行なのにこの能無しのクソはそもそも自分に対する過大評価でそれが全くできない
もう生まれついた人格異常なんやろ
どうせこの先も頭打ちの超低レベルなとこウロウロして終わりやからもうやめとけ

729 名前:132人目の素数さん [2022/04/06(水) 17:50:11.06 ID:NqZL91k4.net]
小平邦彦著『解析入門』

この本、記号がひどすぎますね。

D^{+} f(x) = lim_{h→+0} (f(x+h) - f(x)) / h

という右微分係数を表す記号が定義されています。

その後、

D^{+} |sin(π*k)|

などという記号が登場します。

これが

f(x) = |x| の x = sin(π*k) における右微分係数なのか
f(x) = |sin(x)| の x = π*k における右微分係数なのか
f(x) = |sin(π*x)| の x = π*k における右微分係数なのか

D^{+} |sin(π*k)|

を見ただけでは判断できませんよね。

730 名前:132人目の素数さん [2022/04/06(水) 17:51:20.01 ID:NqZL91k4.net]
訂正します:

小平邦彦著『解析入門』

この本、記号がひどすぎますね。

D^{+} f(x) = lim_{h→+0} (f(x+h) - f(x)) / h

という右微分係数を表す記号が定義されています。

その後、

D^{+} |sin(π*k)|

などという記号が登場します。

これが

f(x) = |x| の x = sin(π*k) における右微分係数なのか
f(x) = |sin(x)| の x = π*k における右微分係数なのか
f(x) = |sin(π*x)| の x = k における右微分係数なのか

D^{+} |sin(π*k)|

を見ただけでは判断できませんよね。

731 名前:132人目の素数さん [2022/04/06(水) 17:55:42.56 ID:NqZL91k4.net]
関数を f(x) などと書くのも良くないですね。

732 名前:132人目の素数さん [2022/04/06(水) 19:54:03.29 ID:NqZL91k4.net]
小平邦彦著『解析入門』

病的な関数の紹介が多すぎます。

これは良いことなのか悪いことなのかよく分かりません。

733 名前:132人目の素数さん mailto:sage [2022/04/06(水) 19:57:01.27 ID:jeXzMxlV.net]
>>723
おまえもそのうち精神病理の教科書に載りそう。

734 名前:132人目の素数さん [2022/04/07(木) 06:38:13.77 ID:n18e/PjB.net]
わけわからん



735 名前:132人目の素数さん [2022/04/07(木) 08:06:05.35 ID:I6NqjFX4.net]
>>711
f'(α)=lim_{m/n->α)(1/n-0)/(m/n-α)
=lim_{m/n->α)1/(m-nα)
不定
g(m/n)=1/n^2
g'(α)=lim_{m/n->α)(1/n^2-0)/(m/n-α)
=lim_{m/n->α)1/n(m-nα)
=0

736 名前:132人目の素数さん [2022/04/07(木) 13:12:35.51 ID:or2L+ANl.net]
小平邦彦著『解析入門』

↓定理の成り立つ条件について細かすぎます。

f(x) = f(a) + f'(a)/1! * (x - a) + … + f^(n)(a)/n! * (x - a)^n + o((x - a)^n)

この式が f(x) が I で n - 1 回微分可能で f^(n-1)(x) が点 a で微分可能ならば成立する。

737 名前:132人目の素数さん [2022/04/07(木) 13:15:08.93 ID:or2L+ANl.net]
「I で n 回微分可能であれば、成り立つ」でいいですよね。
↑のコメントは細かすぎませんか?

738 名前:132人目の素数さん mailto:sage [2022/04/07(木) 13:52:34.61 ID:jPOlDp66.net]
相変わらずの能無しぶり

739 名前:132人目の素数さん [2022/04/07(木) 17:03:00.64 ID:0k42bftw.net]
実際に

>「I で n 回微分可能であれば、成り立つ」

と本に書かれてたら「Iでn回微分可能じゃなくても成り立ちますよね。証明もそのまま変わらないのに余計な仮定をつけるなんて小平さんは大丈夫な人(ry」とケチつけてたんだろうなあ

740 名前:132人目の素数さん [2022/04/07(木) 17:30:27.31 ID:or2L+ANl.net]
小平邦彦著『解析入門』

x_1 ≠ x_2
λ

741 名前: + μ = 1

f(λ*x_1 + μ*x_2) < λ*f(x_1) + μ*f(x_2)

が常に成り立つならば、 f(x) は狭義に凸であるという。

これだと狭義に凸であるような関数は存在しないことになってしまいますね。
λ = 0 or μ = 0 のときには不等式が成り立たないからです。
[]
[ここ壊れてます]

742 名前:132人目の素数さん [2022/04/07(木) 17:37:49.87 ID:BeIyTjXH.net]
数学の本は間違いを直しながら読むもの
上の例はどう訂正すればよいかすぐにわかる

743 名前:132人目の素数さん mailto:sage [2022/04/07(木) 19:18:24.30 ID:/Oz/8ydl.net]
自分の無能ぶりを指摘されると今度はムキになってしょうもない粗探し
精神構造が小学生
学問的才覚以前の問題

744 名前:132人目の素数さん mailto:sage [2022/04/07(木) 20:50:22.71 ID:1EFZZmtr.net]
勉強してますアピールの日報代わりに書き込んでるような内容。



745 名前:132人目の素数さん mailto:sage [2022/04/08(金) 03:19:08.07 ID:YBywbTF1.net]
>>723
数学以外をバックグラウンドに持つ人と話して何かの反例を出したりすると病的という単語で逃げることが多いね
それより君の主張が間違ってたことに対する訂正が先だろと思いながら見てる

746 名前:132人目の素数さん [2022/04/08(金) 08:07:03.90 ID:IJAwejbE.net]
小平邦彦著『解析入門』

ψ(x) = 0 if x ≦ 0
ψ(x) = e^{-1/x} if x > 0

という関数が C^∞ 級ではあるが、実解析的ではない例として登場します。

もちろん、 C^∞ 級の関数なので、任意の n に対して、Taylorの公式

ψ(x) = (ψ^(n)(ξ) / n!) * x^n

が成り立ちます。

x > 0 とすると、

ψ(x) = (ψ^(n)(ξ) / n!) * x^n、 0 < ξ < x

です。

n → ∞ のとき、 (ψ^(n)(ξ) / n!) * x^n → 0 とならない。

当たり前のことが書いてあります。

747 名前:132人目の素数さん [2022/04/08(金) 08:11:10.64 ID:IJAwejbE.net]
n → ∞ のとき、
ψ^(n)(ξ) = n! * e^{-1/x} / x^n → ∞ ですが、

lim_{x → +0} ψ^(n)(x) = 0

であるにもかかわらず、

x としていかに小さい値をとって固定しても、

n → ∞ のとき、 ψ^(n)(ξ) → ∞ になるというのは不思議じゃないですか?

もちろん、 ξ は n に依存しますが、これはどう考えればいいのでしょうか?

748 名前:132人目の素数さん mailto:sage [2022/04/08(金) 08:28:47.04 ID:T5T5pA/V.net]
>>736
実解析的とC^∞の定義を理解し損ってる
そこの違いを明確にしときなさいという例だよ
まだそのレベルか

749 名前:132人目の素数さん [2022/04/08(金) 08:53:48.04 ID:IJAwejbE.net]
>>738

Taylor展開はできませんが、Taylorの公式は任意の n に対して、 C^n 級なので成り立ちます。

750 名前:132人目の素数さん [2022/04/08(金) 09:10:20.85 ID:IJAwejbE.net]
e^{-1/x} の n 階導関数のグラフって x = 0 の近くでのグラフを書いてみて納得しました。
普通じゃない関数なんですね。

751 名前:132人目の素数さん [2022/04/08(金) 09:11:09.66 ID:IJAwejbE.net]
訂正します:

e^{-1/x} の n 階導関数のグラフを x = 0 の近くで描いてみて納得しました。
普通じゃない関数なんですね。

752 名前:132人目の素数さん [2022/04/08(金) 09:19:20.91 ID:IJAwejbE.net]
x を 0 に近い値に固定する。

lim_{n → ∞} exp^{-1/x} / x^n = +∞

ですね。

でも、

lim_{x → +0} exp^{-1/x} / x^n = 0

なんですね。

異常です。

753 名前:132人目の素数さん mailto:sage [2022/04/08(金) 09:23:46.73 ID:wUCOOvCy.net]
その程度がこれだけ本読んできてまだわかってないのが異常なんだよ能無し
粗探しばっかりしてるからホントに大切なポイント外して読んだ“フリ”しか出来てない能無しなんだよ
そしてコレは心の問題、一生解決できんやろ
今のまんまの初心者レベルで一生終わる
能無し

754 名前:132人目の素数さん [2022/04/08(金) 09:56:46.02 ID:IJAwejbE.net]
小平邦彦著『解析入門』

f, g を R 上の C^∞ 関数とする。
a, b を a < b であるような実数とする。
ε を任意の正の実数とする。

x ≦ a - ε のとき、 h(x) = f(x)
a ≦ x ≦ b のとき、 h(x) = g(x)
b + ε ≦ x のとき、 h(x) = f(x)

となるような R 上の C^∞ 関数 h が存在する。

これに類する定理をいくつか挙げていますが、どれも以下の ψ という一つの特殊な関数に頼り切っていますね。
結果自体は面白いですが、 ψ 一つに頼り切っていて異常な状況ですよね。

ψ(x) = 0 if x ≦ 0
ψ(x) = e^{-1/x} if x > 0



755 名前:132人目の素数さん [2022/04/08(金) 14:59:30.64 ID:IJAwejbE.net]
小平邦彦著『解析入門』

記述が非常に丁寧な点は評価できますが、ネチネチとしていますね。

756 名前:132人目の素数さん [2022/04/08(金) 15:15:35.42 ID:IJAwejbE.net]
小平邦彦著『解析入門』

定積分のところですが、区間 [a, b] の分割のmeshを δ[Δ] とします。

リーマン和の極限の式

s = lim_{δ[Δ] → 0} Σ_{k=1}^{m} f(ξ_k) * (x_{k} - x_{k-1})

の後に、「δ[Δ] → 0 のとき m → +∞ となることはいうまでもない。」

などと書いています。

これを正確に述べると、


任意の正の実数 M に対し、 正の実数 δ_0 で、

δ[Δ] < δ_0 を満たすような任意の分割 Δ に対し、 Δ の分割された区間の個数 m は M < m を満たす

ようなものが存在する。


で合っていますか?

757 名前:132人目の素数さん [2022/04/08(金) 15:19:29.69 ID:IJAwejbE.net]
>>746

まるで極限 s は δ[Δ] → 0 としないと得られないかのような書き方ですが、 f が
定数関数の場合には、区間 [a, b] を分割する必要すらないですよね。

758 名前:132人目の素数さん [2022/04/08(金) 16:58:03.49 ID:IJAwejbE.net]
小平邦彦著『解析入門』

「∫_{0}^{b} x^2 dx を定積分の定義から直接求めてみよう。」

などと書いて、

分割 Δ を与えたとき、

3 * Σ_{k=1}^{m} ξ_k^2 * (x_{k} - x_{k-1}) = b^3

となるような ξ_k を求めた上で、

∫_{0}^{b} x^2 dx = b^3/3 であると書いています。

これって説明が足らないですよね。

∫_{0}^{b} x^2 dx ≠ b^3/3 ならば、矛盾することを背理法で示さないといけないですよね。

759 名前:132人目の素数さん [2022/04/08(金) 17:43:56.81 ID:o5aOzIlv.net]
>>735
多分あなたが屁理屈をこねているだけだと思う

760 名前:132人目の素数さん mailto:sage [2022/04/09(土) 04:57:53.91 ID:/FFR+xcg.net]
>>749
反論できないけど何とかして反論したい人がよく屁理屈という言葉使うね

761 名前:132人目の素数さん [2022/04/09(土) 11:05:27.07 ID:VGfmJKH7.net]
https://en.wikipedia.org/wiki/Vandermonde%27s_identity
の1つ目の恒等式で
(右辺の部分和)/(左辺) の値を計算(評価)する一般的な方法はありますか?

762 名前:132人目の素数さん [2022/04/09(土) 13:31:00.92 ID:v4RdLh0t.net]
積分の平均値の定理って何の役に立つんですか?

763 名前:132人目の素数さん mailto:sage [2022/04/09(土) 13:49:54.04 ID:0UGdv1bB.net]
いろんなところで役に立つ

764 名前:132人目の素数さん [2022/04/09(土) 16:26:44.83 ID:v4RdLh0t.net]
池田岳著『テンソル代数と表現論: 線型代数続論』

結局、注文してしまいました。
書店でぱらぱら見た感じでは、そんなに分かりやすい本という感じではありませんでしたが。



765 名前:132人目の素数さん [2022/04/09(土) 16:32:32.76 ID:zaGY4urx.net]
まだ読んでいませんが、分かりやすい本という感じではありません。

池田岳さんは大丈夫な人なのでしょうか。

766 名前:132人目の素数さん [2022/04/09(土) 17:23:01.16 ID:v4RdLh0t.net]
杉浦光夫著『解析入門I』

積分の定義をリーマン和の極限で定義していたんですね。
小平邦彦の本でもそうですね。

767 名前:132人目の素数さん [2022/04/09(土) 18:05:09.56 ID:heLwMQOE.net]
>>750
それはあなたの理屈がそれに該当する事を意味しない

768 名前:132人目の素数さん [2022/04/09(土) 19:16:19.39 ID:v4RdLh0t.net]
以下の命題は正しいか正しくないか?

g(x) は点 b で微分できないとする。
f(x) は点 a で微分可能とする。
b = f(a) とする。
g(f(x)) は点 a で微分できない。

769 名前:132人目の素数さん [2022/04/09(土) 19:22:03.08 ID:v4RdLh0t.net]
g(x) = √x は x = 0 で微分できない。
f(x) = x^2 - 1 は x = 1 で微分可能である。

g(f(x)) = √(x^2 - 1) は x = 1 で微分できない。

770 名前:132人目の素数さん [2022/04/09(土) 19:23:34.82 ID:v4RdLh0t.net]
>>758

小平邦彦著『解析入門』の原始関数の表を眺めていて、思いついた問題です。

771 名前:132人目の素数さん [2022/04/09(土) 19:48:02.38 ID:v4RdLh0t.net]
正解は「正しくない」です。

例:

g(x) = x^{1/3}
f(x) = x^3

g(x) は x = 0 で微分可能でない。
g(f(x)) は x = 0 で微分できる。

772 名前:132人目の素数さん [2022/04/09(土) 20:02:30.95 ID:v4RdLh0t.net]
同様に以下も正しくありません。

g(x) は点 b で微分できるとする。
f(x) は点 a で微分できないとする。
b = f(a) とする。
g(f(x)) は点 a で微分できない。

773 名前:132人目の素数さん [2022/04/09(土) 20:06:05 ID:v4RdLh0t.net]
そこで質問があります。

f(x) = log(|x|)
g(x) = x + √(x^2 - 1)

とします。

g(x) は x = 1 で微分できません。
f(x) は x = g(1) = 1 で微分できます。

f(g(x)) が x = 1 で微分できないことはわざわざ微分の定義に戻って確かめる必要があるでしょうか?

774 名前:132人目の素数さん [2022/04/09(土) 20:09:12 ID:v4RdLh0t.net]
ちなみに、 f(g(x)) は (-∞, -1) ∪ (1, +∞) で微分できて、導関数は、 1/√(x^2 - 1) です。



775 名前:132人目の素数さん [2022/04/09(土) 20:25:16 ID:v4RdLh0t.net]
ちなみに、小平邦彦著『解析入門』に以下の定理があります:

p.125 定理3.10

区間 [c, b) で定義された連続関数 f(x) が、 (c, b) で微分可能で lim_{x → c+0} f'(x) が存在するならば、 f(x) は c においても微分可能で

f'(c) = lim_{x → c+0} f'(x).

776 名前:132人目の素数さん [2022/04/09(土) 20:26:56 ID:v4RdLh0t.net]
以下は正しいですか?

区間 [c, b) で定義された連続関数 f(x) が、 (c, b) で微分可能で lim_{x → c+0} f'(x) が存在しないならば、 f(x) は c においても微分できない。

777 名前:132人目の素数さん [2022/04/09(土) 20:27:51 ID:v4RdLh0t.net]
>>766

訂正します:

以下は正しいですか?

区間 [c, b) で定義された連続関数 f(x) が、 (c, b) で微分可能で lim_{x → c+0} f'(x) が存在しないならば、 f(x) は c において微分できない。

778 名前:132人目の素数さん mailto:sage [2022/04/09(土) 20:52:58.85 ID:g3mdVD+B.net]
正しくない

779 名前:132人目の素数さん [2022/04/09(土) 22:03:58.60 ID:v4RdLh0t.net]
>>768

では、以下は正しいですか?

区間 [c, b) で定義された連続関数 f(x) が、 (c, b) で微分可能で lim_{x → c+0} f'(x) = +∞ or -∞ ならば、 f(x) は c において微分できない。

780 名前:132人目の素数さん [2022/04/09(土) 22:12:23.70 ID:v4RdLh0t.net]
>>769

正しいですね。
定理3.10と全く同じ証明で示せますね。

781 名前:132人目の素数さん [2022/04/09(土) 22:20:22 ID:v4RdLh0t.net]
ということで、

↓わざわざ確かめる必要はないということになりますね。

f(g(x)) が x = 1 で微分できないことはわざわざ微分の定義に戻って確かめる必要があるでしょうか?

782 名前:132人目の素数さん [2022/04/09(土) 23:16:09.19 ID:ORLs89zo.net]
>>771
わざわざ>>769を確かめる必要も無く
f(g(x))がx=1で微分できないことを微分の定義に戻って確かめることで示せますね

783 名前:132人目の素数さん [2022/04/10(日) 11:04:49.83 ID:A0iJeNrk.net]
杉浦光夫著『解析入門1』

多変数のテイラーの定理についてはもちろん書いてあるのですが、
多変数の関数のテイラー展開については何も書いてありません。

他の本でも1変数の場合にはテイラー展開について書いてあるのに、多変数になると
テイラーの定理しか書いてありません。

小平邦彦著『解析入門』には多変数のテイラー展開の例は出てきませんが、テイラー展開
の定義についてのみ書いてあります。例はありません。

784 名前:132人目の素数さん [2022/04/10(日) 11:05:38.66 ID:A0iJeNrk.net]
なぜですか?



785 名前:132人目の素数さん [2022/04/10(日) 11:14:12.02 ID:A0iJeNrk.net]
小平邦彦著『解析入門』

f, g が C^n 級ならば、 a*f + b*g, f*g, f/g も C^n 級であること
f, g が C^n 級ならば、 g(f(x)) も C^n 級であること
単調関数 f が C^n 級ならば、 f^{-1} も C^n 級であること

を非常に丁寧に証明しています。

杉浦光夫著『解析入門1』では、これらの定理のステートメントすら書いてありません。

杉浦光夫さんは一体何を考えていたのでしょうか?

786 名前:132人目の素数さん [2022/04/10(日) 11:58:00 ID:A0iJeNrk.net]
一松信著『解析学序説上巻(旧版)』

「積分には、区間内で有界でない点のある場合、および無限区間での積分が必要である。」

などと書かれています。

「有界でない点」とは一体何でしょうか?

関数 f がある区間で有界でないというのなら意味が通じます。
「関数 f がある区間内の点で有界でない」とは一体何を意味するのでしょうか?

一松信さんは大丈夫な人なのでしょうか?

787 名前:132人目の素数さん [2022/04/10(日) 12:16:17 ID:X2RwtncV.net]
「大丈夫な人なのでしょうか」ってかなり破壊力あるフレーズだよね

788 名前:132人目の素数さん mailto:sage [2022/04/10(日) 12:21:48 ID:sEjts1xl.net]
>>ID:A0iJeNrk
統失、薬飲んでるか?

789 名前:132人目の素数さん [2022/04/10(日) 12:38:12 ID:A0iJeNrk.net]
一松信著『解析学序説上巻(新版)』でも依然として、

「積分には、区間内で有界でない点のある場合、および無限区間での積分が必要である。」

などと書かれています。

790 名前:132人目の素数さん mailto:sage [2022/04/10(日) 13:05:33 ID:RxqB7OvB.net]
クロスエントロピー誤差の偏微分って出力変数の合計が1になるって制約は考えなくていいのはナゼ(・・?
出力変数がz1とz2の2つならz1について偏微分するときはz2=1-z1としなくていい?

791 名前:132人目の素数さん mailto:sage [2022/04/10(日) 15:59:38.26 ID:neV5spWy.net]
爺さんたちの日本語は勿体ぶって偉そうに書いてるだけで実際は雑
適当に雰囲気を読み取って解釈するしかない

792 名前:132人目の素数さん [2022/04/10(日) 22:20:12 ID:cxWVCRxO.net]
とりあえず数論には手を出すな
というのが伝わるNHKスペシャルだった

793 名前:132人目の素数さん [2022/04/10(日) 22:34:39 ID:Eb5aj5pr.net]
まゆゆはでたの?まゆまゆ!

794 名前:132人目の素数さん [2022/04/10(日) 23:10:08 ID:hmV4WVUe.net]
>>777
大丈夫かどうか怪しい人がそれ書いてるしね



795 名前:132人目の素数さん mailto:sage [2022/04/10(日) 23:20:19 ID:kjm0hrhA.net]
まぁ直らんわな
直す気もないだろうし
どうでもいい

796 名前:132人目の素数さん [2022/04/11(月) 07:45:34.73 ID:Pz4vsRKO.net]
小平邦彦著『解析入門』

区間 (a, b) で連続な関数 f(x) に対して lim_{t → b-0, s → a+0} ∫_{s}^{t} f(x) dx
が存在するならば

(4.35) ∫_{a}^{b} f(x) dx = lim_{t → b-0, s → a+0} ∫_{s}^{t} f(x) dx

と定義する。ここで(4.35)は任意の正の実数 ε に対応して一つの正の実数 δ(ε) が定まって、
b - δ(ε) < t < b, a < s < a + δ(ε) ならば

|∫_{a}^{b} f(x) dx - ∫_{s}^{t} f(x) dx| < ε

となることを意味するが、点 c, a < c < b, を一つ定めれば

∫_{s}^{t} f(x) dx = ∫_{s}^{c} f(x) dx + ∫_{c}^{t} f(x) dx

であるから

lim_{t → b-0, s → a+0} ∫_{s}^{t} f(x) dx = lim_{s → a+0} ∫_{s}^{c} f(x) dx + lim_{t → b-0} ∫_{c}^{t} f(x) dx

したがって(4.35)は

∫_{a}^{b} f(x) dx = lim_{s → a+0} ∫_{s}^{c} f(x) dx + lim_{t → b-0} ∫_{c}^{t} f(x) dx

とも書かれる。

797 名前:132人目の素数さん [2022/04/11(月) 07:48:45.73 ID:Pz4vsRKO.net]
lim_{t → b-0, s → a+0} ∫_{s}^{t} f(x) dx

が存在するときに、

lim_{s → a+0} ∫_{s}^{c} f(x) dx および lim_{t → b-0} ∫_{c}^{t} f(x) dx

が両方とも存在することを証明しなければなりませんが、していませんね。

lim_{s → a+0} ∫_{s}^{c} f(x) dx および lim_{t → b-0} ∫_{c}^{t} f(x) dx

の両方の存在が証明されれば、

lim_{t → b-0, s → a+0} ∫_{s}^{t} f(x) dx = lim_{s → a+0} ∫_{s}^{c} f(x) dx + lim_{t → b-0} ∫_{c}^{t} f(x) dx

は自明と言ってもいいと思いますが、

im_{s → a+0} ∫_{s}^{c} f(x) dx および lim_{t → b-0} ∫_{c}^{t} f(x) dx

の両方の存在の証明は、決して自明なことではありません。

798 名前:132人目の素数さん [2022/04/11(月) 07:50:28.54 ID:Pz4vsRKO.net]
杉浦光夫さんの『解析入門1』でも、同じ過ちを犯しています。

799 名前:132人目の素数さん [2022/04/11(月) 07:54:02.28 ID:Pz4vsRKO.net]
そこで以下の問題を出しておきます:

lim_{s → a+0} ∫_{s}^{c} f(x) dx および lim_{t → b-0} ∫_{c}^{t} f(x) dx

が両方とも存在することを証明せよ。

800 名前:132人目の素数さん [2022/04/11(月) 09:08:17.49 ID:Pz4vsRKO.net]
>>789

目標となる極限値があらかじめ与えられていないところが難しいところだと思います。

801 名前:132人目の素数さん mailto:sage [2022/04/11(月) 09:13:20.22 ID:BbeHwTpV.net]
ええ加減にせい能無し
お前にこの板で問題出すほどの実力あるわけないやろカス

802 名前:132人目の素数さん [2022/04/11(月) 09:17:52 ID:Pz4vsRKO.net]
広義積分って定義だけ見ると、非常に人工的に見えますけど、ガンマ関数とか重要な関数が
広義積分を使って定義されるんですよね。

803 名前:132人目の素数さん [2022/04/11(月) 10:53:54 ID:Pz4vsRKO.net]
小平邦彦著『解析入門』

広義積分のところで、普通の積分について成り立つ命題をいちいち広義積分の場合にも証明していて、
面倒くさすぎます。

804 名前:132人目の素数さん [2022/04/11(月) 11:38:12 ID:/PWg5M3T.net]
>>793
自明じゃないからね



805 名前:132人目の素数さん [2022/04/11(月) 11:59:58.32 ID:UoGGbG9Q.net]
そんなに面倒くさいなら読まなければいいだけ

806 名前:132人目の素数さん [2022/04/11(月) 18:37:30 ID:Pz4vsRKO.net]
>>789

解答がありませんね。
難しすぎましたかね。

807 名前:132人目の素数さん mailto:sage [2022/04/11(月) 19:50:21.99 ID:8ttuGPfz.net]
そう言ったら相手にしてもらえると思ってる時点で小学生なんだよ
そしてそれがお前が数学できない全ての理由なんだよ

808 名前:132人目の素数さん [2022/04/11(月) 21:33:07 ID:Pz4vsRKO.net]
>>789

ヒントを出しておきます:

コーシーの判定法を使う。

809 名前:132人目の素数さん [2022/04/12(火) 09:21:06 ID:PrHDB321.net]
R[x] ∋ x^2 + 1 とする。

x^2 + 1 = 0 が R に解を持たないことを証明せよと言われたら、
R の順序に関する性質を使って証明すると思います。

R を四則演算のみ定義された可換体と考えるときには、どうやって証明しますか?

810 名前:132人目の素数さん [2022/04/12(火) 09:25:26.07 ID:PrHDB321.net]
R を四則演算のみ定義された可換体と考えるときに、そもそも

x^2 + 1 = 0

に解は存在しませんか?

811 名前:132人目の素数さん [2022/04/12(火) 09:29:34.67 ID:PrHDB321.net]
R を構成するときには、順序が必要です。
順序体 R を作った後に、順序については忘れるということをすると R は一体何になるんですか?

812 名前:132人目の素数さん [2022/04/12(火) 09:34:52.18 ID:PrHDB321.net]
>>799-800
順序を忘れた可換体 R は順序体 R と同形だから x^2 + 1 = 0 は解を持ちませんね。

813 名前:132人目の素数さん mailto:sage [2022/04/12(火) 09:41:26.96 ID:aaJo9gW4.net]
>>799
「R[x] ∋ x^2 + 1 とする」などと書かれています。
x^2+1は初めからR[x]の元なので、著者がそう置いたかのような書き方はおかしいですね。

814 名前:132人目の素数さん [2022/04/12(火) 09:58:39.60 ID:PrHDB321.net]
順序体 R を作った後に、順序については忘れた体を S とします。
S を順序を考えずに構成できますか?



815 名前:132人目の素数さん [2022/04/12(火) 10:35:07.70 ID:PrHDB321.net]
石田信著『代数学入門』

F = Z/(p) とする。
F[X] の元 X^p - a について考える。
フェルマの定理によって a^p = a であるから、 X^p - a = (X - a)^p である。

というような話が書いてあります。

a^p = a から分かるのは、 X = a が X^p - a = 0 の解であるということだけですよね。

普通、 (X - a)^p を展開すると p 次の係数と 0 次の係数以外はすべて 0 になるということを確認して、
(X - a)^p = X^p - a を証明しますよね。

816 名前:132人目の素数さん [2022/04/12(火) 11:22:41.84 ID:nG/E6vR2.net]
>>801
順序を忘れたRになるだけ

817 名前:132人目の素数さん [2022/04/12(火) 11:24:04.40 ID:nG/E6vR2.net]
>>804
作ろうと頑張ってみてよ
作れないことが証明できたら良いと思うけど
そういう証明が歩かないかは知らない

818 名前:132人目の素数さん [2022/04/12(火) 11:25:35.32 ID:nG/E6vR2.net]
>>805
p乗が体の準同形だからだけど
それは2項定理から証明する

819 名前:132人目の素数さん [2022/04/12(火) 12:00:47 ID:ZbLim+zU.net]
構うなよ、、、

820 名前:132人目の素数さん [2022/04/13(水) 06:33:44.29 ID:0ixtg4GU.net]
質問いいですか

821 名前:132人目の素数さん [2022/04/13(水) 06:52:11.62 ID:0ixtg4GU.net]
大学で
空集合の定義を
Φ:={}

0の定義を
0:=Φ

1の定義を
1:={Φ}

と習ったのですがこれって
1={Φ}={0}={{}}だから、

{{}}⊇ {}は真。逆は偽。よって{{}}≠ {}
{{}}∋{}は真。逆は偽。
っていうところまではよかったんですけど、

{{}}⊃{}って真ですか?偽ですか?
{{},{}}⊃{}は真だと思うんですけれど……

822 名前:132人目の素数さん [2022/04/13(水) 06:59:31.20 ID:0ixtg4GU.net]
先生に聞いたら{{},{}}={{}}とするみたいで、
でもそれだとやっぱり{{}}⊃{}の真偽が定まりません。???

823 名前:132人目の素数さん [2022/04/13(水) 07:15:31.82 ID:0ixtg4GU.net]
解決しました。

824 名前:132人目の素数さん [2022/04/13(水) 08:53:40.28 ID:pIEgW9a2.net]
>>811
何を誤解していたか読めない



825 名前:132人目の素数さん [2022/04/13(水) 12:26:07.03 ID:FgKJOfZP.net]
第2同型定理HN/N=H/(H∩N)の証明が以下のページにあります
https://レポート代行.com/%e4%bb%a3%e6%95%b0%e5%ad%a6/%e7%ac%ac2%e5%90%8c%e5%9e%8b%e5%ae%9a%e7%90%86
「この写像が well-defined であることは、以下のようにして分かる。
h1N=h2N, (h1,h2∈H) とする。
すなわち、ある n1,n2∈N が存在して、h1∘n1=h2∘n2 が成り立つ。
このとき、
h1∘n1=h2∘n2
h1=h2∘n2∘n1^(−1)
h1(H∩N)=(h2∘n2∘n1^(−1))(H∩N)(*1)
h1(H∩N)=(h2∘n2)(H∩N)(*2)
h1(H∩N)=h2(H∩N)(*3)
より、h1(H∩N)=h2(H∩N) が言える。
従って、この写像 φ は well-defined である。」
(*1)から(*2)、(*2)から(*3)が成立する理由が分かりませんでした。
どうやれば示せますか?

826 名前:132人目の素数さん mailto:sage [2022/04/13(水) 14:30:11.88 ID:YQZhWMvU.net]
h2^(-1)h1=n1^(-1)n2∈Nだからh2^(-1)h1N ⊂ N
∴ h1N = h2h2^(-1)h1N ⊂ h2N
逆も同様

827 名前:132人目の素数さん [2022/04/13(水) 15:54:14.14 ID:FgKJOfZP.net]
>>816
ありがとうございます。
リンク先では写像を
「φ(hN)=h(H∩N) で定める。
この写像が well-defined であることは、以下のようにして分かる。」
とあります。
(*1)から(*2)の
(h2∘n2∘n1^(−1))(H∩N)=(h2∘n2)(H∩N)
はどうやれば示せるのでしょうか?

828 名前:132人目の素数さん [2022/04/13(水) 16:19:29.28 ID:BNHMSGAw.net]
(X、d)を距離空間とする、Xの部分集合A、Bに対して
dist(A,B)=inf{d(a,b)|a in A ,b in B}とおく
って書いてあるのですが、dist(A,B)はうまく定義されてるのでしょうか。

829 名前:132人目の素数さん [2022/04/13(水) 16:38:51.73 ID:FgKJOfZP.net]
距離関数はd(a,b)≧0なので下に有界でinfは存在するから問題ない。

830 名前:132人目の素数さん [2022/04/13(水) 16:46:57.90 ID:cqeXNhVh.net]
>>819
xをBの元でないXの元とすると
任意のεに対してあるBの元bがあってd({x}、b) ≤β+εを満たすβって存在するでしょうか。

831 名前:132人目の素数さん [2022/04/13(水) 16:57:14.22 ID:FgKJOfZP.net]
>>820
dは実数値関数。実数の性質。デデキントの切断から証明できる。
§3 上限と下限 定理1(A)
https://nekodamashi-math.blog.ss-blog.jp/2018-03-29-1

832 名前:132人目の素数さん [2022/04/13(水) 17:16:21.92 ID:fBdRfzsR.net]
>>815
人の証明読むんでなくて自分で考えてみたらどうかも

833 名前:132人目の素数さん [2022/04/13(水) 17:21:35.50 ID:4+vDbrq9.net]
>>821
{d(a,b)|a A b B}は実数の集合で下に有界だからinfは存在するのか。ありがとうございます。

834 名前:132人目の素数さん [2022/04/13(水) 23:27:25.32 ID:apLYO+gu.net]
【質問】行列の積は行に対して列を掛けますが、和の演算では同じ行・列のものどうし
を足します。なんでこのようになるのですか?
行列どうしの積の意味は何ですか?



835 名前:132人目の素数さん [2022/04/14(木) 00:36:27 ID:QYH2In8M.net]
>>815
h_1N=h_2N
h_2^{-1}h_1N=N
よってh_2^{-1}h_1 ∈N
よってあるn ∈Nがあって、h_2^{-1}h_1 =nと書ける
よってn ∈Hである
h_2^{-1}h_1 =nの両辺にh_2をかけて
h_1= h_2 nよって
h_1(H ⋂N)= h_2 n (H ⋂N)
nはHの元でもNの元でもあるのでH ⋂Nに吸収されて
h_1(H ⋂N )=h_2 (H ⋂N)

836 名前:132人目の素数さん [2022/04/14(木) 01:22:20.44 ID:5x5JkEZd.net]
>>825
ありがとうございます。理解できました。
元のサイトの説明だと
(h2∘n2∘n1^(−1))(H∩N)
=(h2∘n2)(H∩N)
としているのですが、これは成り立たないですよね?
n1^(-1)∈H∩N
とまでは言えない。

837 名前:132人目の素数さん [2022/04/14(木) 03:22:27 ID:uHdSj82h.net]
自分の頭の悪さを本の説明の悪さに転嫁する馬鹿がこのスレの常連さんなので、そういう書き方には賛同しにくい。

838 名前:132人目の素数さん [2022/04/14(木) 07:54:13 ID:4rat+pCv.net]
雪江明彦著『代数学2 環と体とガロア理論』

deg f(x) * g(x) = deg f(x) + deg g(x) であることの証明中で以下の事実が証明なしで使われています。
自明ではないですよね?

--------------------------------------------------------------------------------

A を整域とする。
x = (x_1, …, x_n) を変数とする。
f(x), g(x) を A[x] の元とする。
f(x) は l 次斉次式式、 g(x) は m 次斉次式とする。

このとき、 f(x) * g(x) は (l + m) 次斉次式である。

839 名前:132人目の素数さん [2022/04/14(木) 07:55:10 ID:4rat+pCv.net]
訂正します:

雪江明彦著『代数学2 環と体とガロア理論』

deg f(x) * g(x) = deg f(x) + deg g(x) であることの証明中で以下の事実が証明なしで使われています。
自明ではないですよね?

--------------------------------------------------------------------------------

A を整域とする。
x = (x_1, …, x_n) を変数とする。
f(x), g(x) を A[x] の元とする。
f(x) は l 次斉次式、 g(x) は m 次斉次式とする。

このとき、 f(x) * g(x) は (l + m) 次斉次式である。

840 名前:132人目の素数さん [2022/04/14(木) 07:56:26.61 ID:4rat+pCv.net]
うまくキャンセルされて f(x) * g(x) = 0 となってしまう可能性がありますが、そういうことは
起こらないということを証明しなければならないですよね?

841 名前:132人目の素数さん [2022/04/14(木) 08:03:01.88 ID:4rat+pCv.net]
あ、成り立つ理由が分かりました。

ですが、自明とまではいえないと思います。

842 名前:132人目の素数さん [2022/04/14(木) 08:53:53.49 ID:4rat+pCv.net]
f(x) の各項のうち辞書式順序に関して最大の項を a*x_1^{i_1}*…*x_n^{i_n} とする。
g(x) の各項のうち辞書式順序に関して最大の項を b*x_1^{j_1}*…*x_n^{j_n} とする。
f(x) * g(x) の各項のうち辞書式順序に関して最大の項は a*b*x_1^{i_1+j_1}*…*x_n^{i_n+j_n} になる。

ゆえに、 f(x) * g(x) ≠ 0 である。

843 名前:132人目の素数さん [2022/04/14(木) 09:22:45 ID:4rat+pCv.net]
訂正します:

f(x) の次数最大の各項のうち辞書式順序に関して最大の項を a*x_1^{i_1}*…*x_n^{i_n} とする。
g(x) の次数最大の各項のうち辞書式順序に関して最大の項を b*x_1^{j_1}*…*x_n^{j_n} とする。
f(x) * g(x) の次数最大の各項のうち辞書式順序に関して最大の項は a*b*x_1^{i_1+j_1}*…*x_n^{i_n+j_n} になる。

ゆえに、 f(x) * g(x) ≠ 0 である。

844 名前:132人目の素数さん [2022/04/14(木) 09:26:13 ID:zI/25SNd.net]
Aは整域ならA[x](一変数)も整域←自明
帰納的にA[x_1,…x_n]も整域←自明

>>828の主張←自明



845 名前:132人目の素数さん [2022/04/14(木) 09:34:44 ID:4rat+pCv.net]
>>833

あ、訂正の必要はなかったですね。

>>834

あ、そうですね。

846 名前:132人目の素数さん [2022/04/14(木) 10:01:37.27 ID:4rat+pCv.net]
松坂和夫著『代数系入門』
石田信著『代数学入門』

環について本当にベーシックなことしか書いていないですね。
こんなんでいいんですかね?

847 名前:132人目の素数さん [2022/04/14(木) 10:47:51 ID:8l8MzYwb.net]
>>824
森毅の本の説明が分かりやすい
でも分かりやすいのはそこだけ

848 名前:132人目の素数さん [2022/04/14(木) 10:56:07.90 ID:4rat+pCv.net]
>>824

松坂和夫著『代数系入門』のpp.193-194の説明が自然だと思います。

849 名前:132人目の素数さん [2022/04/14(木) 11:02:09.98 ID:4rat+pCv.net]
線形写像 f の表現行列を A
線形写像 g の表現行列を B

とする。

線形写像の合成 f ・ g の表現行列を A * B と定義したいということだと思います。

そうすると結合法則や分配法則などが成り立ちます。

850 名前:132人目の素数さん [2022/04/14(木) 11:12:39.08 ID:zI/25SNd.net]
>>824
高校生なら連立一次方程式を行列の形で書き直して、変数変換したらどうなるか考えてみたら?

851 名前:132人目の素数さん [2022/04/14(木) 11:16:08.57 ID:4rat+pCv.net]
>>840
B*(A*x) = C*x となるような行列 C を B*A と定義するということですね。

852 名前:132人目の素数さん mailto:sage [2022/04/14(木) 11:46:01.11 ID:f0j2UYsy.net]
>>839
こんなおバカな事ばっかり考えてるからいつまで経っても圏論的センスが身につかない
そしてそれが身についてこない事が勉強が次の段階に進まない理由だとわからん能無し

853 名前:132人目の素数さん mailto:sage [2022/04/14(木) 13:09:35.04 ID:GFjlvlg2.net]
だって手帳持ちの真性キチガイだし

854 名前:132人目の素数さん [2022/04/14(木) 13:31:15.82 ID:zI/25SNd.net]
>>841
そんな理解してる謎アピールは要らないです



855 名前:132人目の素数さん [2022/04/14(木) 13:50:18.77 ID:8DUAJbGC.net]
>>826
h_1n_1=h_2n_2
h_1n_1n_2^{-1}=h_2
n_1n_2^{-1}=h_1

856 名前:^{-1}h_2
だからn_1n_2はHの元
[]
[ここ壊れてます]

857 名前:132人目の素数さん [2022/04/14(木) 14:05:36.04 ID:kc6aDZcl.net]
>>845
そこからn_1はHの元またはn_2はHの元は言える?

858 名前:132人目の素数さん [2022/04/14(木) 14:35:21.02 ID:4rat+pCv.net]
前にもかきましたが、松坂和夫著『代数系入門』では、普通、既約元とよばれるものを
素元とよんでいます。

そして、PID上では、任意のゼロでも単元でもない元が、素元の積に一意的に分解されることを
証明しています。

要するに、普通の言葉で言えば、PID上では、任意のゼロでも単元でもない元が、既約元の積に一意的に分解されることを
証明しているわけです。

PID上では素元は既約元であり、既約元は素元です。

このことを悪用したのが『代数系入門』ですね。

859 名前:132人目の素数さん [2022/04/14(木) 14:37:22.41 ID:4rat+pCv.net]
他の代数学の本を読まない読者にとっては、非常に有害ですよね。

860 名前:132人目の素数さん [2022/04/14(木) 16:15:16.29 ID:4rat+pCv.net]
同レベルの本である石田信著『代数学入門』では、きちんと素元と既約元を別々に定義しています。

なぜ松坂和夫さんがあんなことをしたのか理解に苦しみます。

861 名前:132人目の素数さん [2022/04/14(木) 17:25:18.44 ID:4rat+pCv.net]
あ、UFDの定義ですけど、素元に分解されるという定義と既約元に分解されるという定義があるんですね。

862 名前:132人目の素数さん [2022/04/15(金) 10:52:01.37 ID:OUlSMVpT.net]
小平邦彦著『解析入門』

定理4.7(1)
f(x) を開区間 (a, b) で連続な x の関数とする。
広義積分 ∫_{a}^{b} f(x) dx が収束するならば、点 c, a < c < b, を一つ選んで

F(x) = ∫_{c}^{x} f(x) dx

とおいたとき、 F(x) は閉区間 [a, b] で連続、開区間 (a, b) では微分可能で F'(x) = f(x)
である。

--------------------------------------------------------------------------------------------------------

まず微分可能のほうは有名な定理そのものです。
そして、連続のほうは、例えば、 ∫_{c}^{x} f(x) dx が x = b で連続になるように広義積分を定義している
ので明らかです。
わざわざ証明まで書いていますが、定理のステートメント自体不要だと思います。

863 名前:132人目の素数さん [2022/04/15(金) 11:00:20.85 ID:OUlSMVpT.net]
おそらく日本語の本の中で、小平邦彦さんの本が広義積分について一番詳しく書いてあると思いますが、あっていますか?

864 名前:132人目の素数さん [2022/04/15(金) 13:52:08.24 ID:OUlSMVpT.net]
小平邦彦著『解析入門』

広義積分について色々書いています。

例えば、以下の広義積分など使われることは一度でもあるのでしょうか?

関数 f(x) がすべての点 t, t > a に対して (a, t) で高々有限個の点を除いて連続で
広義積分 ∫_{a}^{t} f(x) dx が収束しているとき、極限 lim_{t → +∞} ∫_{a}^{t} f(x) dx
が存在するならば、広義積分 ∫_{a}^{+∞} f(x) dx を

∫_{a}^{+∞} f(x) dx = lim_{t → +∞} ∫_{a}^{t} f(x) dx

と定義し、広義積分 ∫_{a}^{+∞} f(x) dx は収束するという。



865 名前:132人目の素数さん [2022/04/15(金) 13:52:59.62 ID:OUlSMVpT.net]
これなど理論のための理論ではないでしょうか?

866 名前:132人目の素数さん [2022/04/15(金) 14:15:35.90 ID:tLRzmP2n.net]
>>854
顧みられない質問があるのと同じよ

867 名前:132人目の素数さん mailto:sage [2022/04/15(金) 14:29:35.04 ID:u6Ija5cW.net]
>>853
統失のアホは、累積分布関数とか見たことないんか?

868 名前:132人目の素数さん [2022/04/15(金) 19:17:34.23 ID:OUlSMVpT.net]
点 b が第二種不連続点の場合に、広義積分 ∫_{a}^{b} f(x) dx が存在する例、存在しない例ってありますか?

869 名前:132人目の素数さん mailto:sage [2022/04/16(土) 07:16:13.36 ID:kdp3FkZ+.net]
1と0からなる数列a=(a[1],a[2],a[3],...)全体からなる集合Xは連続濃度ですが
その中でa[n]=a[n+m]=a[n+2m],a[n+1]=a[n+m+1]=a[n+2m+1],...,a[n+m-1]=a[n+2m-1]=a[n+3m-1]となるような部分
つまり同じ部分を3回繰り返すような数列(たとえばa=(0,0,1,0,1,0,1,1...)みたいな)をXから取り除いたX'を考えます
X'は空集合じゃなければ無限集合になりそうですが実際濃度はどうなるんでしょうか

870 名前:132人目の素数さん [2022/04/16(土) 07:48:50.57 ID:d6AgvgDx.net]
>>857
簡単に見つかりました。
https://www.wolframalpha.com/input?i2d=true&i=Integrate%5Bsin%5C%2840%29Divide%5B1%2Cx%5D%5C%2841%29%2C%7Bx%2C0%2CDivide%5B1%2Cpi%5D%7D%5D&lang=ja

871 名前:132人目の素数さん [2022/04/16(土) 07:52:15.86 ID:d6AgvgDx.net]
こんな関数でも収束するんですね。
https://www.wolframalpha.com/input?i2d=true&i=Integrate%5BDivide%5B%EF%BC%91%2CPower%5B%EF%BD%98%2C%EF%BC%91%5D%5Dsin%5C%2840%29Divide%5B1%2Cx%5D%5C%2841%29%2C%7Bx%2C0%2CDivide%5B1%2Cpi%5D%7D%5D&lang=ja

872 名前:132人目の素数さん [2022/04/16(土) 07:53:40.42 ID:d6AgvgDx.net]
やっと発散しました。
https://www.wolframalpha.com/input?i2d=true&i=Integrate%5BDivide%5B%EF%BC%91%2CPower%5B%EF%BD%98%2C2%5D%5Dsin%5C%2840%29Divide%5B1%2Cx%5D%5C%2841%29%2C%7Bx%2C0%2CDivide%5B1%2Cpi%5D%7D%5D&lang=ja

873 名前:132人目の素数さん [2022/04/16(土) 08:12:38.43 ID:Zc1rPk1g.net]
>>858
非可算個あるみたいです
https://mathoverflow.net/questions/61615/are-there-uncountably-many-cube-free-infinite-binary-words

874 名前:132人目の素数さん [2022/04/16(土) 09:45:19.49 ID:+uUNq8bS.net]
むしろ物理とかだと積分って広義積分がデフォルトみたいなところがありますよね

積分範囲が∞になってないと面倒だなって思いますね



875 名前:132人目の素数さん mailto:sage [2022/04/16(土) 10:00:41 ID:sj4+BJCN.net]
統失は、物理板にも来ててアホ晒してるわ

876 名前:132人目の素数さん [2022/04/16(土) 14:19:04.22 ID:Iu6Z0Ct6.net]
質問です。
距離空間の直積距離空間と
距離空間からできる距離位相空間の直積空間は同じものになりますか?

877 名前:132人目の素数さん [2022/04/16(土) 17:26:12.57 ID:Zc1rPk1g.net]
有限個の直積なら自明

878 名前:132人目の素数さん [2022/04/16(土) 18:24:00 ID:FzTxMFsC.net]
そして非可算個の直積だとそもそも距離づけ不可能

879 名前:132人目の素数さん [2022/04/16(土) 19:25:01.32 ID:d6AgvgDx.net]
池田岳著『テンソル代数と表現論』が届きました。

これから読み始めようと思います。

880 名前:132人目の素数さん [2022/04/16(土) 19:27:46.45 ID:IB0OBOos.net]
これ↓コピペして使っていいよ

池田岳さんは大丈夫な人なのでしょうか?

881 名前:132人目の素数さん [2022/04/16(土) 19:28:46.90 ID:IB0OBOos.net]
スレとあんまり関係ないけどIDが結構かっこいい

882 名前:132人目の素数さん [2022/04/17(日) 14:53:34.62 ID:WHuG1b+m.net]
池田岳著『テンソル代数と表現論』

カバーと帯の配色が綺麗ですね。

883 名前:132人目の素数さん [2022/04/17(日) 16:51:17.32 ID:WHuG1b+m.net]
池田岳著『テンソル代数と表現論』

第1章の途中まで読みましたが、よくまとまっていて、読みやすいと思います。

884 名前:132人目の素数さん [2022/04/18(月) 00:33:40.37 ID:HsfpgeqQ.net]
↓コピペでどうぞ

池田さんは一体どんな数学的センスの持ち主なのでしょうか?



885 名前:132人目の素数さん [2022/04/18(月) 11:51:56.32 ID:KcHBreVd.net]
質問です。
K=C(複素数)上のベクトル空間をVc、KをR(実数)に制限したベクトル空間をVrとします。
dimVc=dimVr は成り立ちますか?
成り立たないから反例を成り立つなら証明を教えて下さい。

886 名前:132人目の素数さん [2022/04/18(月) 11:55:58.40 ID:BGO5j9mA.net]
C 上のベクトル空間 C は1次元ベクトル空間
R 上のベクトル空間 C は2次元ベクトル空間

887 名前:132人目の素数さん [2022/04/18(月) 12:01:25.44 ID:KcHBreVd.net]
>>875
了解ですw
さすが早いですね…。

888 名前:132人目の素数さん [2022/04/18(月) 12:56:10.28 ID:BGO5j9mA.net]
池田岳著『テンソル代数と表現論』

「〜がしたがう。」という非常に奇妙な日本語を多用しています。

889 名前:132人目の素数さん [2022/04/18(月) 13:02:13.65 ID:BGO5j9mA.net]
池田岳著『テンソル代数と表現論』

ジョルダン分解の話を読み終われば、第1章を無事読み終えることになります。
第1章は非常に分かりやすいです。

890 名前:132人目の素数さん [2022/04/18(月) 22:55:36.39 ID:9Ip71OTU.net]
>>876
というか
例を自分で考えてみたらこれはすぐ思いつかねばならないのに

891 名前:132人目の素数さん [2022/04/19(火) 11:49:58.52 ID:TCoFcnyb.net]
池田岳著『テンソル代数と表現論』

第1章を読み終わりました。非常に分かりやすかったです。

892 名前:132人目の素数さん [2022/04/19(火) 18:27:17.57 ID:mKgyKMR0.net]
Kが可換体でf(x)∈Kが既約ならKの任意の有限次ガロア拡大におけるf(x)の既約因子は全て同じ次数である事を示せ
という問題が分かりません。教えていただけないでしょうか。

893 名前:132人目の素数さん mailto:sage [2022/04/19(火) 18:50:32.81 ID:fNfHllwS.net]
>>881
L/Kをガロア拡大、M/Lをf(x)の完全分解体とする
g(x),h(x)をf(x)ほL(x)での規約因子とするときg(x), h(x)はGal(M/K)の作用で移り合う、(∵ g(x)の根α、h(x)の根をβとするときg(x),h(x)はα、βの最小多項式でα、βはGal(L/zk)の作用で共役)
よって主張が成り立つ

894 名前:132人目の素数さん [2022/04/19(火) 19:10:40.73 ID:mKgyKMR0.net]
>>882
これでf(x)のL上の全ての既約因子の次数が等しいことが言えたんですか?



895 名前:132人目の素数さん [2022/04/19(火) 19:11:42.66 ID:mKgyKMR0.net]
Gal(M/K)の作用で移り合うって部分がわからないです

896 名前:132人目の素数さん [2022/04/19(火) 19:14:49.37 ID:XMPzBtyf.net]
>>883
人に教えて貰っといて何その偉そうな態度

897 名前:132人目の素数さん [2022/04/19(火) 19:17:49.21 ID:mKgyKMR0.net]
すいません。

898 名前:132人目の素数さん [2022/04/19(火) 19:21:40.85 ID:mKgyKMR0.net]
>>885
α、βはGal(L/zk)の作用で共役
の部分がわからないです。zkとはなんなのでしょうか。
またどんな作用であるのでしょうか

899 名前:132人目の素数さん mailto:sage [2022/04/19(火) 20:03:30.87 ID:rE9xLsQH.net]
>>887
zはタイプミス
各係数にGal(M/K)を作用させる

900 名前:132人目の素数さん [2022/04/19(火) 20:51:47.42 ID:Re5udWBt.net]
>>882
これって
g(x),h(x)が移り合うある作用σ∈Gal(M/K)が存在するって事ですよね

Gal(M/K)の全ての元に対してg(x),h(x)は移り合うわけではないですよね

901 名前:132人目の素数さん mailto:sage [2022/04/19(火) 20:56:21.15 ID:gnosbtOT.net]
>>889
ないよ
だから書いてるやん
g(x)の根がα、h(x)の根がβ、σがα→βのときg(x)→h(x)

902 名前:132人目の素数さん [2022/04/19(火) 22:01:22.10 ID:Re5udWBt.net]
>>890
うまくg(x)の根とh(x)の根が対応する様に延長したという認識で大丈夫でしょうか

903 名前:132人目の素数さん [2022/04/19(火) 22:19:12.85 ID:XMPzBtyf.net]
>>891
ガロア拡大の定義をもう一度復習してみることを勧める

904 名前:132人目の素数さん mailto:sage [2022/04/19(火) 22:32:16.89 ID:dEFn3hsU.net]
せやな
ここまで書いてもらって行間埋められないレベルだと多分その本にで出していいレベルにないやろな



905 名前:132人目の素数さん [2022/04/19(火) 23:52:17.02 ID:Re5udWBt.net]
やっとわかった
g(x)の根がα、h(x)の根がβ、σがα→βのときg(x)→h(x)
を示す事ができた。むしろこれが証明できれば明らか。
これは自明では無いですよね。もしかして常識?

906 名前:132人目の素数さん mailto:sage [2022/04/20(水) 00:38:54 ID:dgcoUtFo.net]
常識ではないやろ
頻出のテクニックかもしれんが
というか知らなくても>>881の問題見てどうするべと2、3分考えてフットひらめかないとダメやろ
4回になって研究室のゼミとか始まって論文とか読み始めたらこの程度の問題はできて当然とばかりにビュンビュン行間飛ばしてくるからな

907 名前:132人目の素数さん mailto:sage [2022/04/20(水) 18:25:31.07 ID:TRdJ37K5.net]
微積分や線形代数の教科書は大丈夫でない人が書いているということがわかったのでこれからはもっとちゃんとした人だという噂の人たちが書いた本を読むことにします

手始めにアンドレ・ヴェイユとかジャン-ピエール・セールという人などの本を探して読むことにします

整数論入門や楕円関数の本があるそうなので私にも読めると思います

908 名前:132人目の素数さん mailto:sage [2022/04/20(水) 19:40:19 ID:WB7kMI0k.net]
実際、Amazonすらなかった時代と違って、外国語が読めない以外の理由で今和書を読む理由はない
EGAみたいな一部の本以外の、セールなどの古書でないとより良い

909 名前:132人目の素数さん mailto:sage [2022/04/20(水) 19:58:58.32 ID:KIZDLrdx.net]
>>869コピペしたらいいのに
>>873とか

910 名前:132人目の素数さん [2022/04/20(水) 20:08:59.48 ID:/IpzeaaI.net]
斎藤毅著『線形代数の世界』

行列表示を使えば、線型空間と線形写像についての問題を、ベクトルと行列についての
問題に帰着させて解くことができる。例えば、 y = f(x) をみたす x ∈ V を求めるには、
対応する連立1次方程式 b = A*a を解けば、その解 a = (a_1, … ,a_n) ∈ K^n に
対応する x = a_1*x_1 + … + a_n*x_n ∈ V が求められる。

↑のことをちゃんと証明するとすると、以下のように証明しなければなりませんよね?

y ↔ b とする。
f(x) = y に解 x が存在すれば、 x ↔ a とすると、 b = A*a である。

y ↔ b とする。
a が b = A*a をみたすとする。
x ↔ a とする。
y' = f(x) とおく。
y' ↔ b' とすると、 b' = A*a が成り立つ。
b' = A*a = b であるから、 y' = y である。
∴ y = f(x) が成り立つ。

911 名前:132人目の素数さん mailto:sage [2022/04/22(金) 22:49:31.55 ID:e0MLUOTa.net]
「群G, G'が同型であれば、群の演算にのみ依存する性質Pに関して、P(G)=P(G')である。」的な話はよくあるけど、「群の演算にのみ依存する」の辺りって数理論理学的にはどう厳密に定式化されるの?

912 名前:132人目の素数さん [2022/04/23(土) 19:16:59.20 ID:b/pvmdyR.net]
池田岳著『テンソル代数と表現論』

第2章をもう少しで読み終わります。

この章も非常に分かりやすいです。

913 名前:132人目の素数さん mailto:sage [2022/04/24(日) 00:42:58.16 ID:s7toxtS0.net]
三次方程式の解の公式をガロア理論的観点で見てみるという『環と体とガロア理論』に書いてある話に関して質問です

具体的には、
・体Kを標数0で、かつ、1でない1の三乗根を含む体とする
・f(x)は既約で、f(x)のガロア群はσ_3 (3次の置換群)
・体K上の多項式f(x) = x^3 + a_1 x^2 + a_2 x + a_3の根をα_1, α_2、α_3とし、L = k(α_1, α_2、α_3)とする
という設定で、
ガロア群と体の拡大の対応
L--{1}⊂σ_3
|  |
M--<(123)>⊂σ_3
|  |
K--σ_3
と、解の公式との関係を考えるという話に関してです。

KをMに拡大する部分は、
「f(x)の判別式をDとするとD^(1/2)は<(123)>⊂σ_3では不変で、(12)では不変でないので、M=K(D^(1/2))である」
ということが書いてあり、
MをLに拡大する部分は
「三次方程式の解の公式の形を見るとLはMに三乗根を添加したものであることが分かる」
ということが書いてあります。

KをMに拡大する部分は、
a_1,a_2,a_3の四則演算とべき根で表せて、かつ、σ_3のある元に関して不変でなく、かつ、<(123)>⊂σ_3では不変である、という元をKに添加すればいいんだなということで、
ガロア群を考えることで、解の公式を知らないという前提でもどのように体を拡大すればいいかの参考になる情報が得られていて、なるほどな、と思った一方で
MをLに拡大する部分はそういう記載はなかったので、少しもやっとしています。

KをMに拡大する部分と同じ感じで、MをLに拡大する部分について、Mに何を添加すればLになるのかを、解の公式を知らない前提で、ガロア群との対応を用いて考えることはできますか?

914 名前:132人目の素数さん mailto:sage [2022/04/24(日) 01:02:53.70 ID:s7toxtS0.net]
>>902
同じ話でもう一つ質問です
f(x)をM上の多項式と思うと、ガロア群が<(123)>なのでf(x)はM上可約だと思うのですが、これは合ってますか?

f(x) = (x - α_1)(x- α_2)(x-α_3)ですが、α_1, α_2、α_3はいずれも三乗根を含んでいるのでMに含まれず、f(x)はM上既約のようにも思えてしまうのですが



915 名前:132人目の素数さん mailto:sage [2022/04/24(日) 01:49:28 ID:eZpJNZW1.net]
>>902
3解をα、β、γ、1の原始三乗根をζとし、λ=α+βζ+γ/ζ とおけば
L = M( λ )
実際σをσ(α)=β、σ(β)=γ、σ(γ)=αであるGal(L/M)の元とすると
σ(λ) = λζからλはL\Mの元でLはM上λで生成される
一般にこのようなλはGal(L/M)が巡回群のときα+σ(α)+σ^2(α)+...で作ることができる
そのM上の最小多項式は今の場合
x^3 - λ^3=0
となる
解の公式に仕立

916 名前:トるにはこのλ^3がMの生成元である判別式の平方根(α-β)(β-γ)(γ-α)で表示してやれば良い
実際
λ^3
=α^3+β^3+γ^3 + 6αβγ
 + 3ζ(α^2β + β^2γ + γ^2α )
 + 3/ζ(αβ^2 + βγ^2 + γα^2 )
=α^3+β^3+γ^3 + 6αβγ
 + 3(ζ+1/ζ)(α^2β + β^2γ + γ^2α + αβ^2 + βγ^2 + γα^2 )
 - 3(ζ-1/ζ)(α^2β + β^2γ + γ^2α - αβ^2 - βγ^2 - γα^2 )
で前半2行は対称式なのでKの元、最後の一行は交代式なのでMの元なのでλ^3もf(x)の係数と±√Δで表示する事ができる
[]
[ここ壊れてます]

917 名前:132人目の素数さん mailto:sage [2022/04/24(日) 01:57:18.48 ID:eZpJNZW1.net]
>>903
合ってる
Mの候補としては
M=K(α), K(β)、K(γ)どれをとっても同じ
例えばK(α)をとればf(x)=x^3+px^2+qx+rとして
f(x) = (x-α)(x^2+(p+α)x+ q+pα+α^2)
と因数分解される

918 名前:132人目の素数さん mailto:sage [2022/04/24(日) 07:04:50.10 ID:P5W6dpFx.net]
菓子Aの重さと菓子Bの重さはそれぞれ独立で正規分布(10,5)と(30,10)に従う
菓子Aを4つ、菓子Bを4つ、箱に詰めた時の平均と分散はいくつか?

よろしくお願いします

919 名前:132人目の素数さん mailto:sage [2022/04/24(日) 07:25:34.47 ID:6T57fZCC.net]
https://k-san.link/normal-reproductive/

920 名前:132人目の素数さん [2022/04/24(日) 07:41:37.68 ID:BRWood23.net]
>>907
ありがとうございます

4つずつ取っても
(μ1+μ2,σ1^2+σ2^2)
と言うことですか?

(4μ1+4μ2,16σ1^2+16σ2^2)
かと思っていました

921 名前:132人目の素数さん mailto:sage [2022/04/24(日) 07:50:39 ID:ed0WovFy.net]
>>908
そのページ見てそう思うならそうなんやろ

922 名前:132人目の素数さん [2022/04/24(日) 08:05:40 ID:rN44uxC+.net]
>>909
文系なのに会社の関係で統計勉強し始めた
さっぱりわからん

助けてください

923 名前:132人目の素数さん [2022/04/24(日) 10:10:05.70 ID:ut1WHkIF.net]
Aから取り出した重さx1, x2
Bから取り出した重さy1, y2
E[x1+x2+y1+y2]=E[x1]+E[x2]+E[y1]+E[y2]
=10+10+30+30
V[x1+x2+y1+y2]=V[x1]+V[x2]+V[y1]+V[y2]
=5+5+10+10 (独立だから)

924 名前:132人目の素数さん [2022/04/24(日) 10:31:24.33 ID:Akyn0GPL.net]
>>911
ありがとう!
ありがとう!
ありがとうございます!

今後も勉強がんばります。



925 名前:132人目の素数さん mailto:sage [2022/04/24(日) 12:13:46.10 ID:s7toxtS0.net]
>>904
詳しい説明ありがとうございます、とてもスッキリしました!

>>905
>Mの候補としては
>M=K(α), K(β)、K(γ)どれをとっても同じ
ここが分かりませんでした
M=K(D^(1/2))であり、また、ガロア群の部分群と中間体は一対一に対応するので、候補が複数あるというのはおかしいのでは?という気がするのですが。
(M=K(α), K(β)、K(γ)のどれとみなすこともできる、という意味だとすると、Mがαもβもγも含んでいることになるので、M=Lになり、やはりおかしいように思います)

926 名前:132人目の素数さん mailto:sage [2022/04/24(日) 13:31:06.50 ID:DZQ3BFjO.net]
>>913
”Kにf(x)の根をひとつ添加して得られる体”は同型なものが3つできる
“同型である”と“同じ”とは違う、ここの違いを混同してはいけない
”Kにf(x)の根をひとつ添加して得られる体”はこの場合K(α), K(β), K(γ)の3つあってコレらは同型ではあるけどf(x)の分解体Lをひとつ固定して考えたとき“同型な異なる3つの体”として出てくる
それぞれ位数2の部分群<(1,2)>, <(1,3)>, <(2,3)>に対応する体として出てくる

927 名前:132人目の素数さん [2022/04/24(日) 13:42:53.02 ID:hUk4tLE9.net]
>>914
Mは<(123)>に対応してるですが

928 名前:132人目の素数さん mailto:sage [2022/04/24(日) 14:30:30 ID:t2KAYlcf.net]
すみません、>>903については勘違いでした

>>903は誤って
三次方程式f(x)が可約↔ガロア群がz/3z
と思っていて出てきた疑問だったのですが、正しくは、f(x)の係数を用いて作られる別の多項式g(x)について
g(x)が可約↔ガロア群がz/3z
でした

なので、
>f(x)をM上の多項式と思うと、ガロア群が<(123)>なのでf(x)はM上可約
というのは間違いでした

929 名前:132人目の素数さん [2022/04/24(日) 15:32:04 ID:RMn+K5ZE.net]
佐武一郎著『線型代数学(新装版)』の
pp.155-157 例3 冪零行列の標準形

佐武一郎著『線型代数学(旧装版)』の
pp.148-150 例2 冪零行列の標準形

について質問があります。

冪零行列が基底を変えることにより、標準形に変形できるところまでは分かりました。
最後の標準形の一意性のところが分かりません。

「N に相似な標準形があれば、その中に現れる(§§)の形の i 次行列の個数は明らかに

r_i - r_{i+1} = 2*m_i - m_{i-1} - m_{i+1} = rank N^{i-1} + rank N^{i+1} - 2*rank N^{i}

である。従ってそれは N によって一意的に定まる。」

と書いてあります。

なぜ標準形は一意的なのでしょうか?

930 名前:132人目の素数さん mailto:sage [2022/04/24(日) 17:37:27.64 ID:TnO6EH3c.net]
>>916
そもそも可約(reducible)と可解(solvable)がごっちゃになってないか?

931 名前:132人目の素数さん [2022/04/25(月) 18:31:28.83 ID:ddodmtQl.net]
直観主義論理を勉強しようとしてるのですが、排中律が成立しない命題の具体例というのはあるのでしょうか?

よく挙げられる、α^βが有理数となるような無理数α, βが存在することの証明では、
√2^√2が有理数であればα=β=√2となってα^βは有理数となり、
√2^√2が無理数であればα=√2^√2, β=√2とおけばα^β=√2^(√2×√2)=√2^2=2となって有理数となり、
√2^√2が有理数か無理数かわからずとも証明できてしまうことを問題視しているようですが、
実際には√2^√2は無理数なので「√2^√2は有理数(または無理数)である」という命題は排中律が成立しない命題の具体例にはなっていません

直観主義論理はあくまで排中律を使わない構成的な証明を良しとして排中律を公理から除いているだけで、実際に排中律が成立しない命題を想定しているわけではないのでしょうか?

932 名前:132人目の素数さん mailto:sage [2022/04/25(月) 19:14:10.68 ID:IPkQMB4N.net]
>>919
単に直観主義ということなら、例えば爆発律や二重否定を証明できないことが証明されてる

933 名前:132人目の素数さん mailto:sage [2022/04/25(月) 19:20:23.01 ID:IPkQMB4N.net]
>>920
間違えた
爆発律は証明できるわ

934 名前:132人目の素数さん mailto:sage [2022/04/25(月) 19:24:24.87 ID:IPkQMB4N.net]
>>921
二重否定も間違いだったわ
二重否定の導入は証明できるけど、二重否定の除去は証明できないことが証明されてる



935 名前:132人目の素数さん [2022/04/25(月) 23:51:14 ID:jWSIJ68l.net]
>>922
二重否定除去は排中律と同値で、排中律は他の公理から導けないので証明できないという話ですよね?
もう少し調べてみたらわかったのですが、どうも無限に関する命題で排中律が成立しないと想定しているみたいですね

936 名前:132人目の素数さん [2022/04/26(火) 01:15:56.34 ID:Ikaggb7R.net]
>>923
>どうも無限に関する命題で排中律が成立しないと想定している
そんなの想定してるかね
排中律が成立しないってのはただ成立しないってだけ
それを使わない証明しか認めないってことだよ
排中律が成立しない例云々より
排中律はそもそも成立していないので
君が気にするべきは
排中律なしには証明できない命題にはどんなものがあるかだろう
たとえば二重否定の除去は排中律なしには証明できないことが照明されて入るものの
それは一般の話であって
排中律なしに三重否定から否定を2つ取り除くことは排中律なしに可能
ではどんな命題が排中律なしには証明できないかといえば
まさに一般の排中律が証明できない
つまりPを命題変数としてP∨¬Pは証明できない
一方¬P∨¬¬Pは排中律なしに証明できる
(一般に古典論理で証明できる論理式のすべての命題変数を二重否定に置き換えた論理式は排中律なしに証明できるので¬¬P∨¬¬¬Pは排中律なしに証明できて
三重否定から否定を2つ取り除くのも排中律なしにできるから¬P∨¬¬Pは排中律なしに証明できる)

937 名前:132人目の素数さん [2022/04/26(火) 02:12:21.85 ID:L20ICerH.net]
>>924
そもそもは、構成的証明を掲げてる直観主義が、排中律が成立しない命題の具体例を構成しないで済ますなんてことするのだろうか?という疑問がありました

以下、自分が調べた文献(主にWikipediaですが)について載せます
https://ja.m.wikipedia.org/wiki/%E6%8E%92%E4%B8%AD%E5%BE%8B

古典数学では、「非構成的」あるいは「間接的」な存在証明があるが、直観主義者はそれを受け入れない。例えば、「P(n) が成り立つような n がある」ことを証明するとき、古典数学では全ての n について P(n) が成り立たないと仮定することで矛盾が生じることを示す。古典論理でも直観論理でも、帰謬法により「全ての n について P(n) が成り立たないということはない」ことが示される。古典論理はその結果を「P(n) が成り立つ n が存在する」に変換することを許すが、直観論理では総体として無限な自然数の集合が完全であって、P(n) となるような n が存在するということは言えない。なぜなら、直観主義では自然数が全体として完全であるとは考えないからである。[4] (Kleene 1952:49-50)

一般に、直観主義では有限な集合に関して排中律の適用を許すが、無限集合(例えば、自然数)に対しては許さない。したがって、「無限集合 D に関する全ての命題 P について、P であるかまたは P でないかのどちらかである」(Kleene 1952:48) という言い方は、直観主義では絶対できない。
Wikipediaの他にネットで公開されている大学の講義資料に、直観主義論理上の集合論であるCZF集合論は無限公理を持ち、無限集合を構成することは可能だが、無限集合に関し排中律は成立しない、という記述を見ました

938 名前:132人目の素数さん mailto:sage [2022/04/26(火) 02:33:24.30 ID:XA1ICaw0.net]
直観主義も別に構成的証明に拘ってる訳ではないけどな
メタでは排中律も使うし

939 名前:132人目の素数さん [2022/04/26(火) 07:19:31.28 ID:Ikaggb7R.net]
>>925
>大学の講義資料
URLみせて

940 名前:132人目の素数さん [2022/04/26(火) 12:19:40 ID:LZiNX85w.net]
アホしかいねぇwww

941 名前:132人目の素数さん [2022/04/26(火) 13:05:00 ID:+NmTJpA/.net]
足助太郎著『線型代数学』

馬鹿丁寧な本ですね。

942 名前:132人目の素数さん [2022/04/26(火) 13:31:12.33 ID:+NmTJpA/.net]
書きすぎと言われそうな本ですね。

943 名前:132人目の素数さん mailto:sage [2022/04/26(火) 13:48:56 ID:dpro7H9A.net]
池田岳先生は大丈夫やった?

944 名前:132人目の素数さん [2022/04/26(火) 19:09:08.70 ID:L20ICerH.net]
>>927
https://researchmap.jp/multidatabases/multidatabase_contents/detail/229222/25e68a32570612cc8bd2a4ff3d34f949
pdfの29ページで構成的集合論CZFに触れていて、30ページの注釈*4で「無限集合に関し排中律が成立しない」という記述があります



945 名前:132人目の素数さん [2022/04/26(火) 20:27:55.18 ID:L20ICerH.net]
>>932補足
直観主義論理が想定する排中律が成立しない命題について、
まず自分の手元にあった共立出版 カラー図解数学事典(dtv-Atlas Mathematik)を見てみたのですが、
以下のように記載されていました

普通に行う無限の実際的解釈では、総体として理解可能な有限集合同様、自然数の集合とその性質について語ることができる。
それに反し無限の可能的な解釈では、有限回手順での段階的構築によって到達できることしか認識できない。
(中略)
しかし、高階述語論理の不完全性により、自然数に関する真の命題で、特定の規則に従った有限回の手順では導出されないものが存在する。
ゆえに上記の後者による無限の解釈(注:無限の可能的な解釈)では、その命題が主張する自然数の性質はその肯定・否定のどちらとも認識されえない。
とすればしかし、さかのぼって排中律を無限集合に適用することを、したがって論理の2値原理をも退けねばならない。
直観主義論理は、このような全く異なった基礎の上に構築された論理体系を提示する。
それを数学に適用した場合、すべての非構成的存在証明と背理法による間接証明は失われる。
さらに、構成的に到達可能な枠組みを超えるような公理的手法は一切拒絶することになる。
(以下略)

946 名前:132人目の素数さん [2022/04/26(火) 20:28:13.98 ID:lPRNo7OA.net]
>>925
「構成的証明を掲げてる直観主義が、排中律が成立しない命題の具体例を構成しないで済ますなんてことするのだろうか?という疑問」
ここがわからない
推論規則と命題を混同していないか?

947 名前:132人目の素数さん [2022/04/26(火) 20:29:38.35 ID:L20ICerH.net]
>>933続き
これだけでは自分にはよくわかりませんでした
そこで>>932の著者の先生をネット上で知っていたので、何かしらこの件に触れている資料等はないか調べてみました
するとtogetterにて以下のような記載がありました(抜粋)
https://togetter.com/li/409585

(前略)日本でも「直観主義=実無限の否定」と信じる人が多い。けれど、それは間違っていると思う。構成的数学の枠組みとなる直観主義論理上の集合論(CZFとか)は無限公理を含んでいるし、無限公理は実無限の存在を仮定してるというのはムリのない主張ではないか。

CZFの無限公理より、任意の自然数n に対しnを表現する数値\bar{n} \in \oemga となることは証明でき、つまり集合ωが全ての数値を含む無限集合なこと自体は証明可能で二値原理が働かないのは他の元についてですが、これでは不足でしょうか。

「任意の自然数n に対しnを表現する数値\bar{n} \in \oemga となること」を実無限の存在の仮定とは誰も言わないでしょう.\forall x \in \omega (Fx) という形の文に対して一般に二値原理が成り立つかどうかという問題です.

これらを手掛かりに調べたところ>>932の資料を見つけました
カラー図解数学事典の「無限の実際的解釈」がいわゆる実無限、「無限の可能的な解釈」が可能無限のことであるとわかりました
まとめると、「直観主義=実無限の否定=自然数全体は集合Nのように扱えない体系」と信じてる人が多いけれども間違いで、
直観主義論理上の集合論CZFは無限公理を持ち、自然数全体の集合Nのような無限集合を定義できるが、無限集合に関し排中律が成立しない、
ということのようです

結論として直観主義は、自然数は上に非有界であるとだけ考えるべきで、集合Nのようにその全体を対象として(?)扱うことはできないと想定しており
直観主義論理では無限集合に関する命題で排中律が成立しないと想定しているようです

948 名前:132人目の素数さん [2022/04/26(火) 20:30:41.36 ID:L20ICerH.net]
>>935続き
ただし、>>932の先生の過去の発言に
「直観主義の立場では、排中律は無限集合を参照した途端に意味をなさなくなる」って間違いだよね
というのがありました
気になったのでさらに調べてみると、
この先生は過去に、構成的な素朴集合論における自然数全体の集合は確定的な境界を持たないことを証明したらしく、
また、非古典論理上の素朴集合論では、無限集合(典型的には自然数全体の集合ω)の境界線が確定的でなく、無限と有限の間の中間的な対象が存在することが示せる、
という発言をされていました
厳密には一概に、直観主義論理では無限集合に関する命題で排中律が成立しないとは言えないようです

949 名前:132人目の素数さん [2022/04/26(火) 20:39:01 ID:lPRNo7OA.net]
>>936
> 直観主義では「実無限」を定義できないと言う誤解もある
といった端から
> この点は、「実無限」という言葉の定義の問題である

などとあからさまな論理的詐術を働くような学者は信頼に値しない

950 名前:132人目の素数さん [2022/04/26(火) 20:53:29.29 ID:L20ICerH.net]
>>937
それは自分のここでの抜粋のせいですり替えが起こってるように感じるだけだと思います

恐らくですが>>935のtogetterの議論以前は実無限の定義を、
無限を表す名辞「自然数全体の集合」が指示する対象 ω が存在する、
としていたので、実無限は定義できると考えていたのが、

togetterの議論以降は実無限の定義には、
∀x ∈ Ω (Fx) という形の文に対して一般に二値原理が成り立つ
=無限集合に関し排中律が成立する、
もあり得ると考えるようになったので、実無限の言葉の定義による問題と記述したのだと思います

951 名前:132人目の素数さん [2022/04/26(火) 21:10:37.32 ID:LZiNX85w.net]
馬鹿の長文 休むに似たり

952 名前:132人目の素数さん [2022/04/26(火) 21:10:40.95 ID:lPRNo7OA.net]
>>938
いえ、あなたが教えてくれた

953 名前:132人目の素数さん [2022/04/26(火) 21:12:08.57 ID:lPRNo7OA.net]
いえ、あなたから教えて貰った>>932のリンク先を読んでの感想です。

954 名前:132人目の素数さん [2022/04/26(火) 21:13:14.92 ID:L20ICerH.net]
>>934
自分自身は独学してるだけの素人ですので、混同していると思います
その発言は、実際に論理的に証明すべきという意味ではなく、
>>919のように√2^√2が有理数か無理数かわからずとも証明できてしまうことを問題視しているように見えるのに、
もしそれを問題視するべきなのかをよく議論せずにいるのだとしたらお粗末な話だと思ったという程度の意味です
混乱するような発言をしてしまって申し訳ないです



955 名前:132人目の素数さん [2022/04/26(火) 21:16:52.73 ID:lPRNo7OA.net]
>>942
「素人」と言いながら既成の学問を「お粗末」と言い出す

化けの皮が剥げた

956 名前:132人目の素数さん [2022/04/26(火) 21:21:23.24 ID:L20ICerH.net]
>>943
『もし』それを問題視するべきなのかをよく議論せずにいるのだとしたらお粗末な話だけど
実際はそんなことはないだろうから、何かあるはずだと思ったということです

957 名前:132人目の素数さん mailto:sage [2022/04/26(火) 21:24:48.47 ID:3Hhic58i.net]
ネットの当てにならん情報ばっかり見てるからやろ
教科書買って読むしかないやろ

958 名前:132人目の素数さん [2022/04/26(火) 21:25:50.72 ID:5w1+8reC.net]
すいません。急に失礼します。
ちょっと趣味で地図を使ったスマホアプリ作っていて詰まったので教えてください
ある長さが与えられたときに、その長さがちょうど入るズームレベルを求めたいんですが、
その計算式が分かりますでしょうか

地図タイルについてはここに仕組みが書いてあって、
https://www.trail-note.net/tech/tile/
ズームレベルが0のとき、地球全体(幅・高さ共に40075017m)が入る正方形と考えます
それと、スマホの画面の高さと幅のPixelはプログラムから取得できます
アプリは縦画面固定で動作させるので、
つまり、ズームレベル0のときスマホの画面の高さのPixel数が地球の高さ40075017m に一致します
ここから、たとえば日本列島は大体3000000mとして、これ全体が入るズームレベルを求めるにはどういう計算をしたら良いでしょうか。
ちなみにズームレベルは整数でなく、少数で求めたいです。よろしくお願いします

959 名前:132人目の素数さん mailto:sage [2022/04/26(火) 21:27:41.45 ID:XA1ICaw0.net]
あ、この人の言ってる「排中律が成立しない命題」ってもしかして
⊢P∨¬P
が成り立たない命題Pのことか
とりあえず色々知識足りてないみたいだし、標準的な数理論理学の入門書を一冊読んでから質問した直観主義論理に触れた方がよさげ

960 名前:132人目の素数さん mailto:sage [2022/04/26(火) 21:58:15.00 ID:TOJuKQIF.net]
>>946
そのリンク先を見ると“メルカトル図法”とあるけどおそらくメルカトル図法ではない、メルカトル図法だとy軸方向に無限に伸びる
多分ミラー図法のようななんらかの方法で極が無限に飛ばないように調整してるはずだけどその調整をどうしてるのかのデータがないとわからない

961 名前:132人目の素数さん mailto:sage [2022/04/26(火) 22:01:09.41 ID:TOJuKQIF.net]
>>946
その“地図タイルデータ”は地図タイルデータザーバなりなんなりからもらってくるんでしょ?
その地図タイルデータのマニュアルに地球上の地表面をどう正方形にマップしてるのかのデータは公費されてないの?

962 名前:132人目の素数さん [2022/04/26(火) 22:07:15.04 ID:+NmTJpA/.net]
佐武一郎著『線型代数学(新装版)』の
pp.155-157 例3 冪零行列の標準形

佐武一郎著『線型代数学(旧装版)』の
pp.148-150 例2 冪零行列の標準形

について質問があります。

冪零行列が基底を変えることにより、標準形に変形できるところまでは分かりました。
最後の標準形の一意性のところが分かりません。

「N に相似な標準形があれば、その中に現れる(§§)の形の i 次行列の個数は明らかに

r_i - r_{i+1} = 2*m_i - m_{i-1} - m_{i+1} = rank N^{i-1} + rank N^{i+1} - 2*rank N^{i}

である。従ってそれは N によって一意的に定まる。」

と書いてあります。

なぜ標準形は一意的なのでしょうか?

963 名前:132人目の素数さん mailto:sage [2022/04/26(火) 22:09:42.30 ID:fEGMVdYE.net]
>>947
横からだけど、こういうのって
M |=P∨¬P
みたいなモデルMを見つける以外にできるん?
シーケント計算のカット除去みたいなのを駆使して証明の候補を除外して行ってできたりするもんなん?

964 名前:132人目の素数さん mailto:sage [2022/04/26(火) 22:15:34.10 ID:6HpwdAqE.net]
>>950
標準形とはジョルダンの標準形?



965 名前:132人目の素数さん mailto:sage [2022/04/26(火) 22:27:31.92 ID:GEuvZE+U.net]
>>946
そのページから辿れるリンクに計算式は書いてあった
しかしそもそも何がやりたいん?
少なくともグーグルの地図データザーバは
・ズームレベル,
・欲しいデータの(中央?左端?)のピクセルx座標、
・欲しいデータの(中央?上端?)のピクセルy座標、
で地図データをもらってくる仕様のようだ
zoom levelに整数でない値は指定できないみたいだけど?
引数のx座標, y座標を指定するとそのピクセル座標256個分のデータがもらえるらしい
もらえるデータがピクセル座標x0≦x≦x0+255、y0≦y≦y0+255だとして(ここ資料にない)もちろん日本の最北端、最南端、最東端、最西端がもらえるデータの端っこに合うようになってるとは限らない、そうしたいならデータ大きめにもらっといていらない分切るしかないんじゃないの?

966 名前:132人目の素数さん [2022/04/26(火) 22:30:09.51 ID:5w1+8reC.net]
ズームレベルを計算しなくても、北西と南東の緯度経度を与えたらその範囲を表示するようにカメラ位置を調整してくれる命令があったのでそれでできましたありがとうございました

967 名前:132人目の素数さん [2022/04/26(火) 22:34:51.88 ID:5w1+8reC.net]
地図データをもらってくる部分はライブラリでいい感じにやってもらえるんです
スマホの画面に地図を表示するときに、中心位置とズームレベルを指定する必要があって、
中心位置は左端と右端の中心、上端と下端の中心の緯度経度を指定すれば良いんですが、
丁度表示したい範囲が表示されるようなズームレベルを計算する方法がよくわからなかったんですよね
たぶん log とか 三角関数 とか使わないといけないんだと思うんです

968 名前:132人目の素数さん [2022/04/26(火) 22:37:04.44 ID:5w1+8reC.net]
これでできるのかなあ
ttps://ja.projecthopespeaks.org/533107-how-to-calculate-the-optimal-IPBBXF-article

969 名前:132人目の素数さん [2022/04/26(火) 22:46:46.29 ID:+NmTJpA/.net]
>>952

冪零行列のジョルダンの標準形です。

970 名前:132人目の素数さん mailto:sage [2022/04/26(火) 22:58:10.67 ID:xh0nmOsg.net]
>>957
じゃあC(n,λ)=λIn + Zn (Inはn次単位行列、Znは1がn-1個並ぶやつ)として
X = C(n1,0)⊕C(n1,0)⊕...⊕C(nt,0)
のとき
rankX^k = Σ[ni≧k](ni - k)
より成立する

971 名前:132人目の素数さん mailto:sage [2022/04/27(水) 07:11:46.21 ID:G85XSU0U.net]
>>955
そのサイトからリンク辿っていくとpixel coordinateと極座標 の変換式出てくるけど多分間違ってるな

wikipediaによると極座標(λ,φ)と地図座標(x,y)の変換式は
u = λ/180, v = atanh( sind(φ))
これで-180≦λ≦180、-90≦λ≦90が-1≦x≦1、-∞≦y≦∞に対応付けされる(ただしsind(x) = sin(πx/180)とした)
ここからuの全体が0≦256、-85.05113878≦φ≦85.05113878に対応する部分が0≦y0≦256になるように一次変換したものがz=0でのpixel coordinate(x0,y0)だから
x0 = 128×(u + 1)
y0 = 128×( atanh( sind(φ)) / atanh( sind(L)) + 1 )
今表示したい地図上の左上隅と右下隅の極座標が(λ1,φ1)、(λ2,y2)のとき上の計算式でz=0の場合のピクセル座標(x01,y01)、x02,y02)を計算する
次にx0,y0の差xd、ydとする、すなわち
xd = x02 - x01、yd = y02-y01
これの大きい方が256になるように調節したものが求めるzだから
dmax = max{ xd, yd }
z = log[2](256/dmax)

コレでいけるのではなかろか?

972 名前:132人目の素数さん mailto:sage [2022/04/27(水) 12:30:23.19 ID:6dv+aL9t.net]
このスレでも実際の内容の議論に踏み込めずに、ただ人を非難してるヤツってどうしようもなねぇな
雑談スレがお似合いだからそっち行けよ

973 名前:132人目の素数さん [2022/04/27(水) 12:36:11.67 ID:XpNkxPZ/.net]
>>928,>>939とかな

974 名前:132人目の素数さん mailto:sage [2022/04/27(水) 12:50:07.58 ID:dTVFRxE6.net]
間違ってるものを間違ってると分かるのは良いことだが、
「これこれこういう理由で間違ってる」と説明しないと分からないわな



975 名前:132人目の素数さん [2022/04/27(水) 14:33:20.05 ID:KWDQ3l+k.net]
説明しても納得させられるとは限らない罠

976 名前:132人目の素数さん [2022/04/27(水) 18:32:50.12 ID:X1DQ37NZ.net]
>>950
i 次行列の個数== rank N^{i-1} + rank N^{i+1} - 2*rank N^{i}
Nが与えられれば右辺は1つの値に決まることから、
i次行列(i次のジョルダンブロック)の個数(i=1,2..,n)も一意に決まる。
ジョルダンブロックを対角に並べた行列である標準形は(ブロックの順番の任意性を除いて)一意に決まる。

977 名前:132人目の素数さん mailto:sage [2022/04/27(水) 19:23:40.89 ID:3b0VehzA.net]
>>963
だからって理由を説明せずに批判していいなら、煽りや荒らしの免罪符になる
掲示板なんだから説明してる相手が納得できなくても、第三者が納得できた旨を書き込むとしたらまだ不毛な議論にならずに済む
第三者の意見を聞くためにも説明はした方が建設的だろ

978 名前:132人目の素数さん [2022/04/27(水) 19:48:40.68 ID:saR4xxLN.net]
>>965
>第三者が納得できた旨を書き込むとしたら
反応があればね
大概無いがな

979 名前:132人目の素数さん mailto:sage [2022/04/27(水) 20:13:05.40 ID:d8md8qLH.net]
じゃあ俺もこれからは手当り次第難癖つけることにするか

980 名前:132人目の素数さん [2022/04/27(水) 21:08:29.05 ID:F0fa6+F4.net]
>>967
●█▀█▄⋯⊶≕≍≖≎≢≣≋∺∻ブウウウウウウオオオオオオオ

koredemanzokuka?

981 名前:132人目の素数さん mailto:sage [2022/04/27(水) 21:20:20.21 ID:YJ/4xAtp.net]
>>968
スルーできないなら黙って死ねよ

982 名前:132人目の素数さん [2022/04/27(水) 21:51:55.11 ID:saR4xxLN.net]
>>925
>構成的証明を掲げてる直観主義が、排中律が成立しない命題の具体例を構成しないで済ますなんてことするのだろうか?という疑問
古典論理からすれば直観主義論理は排中律を使わない証明をするてだけ
具体的には背理法とか二重否定の除去を使えない
排中律が成立しない命題は存在しないよ
ある命題Pについて¬(P∨¬P)が成立したとしたら
古典論理でそれは¬P∧Pだから矛盾が成立することになって
論理学は破綻することに
当然ながら直観主義論理でそういう命題を構成することはできない
排中律が成立しない命題が存在しないからといって
排中律が成立するとはいえないのが直観主義論理の取る立場

983 名前:132人目の素数さん [2022/04/27(水) 21:53:22.20 ID:F0fa6+F4.net]
>>969
      /VVVVVVVVVVVVVVVVVVVVVVVVVVVVVVN\
( ・∀・)∩ ウンコビ━━━━━━━━━━━━━━━━━ム  >εε=ヽ( `Д´)ノ ウワァァァァン      
    ⊃  VVVVVVVVVVVVVVVVVVVVVVVVVVVVVVVN/

984 名前:132人目の素数さん [2022/04/27(水) 21:55:00.60 ID:saR4xxLN.net]
古典論理で証明される命題の二重否定は直観主義論理で証明できることが証明できるので
¬¬(P∨¬P)は直観主義論理で証明できる
つまり
直観主義論理でも排中律が成立しないことは無いてこと



985 名前:132人目の素数さん mailto:sage [2022/04/27(水) 22:28:02.78 ID:pe/Jnhz5.net]
このスレは以下雑談スレとなります
皆さん気軽に何でも書き込んでください

986 名前:132人目の素数さん mailto:sage [2022/04/27(水) 22:29:35.74 ID:VCKZPXoB.net]
決定不能も知らない雑魚は黙ってようね^^

987 名前:132人目の素数さん [2022/04/27(水) 22:31:05.73 ID:gQi8e6N3.net]
💩💩💩💩💩💩💩💩💩💩💩💩💩💩💩💩💩💩💩💩💩💩💩💩💩💩💩💩💩💩💩💩💩💩💩💩💩💩💩💩💩💩💩💩💩💩💩💩💩💩💩💩💩💩💩

988 名前:132人目の素数さん mailto:sage [2022/04/27(水) 22:34:06.62 ID:6dv+aL9t.net]
決定不能わかってるだけでイキってて草

989 名前:132人目の素数さん mailto:sage [2022/04/27(水) 22:38:47.94 ID:df7nJEdZ.net]
文字通りのクソスレ

990 名前:132人目の素数さん [2022/04/27(水) 22:48:11.29 ID:saR4xxLN.net]
>>976
たぶん>>974には分かってはいないだろうね

991 名前:132人目の素数さん mailto:sage [2022/04/28(木) 05: ]
[ここ壊れてます]

992 名前:07:07.74 ID:/DbX+kFA.net mailto: >>970
あんまり詳しくないんだけど、それ直観主義論理のモデルとして暗黙のうちに勝手に古典論理のモデルだけを考えてない?
「古典論理のモデルについてだけを考えている限り、全てのモデルで排中律が成立する」ってなこと言ってるように見えるんだけど
[]
[ここ壊れてます]

993 名前:132人目の素数さん [2022/04/28(木) 06:15:11 ID:37/SqDmQ.net]
>>979
どんな論理式Pについても
¬(P∨¬P)は直観主義論理でも偽となるということです

994 名前:132人目の素数さん [2022/04/28(木) 06:24:53 ID:37/SqDmQ.net]
開集合とその補集合の内包の合併の補集合の内包は空なので



995 名前:132人目の素数さん [2022/04/28(木) 19:35:48.17 ID:v4vJlTHY.net]
>>980
ハァ?

996 名前:132人目の素数さん mailto:sage [2022/04/28(木) 20:06:30.98 ID:/DbX+kFA.net]
>>980
そもそも直観主義論理に、そうやって命題に対して一つの真理値を割り当てるような意味論って存在するの?

997 名前:132人目の素数さん mailto:sage [2022/04/28(木) 20:21:07.51 ID:J2tXLzft.net]
作れなくはないでしょ?
ただそれだと完全性定理が成立するかどうかが微妙になるって事じゃないの?

998 名前:132人目の素数さん mailto:sage [2022/04/28(木) 20:24:04.00 ID:J2tXLzft.net]
イヤイヤ当たり前だな
普通のブール代数の意味論なら排中律が恒真だけど排中律は定理式でないからブール代数に意味論を制限する限り完全性は成り立たなくなる

999 名前:132人目の素数さん [2022/04/28(木) 20:27:48.24 ID:37/SqDmQ.net]
直観主義論理なのでブール代数ではないよ?

1000 名前:132人目の素数さん [2022/04/28(木) 20:28:13.62 ID:37/SqDmQ.net]
>>982
はぁ

1001 名前:132人目の素数さん [2022/04/28(木) 20:29:48.24 ID:37/SqDmQ.net]
>>983
簡単なものとしては3値論理だね

1002 名前:132人目の素数さん [2022/04/28(木) 20:31:44.17 ID:37/SqDmQ.net]
>>985
直観主義論理も完全ですよ?
完全かつ健全

1003 名前:132人目の素数さん mailto:sage [2022/04/28(木) 20:36:10.75 ID:oq75KvzG.net]
強制法勉強しようかと思ったんですけど、ここに書いてる対称性って反対称性のことですよね?
https://mathlog.info/articles/204

(∀x∈P)(∀y∈P)[x≤y∧y≤x → x=y]

1004 名前:132人目の素数さん [2022/04/28(木) 20:37:17.90 ID:v4vJlTHY.net]
排中律と矛盾律の区別すらつかんのかおまえら



1005 名前:132人目の素数さん mailto:sage [2022/04/28(木) 21:01:03.50 ID:hKts6vmM.net]
>>989
そもそもまず直観主義に基づく言語体系(コレは主義関係ない)と直観主義に基づく公理系(あるいは推論則)がある
この段階では単に「どんなものが命題と呼べますか?証明できる命題はなんですか?」のみの話でかんぜんせいも健全性もクソもない
そして各命題が意味するところの具体的な対象なり関数なり真偽値なりい対応させていく意味論を合わせていく
その際対応させる代数は“古典主義だからブール代数”、“直観主義だから当然ハイディング代数”とくるわけではない、もちろん“古典主義の理論体系にハイディング代数のモデルを対応させたらどうなるか”など考える分には構わない
もちろん直観主義理論に対してブール代数モデルをアプライしても構わない
しかし直観主義理論で意味論をブール代数に限ってしまうと「恒真なのに証明できない」命題ができてしまう、すなわち直観主義論理で完全性を保証するためには従来の古典主義の意味論、個体記号に集合、関数記号に関数を対応させる意味論では不十分だとわかる
そこで“ブール代数”の制限を緩めてより多い代数のクラスで意味論を考える必要がある
という話しがまず前提
その上で「直観主義でブール代数に値を持つ意味論はあるか?」
もちろんyes、しかし完全性を保証するには足りない

1006 名前:132人目の素数さん [2022/04/28(木) 21:01:48.63 ID:BWdqezfr.net]
>>958
>>964

ありがとうございます。

>>964

その説明は色々な本に書いてありますが、なぜそのことから一意性が成り立つことが言えるのかが分かりません。

N を冪零行列とする。
定理の証明中の手続きにしたがって、 P^{-1} * N * P = ジョルダンブロックの直和
と N を変形した場合には、途中に基底をどのように選択しても、右辺が本質的に一意的なのは分かります。

ですが、定理の証明中の手続きによらずに、 P^{-1} * N * P = ジョルダンブロックの直和
と変形できた場合にも、右辺が本質的に一意的になぜなるのかが分かりません。

1007 名前:132人目の素数さん mailto:sage [2022/04/28(木) 21:06:51.54 ID:hKts6vmM.net]
>>993
具体的な例で自分でやって見ればなぜかわかるやろ
例えば同じ6次正方行列
X=C(3,0)⊕C(2,0)⊕C(1,0)
Y=C(4,0)⊕C(1,0)⊕C(1,0)
でrank(X^k), rank((Y^k)がそれぞれどうなるかk=1,2,3入れてやって見ればいい

1008 名前:132人目の素数さん [2022/04/28(木) 21:14:06.07 ID:oq75KvzG.net]
次スレ立てました
https://rio2016.5ch.net/test/read.cgi/math/1651147986/

1009 名前:132人目の素数さん [2022/04/28(木) 21:15:18.23 ID:BWdqezfr.net]
>>994

具体例でやってみるとすると、定理の証明中の手続きにしたがって、ジョルダン標準形に変形することになります。
その場合には、ジョルダン標準形が本質的に一意的になることは理解しています。

例えば、AさんがBさんに冪零行列 N とそのジョルダン標準形と P^{-1} * N * P = ジョルダン標準形となるような P の組を知らせたとします。
Aさんがどのようにして N のジョルダン標準形を得たかは不明とします。

Bさんは、定理の証明中の手続きにしたがって、自分で N をジョルダン標準形に変形したとします。

Aさんのジョルダン標準形とBさんのジョルダン標準形が本質的に等しいことはどうやって証明するのでしょうか?

1010 名前:132人目の素数さん [2022/04/28(木) 21:30:34.05 ID:37/SqDmQ.net]
>>992
完全性の定義は
すべてのモデルで恒真であるものが証明可能
ですよ?
そして直観主義論理も古典論理同様健全かつ完全です

1011 名前:132人目の素数さん [2022/04/28(木) 21:38:54.46 ID:37/SqDmQ.net]
>>992
>もちろん直観主義理論に対してブール代数モデルをアプライしても構わない
ええっと
ブール代数はハイティング代数ですよ?

1012 名前:132人目の素数さん mailto:sage [2022/04/28(木) 21:40:27.83 ID:+gaZyQqp.net]
>>996
だからAさんが計算したらJordanの標準形がXになりました
Bさんが計算したらYになりました
そんな事が起こるのかでしょ?
もちろん答えは起こらない、なぜか、で紹介されてる話が
XとYが同じ行列Aと相似ならXとYも相似にならざるをえず、その場合任意の整数kに対してrank(X^k)とrank(Y^k)は一致しないといけないでしょ?

1013 名前:132人目の素数さん mailto:sage [2022/04/28(木) 21:40:52.09 ID:+gaZyQqp.net]
>>998
そう、だから広げてるんですよ

1014 名前:1001 [Over 1000 Thread.net]
このスレッドは1000を超えました。
新しいスレッドを立ててください。
life time: 158日 13時間 40分 8秒



1015 名前:過去ログ ★ [[過去ログ]]
■ このスレッドは過去ログ倉庫に格納されています






[ 新着レスの取得/表示 (agate) ] / [ 携帯版 ]

前100 次100 最新50 [ このスレをブックマーク! 携帯に送る ] 2chのread.cgiへ
[+板 最近立ったスレ&熱いスレ一覧 : +板 最近立ったスレ/記者別一覧](;´∀`)<316KB

read.cgi ver5.27 [feat.BBS2 +1.6] / e.0.2 (02/09/03) / eucaly.net products.
担当:undef